Sie sind auf Seite 1von 120

1.What is associated with: Starry sky pattern? Burkitt's lymphoma 2.Which organ most commonly recieves mets?

) Adrenal gland (rich blood supply

3.What is the most common testicular tumor in children? in Men? Yolk sac tumor, Seminoma 4.What is associated with: Auer's rods? Acute myelocytic leukemia (AML)-M3 5.What is associated with: Aschoff's bodies? 6.What is associated with: Birbeck granules? Rheumatic fever Histiocytosis X Alzheimer's disease Multiple myeloma

7.What is associated with: Neurofibrillary tangles? 8.What is associated with: Bence-Jones proteinuria? 9.What is associated with: Cal-Exner bodies? he ovary

Granulosa/thecal cell tumor of t Herpes virus Osteosarcoma

10.What is associated with: Cowdry type A bodies?

11.What is associated with: Codman's triangle on an x-ray?

12.What is associated with: Councilman bodies? Toxic or viral hepatitis 13.What is associated with: Calf pseudohypertrophy? rophy 14.What is associated with: Reed-Sternberg cells? 15.What is associated with: Heinz bodies? Duchenne's muscular dyst Hodgkin's lymphoma

G-6-PD deficiency Neuroblastoma Bronchial asthma (whorle Wilson's disease

16.What is associated with: Homer-Wright rosettes? 17.What is associated with: Curschmann's spirals? d mucous plugs) 18.What is associated with: Kayser-Fleischer rings? 19.What is associated with: Lewy bodies?

Parkinson's disease

20.What is associated with: Orphan Annie cells? Papillary carcinoma of the ovary 21.What is associated with: Russell bodies? Multiple myeloma

22.What is associated with: Reinke's crystals? Leydig cell tumor 23.What is associated with: Blue sclera? Osteogenesis imperfecta

24.What is associated with: Soap-bubble appearance on an x-ray? Giant cell tumor of the bone 25.What is associated with: Pseudorosettes? 26.What is associated with: Lucid interval? Ewing's sarcoma Epidural hematoma Subarachnoid hem

27.What is associated with: Bloody tap on lumbar puncture?

orrhage 28.What is associated with: Pseudopalisades? Glioblastoma multiforme Bronchial asthma (eosino Neurofibromatosi

29.What is associated with: Charcot-Leyden crystals? phil membranes)

30.What is associated with: Cafe au fait spot on the skin? s 31.What is associated with: Streaky ovaries? 32.What is associated with: Keratin pearls?

Turner's syndrome Squamous cell carcinoma

33.What is associated with: Signet ring cells? Gastric carcinoma 34.What is associated with: Mallory's bodies? 35.What is associated with: Blue-domed cysts? Chronic alcoholism Fibrocystic change of the breast Yolk sac tumor

36.What is associated with: Schiller-Duval bodies? 37.What is associated with: Senile plaques?

Alzheimer's disease

38.What is associated with: WBCs in the urine? Acute cystitis 39.What is associated with: RBCs in the urine? Bladder carcinoma 40.What is associated with: RBC casts in the urine? 41.What is associated with: WBC casts in the urine? Acute glomerulonephritis Acute pyelonephritis Acute to

42.What is associated with: Renal epithelial casts in the urine? xic or viral nephrosis 43.What is associated with: Waxy casts? Chronic end-stage renal disease 44.What is the most common: Cause of chronic metal poisoning? Lead

45.What is the most common: Cause of congenital cyanotic heart disease? Tetralog y of Fallot 46.What is the most common: Congenital cardiac anomaly? Ventricular septal defec t (VSD) 47.What is the most common: Cardiac tumor? Left atrial myxoma

48.What is the most common: Vasculitis? Temporal arteritis 49.What is the most common: Primary tumor of the liver? Hemangioma (benign) 50.What is the most common: Primary malignant tumor of the lungs? cinoma (30% to 35%) 51.What is the most common: Cause of nephrotic syndrome? ative glomerulonephritis Adenocar

Membranoprolifer Lipoid n

52.What is the most common: cause of nephrotic syndrome in children? ephrosis

53.What is the most common: Organism that causes pyelonephritis? hia coli 54.What is the most common: Renal cell cancer type? Clear cell

Escheric

55.What is the most common: Tumor of the liver? Metastatic cancer (GI, breast, l ungs) 56.What is the most common: Malignant tumor of the esophagus? rcinoma 57.What is the most common: Tumor arising within the bone? Squamous cell ca Multiple myeloma

58.What is the most common: Primary malignant tumor of the female genital tract in the world? Cervical neoplasia 59.What is the most common: Primary malignant tumor of the female genital tract in the US? Adenocarcinoma of the cervix 60.What is the most common: Tumor of the female genitourinary tract? a 61.What is the most common: Benign tumor of the ovary? Serocystadenoma 62.What is the most common: Benign tumor of the breast? Fibroadenoma 63.What is the most common: Benign lesion that affects the breast? tic change of the breast 64.What is the most common: Malignant tumor of the breast? carcinoma Fibrocys Leiomyom

Invasive ductal

65.What is the most common: Tumor in men between the ages of 15 and 35? Testicul ar tumors 66.What is the most common: Germ cell tumor in men? Seminoma Yolk sac

67.What is the most common: Testicular tumor in infants and children? tumor

68.What is the most common: Malignant germ cell tumor in women? Choriocarcinoma 69.What is the most common: Solid tumor in the body? Nephroblastoma Necrotizing ente Serocyst

70.What is the most common: Acquired GI emergency of infancy? rocolitis of infancy

71.What is the most common: Primary malignant tumor of the ovary? adenocarcinoma 72.What is the most common: Cardiac tumor of infancy? 73.What is the most common: Acute metal poisoning? Rhabdomyoma Arsenic

74.What is the most common: Proliferative abnormality of an internal organ? Benign prostatic hyperplasia (BPH) 75.What is the most common: Malignant tumor in the bone of teenagers? coma Osteosar

76.What is the most common: Site of a cerebral infarct? Middle cerebral artery 77.What is the most common: Cause of dementia between the ages of 60 and 90 year s? Alzheimer's disease 78.What is the most common: Primary CNS tumor in adults? tiforme 79.What is the most common: Primary CNS tumor in children? Glioblastoma mul Medullablastoma

80.What is the most common: Tumor on sun-exposed sites? Basal cell carcinoma 81.What is the most common: Chromosomal disorder? 1) Down syndrome (trisomy 2 Endocardial cush

82.What is the most common: Heart defect in Down syndrome? ion defect

83.What is the most common: Chromosomal disorder involving sex chromosomes? Kleinfelter's syndrome 84.What is the most common: Cardiac pathology in patients with SLE? acks endocarditis 85.What is the most common: Cause of urinary tract obstruction? BPH 86.What is the most common: Eye tumor in children? Retinoblastoma Libman-S

87.What is the most common: Intraspinal tumor? Ependymoma 88.What is the most common: Lymph node affected in non-Hodgkin's lymphoma? Periaortic lymph nodes 89.What is the most common: Renal pathology in patients with SLE? proliferative GN 90.What is the most common: Cause of cirrhosis in the USA? 91.What is the most common: Malignant tumor in women? 92.What is the most common: Cancer of the vulva? Breast Squamous cell carcinoma Yolk sac tumor Alcohol Diffuse

93.What is the most common: Testicular tumor in children? 94.What is the most common: Benign GI tumor? 95.What is the most common: Thyroid cancer? Leiomyoma

Papillary carcinoma ALL Rotavirus

96.What is the most common: Malignancy in children?

97.What is the most common: Cause of diarrhea in children?

98.What is the most common: Cause of hospitalization in children younger than 1 year of age? Respiratory syncytial virus (RSV) 99.What is the most common: Helminthic parasite worldwide? ides 100.What is the most common: Cause of anovulation? Ascaris lumbrico

Polycystic ovaries

101.What is the most common: Cause of death in neonates? tory distress syndrome (NRDS) 102.What is the most common: Cardiac anomaly in children? teriosus (PDA)

Neonatal respira Patent ductus ar

103.What is the most common: Congenital heart defect in adults? Atrial septal de fect (ASD) 104.What is the most common: Complication of PDA? arditis Subacute bacterial endoc Coarctat Amyloido

105.What is the most common: Cardiac anomaly in Turner's syndrome? ion of the aorta 106.What is the most common: Cause of restrictive cardiomyopathy? sis

107.What is the most common: Cause of pulmonary hypertension in children? VSD 108.What is the most common: Cause of reversible hypertension in the USA? Alcohol abuse 109.What is the most common: Inflammatory arthritis? Rheumatoid arthritis

110.What is the most common: Cause of spontaneous pneumothorax? Emphysematous bl eb 111.What is the most common: Cause of nonorganic pneumoconiosis? is 112.What is the most common: Cause of painless hematuria? noma 113.What is the most common: Cause of hematuria? Asbestos Renal cell carci

Infection Sickle c

114.What is the most common: Hematologic cause of papillary necrosis? ell disease 115.What is the most common: Organ involved in amyloidosis? 116.What is the most common: Cause of abnormal bleeding? Kidney

Thrombocytopenia Malaria

117.What is the most common: Cause of a nontraumatic splenic rupture? 118.What is the most common: Cause of death in SLE? Renal failure

119.What is the most common: Cause of infection for a patient on a ventilator? Pseudomonas aeruginosa 120.What is the most common: Esophageal carcinoma? Squamous cell carcinoma Alcohol abuse

121.What is the most common: Cause of chronic pancreatitis?

122.What is the most common: Cause of infectious pancreatitis? Mumps 123.What is the most common: Complication of nasogastric tube feeding? Aspirati on pneumonia

124.What is the most common learning disability?

Dyslexia

125.What is the most common: Cause of insomnia? Depression 126.What is the most common: Form of necrosis? Coagulative 127.What is the most common: Cause of blindness worldwide? matis 128.What is the most common: Cause of blindness in the USA? s{india ---vit a deficieny} 129.What is the most common: Cause of the croup? Chlamydia tracho Diabetes mellitu

Parainfluenza virus

130.What is the most common: Cause of a cold in the winter and summer? Coronavi rus 131.What is the most common: Cause of a cold in the spring and fall? us Rhinovir

132.What is the most common: Cause of viral pneumonia leading to death? RSV 133.What is the most common: Pituitary tumor? Chromophobe adenoma Sheehan's syndro Pituitary adenom

134What is the most common: Cause of panhypopituitarism? me 135.What is the most common: Cause of Cushing's syndrome? a 136.What is the most common: Kidney stone type? Calcium oxalate

137.What is the most common: Site of ischemia in the GI tract? Splenic flexure 138.What is the most common: Cause of intestinal obstructions in adults? Adhesions and hernias 139.What is the most common: Cause of neonatal bowel obstruction? rung's disease 140.What is the most common: Cause of rectal bleeding? Diverticulosis 141.What chromosomal translocation is associated with: Chronic myeloid leukemia (CML? Chromosome 9,22 (Philadelphia chromosome) 142.What chromosome: Ewing's sarcoma? Chromosome 11,22 Hirschsp

143.What chromosome: Adult familial polyposis? Chromosome 5,21 144.What chromosome: Burkitt's lymphoma? Chromosome 8,14

145.What chromosome: Acute promyelocytic leukemia (M3)? Chromosome 15,17 `146.What chromosome: Follicular lymphoma? Chromosome 14,18 Chromosome 5p

147.What chromosome is associated with: Cru di chat? 148.What chromosome: Patau's syndrome? Chromosome 13 149.What chromosome: Neurofibromatosis I?

Chromosome 17

150.What chromosome: Huntington's disease?

Chromosome 4p Chromosome 19

151.What chromosome: Familial hypercholesterolemia? 152.What chromosome: Gaucher's disease? Chromosome 1 153.What chromosome: Neimann-Pick disease?

Chromosome 11p

154.What chromosome: Tay-Sachs disease? Chromosome 15q 155.What chromosome: Cystic fibrosis? Chromosome 7

156.What chromosome: Albinism? Chromosome llp 157.What chromosome: Chronic lymphocytic leukemia (CLL)? 158.What chromosome: Marfan's disease? Chromosome 15 159.What chromosome: Neurofibromatosis II? 160.What chromosome: Down syndrome? Chromosome 22q Chromosome 12

Chromosome 21

161.What chromosome: Edward's syndrome? Chromosome 18 162.What mineral is associated with impaired glucose tolerance? Chromium (Cr) 163.What mineral is associated with hypothyroidism? Iodine (I)

164.What mineral is an important component of the enzyme xanthine oxidase? Molybdenum (Mb) 165.What vitamin deficiency has the following signs: angular stomatitis, glossit is, and cheilosis? Riboflavin (B2) deficiency 166.What vitamin is a component of the coenzyme thiamine pyrophosphate (TPP)? Thymine (Bl) 167.Avidin decreases the absorption of what vitamin? in raw egg whites. Biotin. Avidin is found

168.What are the four Ds of niacin deficiency? 1. Diarrhea 2. Dermatitis 3. Dem entia 4. Death 169.What mineral is an important component of glutathione peroxidase? (Se) Selenium

170.What mineral deficiency in children is associated with poor growth and impai red sexual development? Zinc (Zn) deficiency 171.What mineral, via excessive depositions in the liver, causes hemochromatosis ? Iron (Fe) 172.What vitamin is needed in the production of heme? Pyridoxine (B6)

173.What vitamin is a component of the enzymes fatty acid synthase and acyl CoA? Pantothenic acid 174.What vitamin deficiency has the following signs: homocysteinuria and methylm alonic aciduria? Cyanocobalamin (B12) deficiency [Folic acid deficiency h

as only homocysteinuria as a sign.] 175.What vitamin deficiency is evidenced by the following signs: poor wound heal ing, loose teeth, bleeding gums, petechiae, and ecchymosis? Ascorbic acid (v itamin C) deficiency (These are the signs of scurvy.) 176.What vitamin is given as prophylactic treatment for patients who suffer from alcoholism? Thiamine (B1)-to prevent Wernicke's encephalopathy and Korsakoff 's encephalopathy 177.What are the three carboxylase enzymes that require biotin? 1. Pyruvate 2. A cetyl CoA 3.Propionyl CoA carboxylase 178.What vitamin requires intrinsic factor (IF) for absorption? Cyanocobalamin ( B12) 179.What mineral is a component of cytochrome a/a3? Copper (Cu)

180.Leukopenia, neutropenia, and mental deterioration are signs of what mineral deficiency? Copper (Cu) deficiency 181.What vitamin deficiency causes a glove-and-stocking neuropathy seen in alcoh olics? Pyridoxine (B6) deficiency 182.What mineral deficiency involves blood vessel fragility? ciency Copper (Cu) defi

183.Megaloblastic anemia and thrombocytopenia are signs of what vitamin deficien cy? Folic acid deficiency 184.What is the antidote for an overdose with: Carbon monoxide? Oxygen What is the antidote for an overdose with: Mercury? Dimercaprol What is the antidote for an overdose with: Isoniazid? Pyridoxine What is the antidote for an overdose with: Atropine? Physostigmine What is the antidote for an overdose with: Arsenic? Dimercaprol, D-penicilla mine What is the antidote for an overdose with: Digoxin? Antidigoxin Fab fragment s What is the antidote for an overdose with: Gold? Dimercaprol What is the antidote for an overdose with: Ethylene glycol? Ethyl alcohol What is the antidote for an overdose with: Opiates/narcotics? Naloxone, naltre xone What is the antidote for an overdose with: Organophosphates? Atropine, 2-PAM What is the antidote for an overdose with: Warfarin? Vitamin K What is the antidote for an overdose with: Copper? D-Penicillamine What is the antidote for an overdose with: Heparin? Protamine sulfate What is the antidote for an overdose with: Iron? Deferoxamine What is the antidote for an overdose with: Cyanide? Amyl nitrate, sodium nit rate, or sodium thiosulfate What is the antidote for an overdose with: Methyl alcohol? Ethyl alcohol What is the antidote for an overdose with: Acetaminophen? N-Acetylcysteine What is the antidote for an overdose with: Nitrates? Methylene blue What is the antidote for an overdose with: Lead? EDTA (calcium disodium e detate), dimercaprol, succimer 1.What structure is derived from the prochordal plate? The mouth 2.What is the only organ supplied by the foregut artery that is of mesodermal or igin? Spleen

3.In which direction and how far does the gut rotate? rees

Counterclockwise 270 deg The yolk stalk (

4.What structure connects the primitive gut to the yolk sac? vitelline duct)

5.What is the artery of the embryonic foregut? The celiac artery 6.When does the primitive gut herniate out of the embryo? 7.When does it return back into the embryo? 10 weeks 6 weeks

8.What two pathologic conditions occur when the gut does not return to the embry o? Omphalocele and gastroschisis 9.Around what structure does the midgut rotate? Superior mesenteric artery 10.What three things cause the indifferent gonad to become a testis? 1.Testis -determining factor (TDF) from the short arm of the Y chromosome 2.Miillerian in hibiting factor (MIF) from Sertoli cells 3.Testosterone from Leydig cells 11.Where does the embryologic foregutend? m At the first part of the duodenu

12.What is the artery of the embryonic hindgut? The inferior mesenteric artery 13.What three embryonic cell layers form the chorion? mcytiotrophoblast 3. Extraembrvonic mesoderm 1. Cytotrophoblast 2. Sy

14.Which neuropore closes last? Caudal-and it is the first to open, too. 15.What is the artery of the embryonic midgut? The superior mesenteric artery 16.From where are nephrons derived embryonically? Metanephros

17.What are the five derivatives of the ventral mesentery? 1. Falciform lig ament 2. Hepatoduodenal ligament 3. Hepatogastric ligament 4 and 5. Coronary and triangular ligaments of the liver. All else is derived from the dorsal mesenter y. 18.When do the septum primum and the septum secundum of the heart fuse? After bi rth 19.The cerebral cortex is a derivative of what? The telencephalon 20.What is the adult structure found Ligamentum teres What is the adult structure found in um venosum What is the adult structure found in ule What is the adult structure found in Ligamentum arteriosum What is the adult structure found in Medial umbilical ligament in the embryo as the: Umbilical vein? the embryo as the: Ductus venosus? Ligament the embryo as the: Foramen ovule? Fossa ov the embryo as the: Ductus arteriosus? the embryo as the: Umbilical artery?

21.Where does the hindgut end? At the superior portion of the anal canal 22.Where does the midgut end? At the right two thirds of the transverse colon

23.From where is the tongue musculature derived?

Occipital somites

24.What two branchial arches contribute to the formation of the anterior two thi rds of the tongue? First and some of the second 25.What are the two fourth pharyngeal pouch derivatives? roid glands and the ultimobranchial body Superior parathy

26.What two structures are derived from the fourth aortic arch? Arch of the aort a and the right subclavian artery 27.What adult structures are derived from preotic somites? nternal eye 28.What structure is derived from the first pharyngeal pouch? Muscles of the i The middle ear

29.What two branchial arches contribute to the posterior two thirds of the tongu e? Third and part of the fourth 30.What are the two third pharyngeal pouch derivatives? Inferior parathyroid gla nds and the thymus 31.What structure is derived from the first aortic arch? 32.From what are the urinary bladder and the urethra derived? Maxillary artery Urogenital sinus Truncus

33.From what are the pulmonary trunk and the ascending aorta derived? arteriosum

34.What disorder will result when there is a failure of the urachus to close, ca using a leakage of urine out of the umbilicus? Urachal fistula 35.The common carotid and the internal carotid arteries are derivatives of what embryonic structure? Third aortic arch 36.The palatine tonsils are derived from what embryonic structure? haryngeal pouch Second p

37.What are the sixth aortic arch derivatives? Right and left pulmonary arterie s and the ductus arteriosus 38.The stapedial artery is derived from what? Second aortic arch The collecting d The left

39.The mesonephric ducts contribute to what renal structures? ucts, calyx, renal pelvis, and ureters

40.Of what embryonic structureis the coronary sinus a derivative? horn of the sinus venosus 41.What cranial nerve (CN) CN V What cranial nerve (CN) is What cranial nerve (CN) is What cranial nerve (CN) is What cranial nerve (CN) is degenerates What cranial nerve (CN) is

is associated with the: First pharyngeal arch? associated associated associated associated with with with with the: the: the: the: Second pharyngeal arch? Third pharyngeal arch? Fourth pharyngeal arch? Fifth pharyngeal arch? CN VII CN IX CN X None-it

associated with the: Sixth pharyngeal arch? CN X

42.From where is the external auditory meatus derived? First pharyngeal groove

43.From where is the smooth portion of the right atrium derived? rn of the sinus venosus 44.Meckel's diverticulum is a remnant of what embryonic structure? e duct (yolk stalk)

Right ho Vitellin

45.The pons and cerebellum are derived from what portion of the embryonic neural tissue? Metencephalon 46.The medulla is a derivative of what portion of the embryonic neural tissue? Myelencephalon 47.What structure "tells" the overlying cells to begin neurulation? chord The noto

48.What structure splits the cloacal membrane, resulting in the formation of the perineum? Urorectal septum 49.In the adult, the thoracic veins are derived from what structure? inal veins The card

50.The gastrointestinal tract and abdominal veins are derived from what structur e? Vitelline veins 51.From what is the thyroid gland derived? osterior aspect of the tongue) The floor of the endoderm (the p The dien

52.The thalamus and its related structures are derivatives of what? cephalon 1.Transcriptionally active DNA is known as what? 2.Transcriptionally inactive DNA is called? Euchromatin Heterochromatin

3.What is the only histone not found inside the nucleosomes? unction is to bind nucleosomes together. 4.What coating protects proteins from intracellular degradation? coating 5.A nucleosome is made up of what two components?

H1 histone-Its f Clathrin

Histories and DNA

6.What are the four functions of smooth endoplasmic reticulum (SER)? 1. Stero id synthesis 2. Drug detoxification 3. Ca2+ handling 4. TAG resynthesis 7.What are the long microvilli found in the inner ear and the male reproductive tract called? Stereocilia 8.What cell junction type allows for communication between two adjacent cells? Gap junctions (nexus) 9.Where are the enzymes for ATP production and the ETC located? Inner fold of th e mitochondria membrane 10.What cell membrane structure increases the surface area of a cell and has act in randomly assorted within its structure? Microvillus 11.What is the function of desmosomes? To hold adjacent cells together (i.e., a dhesion)

12.What is the microtubule configuration of a basal body? e arrangement

9 + 0 microtubul

13.What are the four components of the basement membrane? 1. Laminin 2. He paran sulfate (heparitin sulfate) 3. Fibronectin 4. Type IV collagen 14.The proteins that are to stay within the cell are produced by what organelle? Free polysome (polyribosome) 15.What is the lysosomal post- translational modification of proteins? Phosphor ylation of mannose residues 16,What cell surface modification of ependymal cells and respiratory epithelium has a 9 + 2 microtubular configur- ation and movement as its function? Cilia 17.What protein binds hemidesmosomes to the basal lamina? 18.What intermediate filament is found in the zona adherens? Integrin Actin

19.The proteins to be exported or incorporated into the lysosome are produced by what organelle? Bound polysome (polyribosome), attached to rough endopla smic reticulum (RER) 20.What is the function of the zonula occludens and the zonula adherens? To provide attachment between contiguous cells and to maintain a semipermeable b arrier The following intermediate ? Muscle cells The following intermediate ratins? Epithelial cells The following intermediate in? Mesenchymal cells The following intermediate ilaments? Neurons The following intermediate filaments? Astrocytes filaments are associated with what cell types: Desmin filaments are associated with what cell types: Cytoke filaments are associated with what cell types: Viment filaments are associated with what cell types: Neurof filaments are associated with what cell types: Glial

21.The basal lamina + the reticular lamina = what?

The basement membrane Rough endoplasmi

22.What is the name of the organelle where collagen is made? c reticulum (RER)

23.What vitamin is needed for the hydroxylation of proline and lysine in collage n synthesis? Vitamin C - {DEFICIENCY--Survy} 24.What are the two amino acids that cross-link elastin molecules? e and isodesmosine 25.What is the major inorganic component of bone? Hydroxyapatite Osteocla Desmosin

26.What cell in bone is a part of the mononuclear phagocytic system? sts

27.What are the two types of cells located in the perichondrium of cartilage? Fibroblasts and chondroblasts

28.What cell in the CNS is part of the mononuclear phagocytic system? a{CNS PHAGOCYTE} 29.What substance, found in eosinophils, is toxic to parasitic worms? sic protein

Microgli Major ba

30.Where is tropocollagen aggregated to form a collagen fibril? Extracellularly 31.What cell surface extension allows osteocytes in the lacunaeto "talk" to each other? Canaliculi 32.What cell type produces myelin in the CNS? Oligodendrocytes All four ventricles

33.In which ventricles is/are choroid plexus found? 34.What muscle type has calmodulin? Smooth muscle

35.What element is needed for the proper alignment of the tropocollagen molecule s? Copper (Cu+) 36.What is added to the procollagen molecules to prevent intracellular precipita tion? Registration peptides 37.In what tissue can you find intercalated disks? Cardiac muscle

38.What are intercalated disks? Dense bands containing intercellular junctions t hat link adjacent cells mechanically and electrically 39.Of what are intercalated disks composed? mes Gap junctions Fascia adherens (mainly) Desmoso Chondroitin sulfate and

40.What are the proteoglycans of cartilage and bone? keratan sulfate

41.What is the only glycosamino- glycan (GAG) that binds to the linker portion o f the proteoglycan? Hyaluronic acid (all sulfates bind to the core portion) 42.What is the portion of an axon that lacks myelin and is rich in Na+/K+ pumps? Node of Ranvier 43.What type of CNS cells have cilia, line the ventricles, and contribute to the blood-brain barrier? Ependymal cells 44.What are the largest glial cells in the CNS (Hint: They contribute to the blo od- brain barrier.)? Astrocytes 45.Myelin is produced by what type of PNS cells? Schwann cells

46.What is the dominant cell type in the lacunae of cartilage? Chondrocytes 47.What structure runs perpendicular to the Haversian canals in the bone? Volkmann's canal 48.What are the three reasons for the effectiveness of the blood-brain barrier? 1. Tight junctions 2. Capillaries that lack fenestration 3. Very selective pinoc ytosis by the capillaries 49.What types of muscle have troponin? Skeletal and cardiac 50.What type of muscle is uninuclear and nonstriated? Smooth muscle - it lacks

T tubules and has gap junctions 51.On what area of the spleen are the APC{antigen presenting cells}s located? Marginal zone 52.What is the dominant cell type in the red pulp of the spleen? d cells 53.On what area of the lymph node can you locate plasma cells? Medulla 54.What is the name of the area in the thymus where T cells are produced? Hassall's corpuscle 55.What type of muscle is striated and multinuclear? tubules and SR at the A-I junction Skeletal muscle- it of T White pu Red bloo

56.In what region of the spleen are the germinal centers located? lp-where B cell differentiation takes place 57.What layer of the skin is missing in thin skin? 58.What are the phagocytic cells of the GI tract called? neth's granular cells) Stratum lucidum

Paneth cells (Pa

59.Which immunoglobulin is secreted by the plasma cells in the GI tract? IgA 60.In what area of the spleen are the T cells located? Periarterial lymphatic s heath (PALS) 61.In what area of the lymph node are the T cells and the APCs located? Paracort ical (thymic-dependent) area 62.What type of muscle is striated, branched, and uninuclear? yadic T tubules with SR at the Z line Cardiac muscle-d An inter

63.What does the tunica intima of arteries have that veins do not? nal elastic lamina 64.What two layers of skin makeup the malphighian layer? and spinosus (mitotic area)

Stratum basalis

65.What type of skin cells have the mature melanin granules? Keratinocytes. M elanocytes inject melanosomes into the keratinocytes and mature there. 66.What type of skin cells are part of the mononuclear phagocytic system? Langerhans' cells 67.What cells of the epidermis, derived from the neural crest, act as mechano- r eceptors? Merkel's cells (Merkel's tactile cells) 68.What layer of the epidermis acts as a sealant to protect against desiccation? Stratum granulosum 69.What layer of the skin is composed of non-nucleated cells full of keratin? Stratum corneum 70.What are the antigen- presenting cells in the Peyer's patches of the GI known as? M cells

71.What are the three "tunica" layers of a blood vessel wall? 2. Tunica media 3. Tunica adventitia

1. Tunica intima

72.Within what layer of the heart are the nerves and conducting fibers located? Subendocardial space 73.Which organs have fenestrated capillaries with diaphragms? es, Endocrine organs Kidney, Intestin

74.What secondary lymphoid tissue is encapsulated and has germinal centers? Peyer's patches{duodenum} 75.What organs have sinusoid capillaries (leakiest type)? ow, Spleen 76.What type of vessel has a thick tunica media? a thick tunica adventitia.} 77.Is the spleen a capsulated organ with trabeculae? have cortical or medullary regions Arteries Liver, Bone marr {Veins have

Yes-although it does not

78.What secondary lymphoid organ is found just below the stratified squamous epi thel- ium and is partially capsulated? Tonsils 79.What type of capillary lacks fenestrations and has pinocytotic vesicles? Continuous capillary 80.What region of the body has fenestrated capillaries without diaphragms? Kidney glomeruli. 81.Does the thymus have germinal centers? ted with B cells. No. Germinal centers are associa

82.What cell of the liver is part of the mononuclear phagocytic system? Kupffer cells 83.What papillae are respons- ible for sweet taste? Circumvalate papillae

84.What are the three epi- dermal derivatives? 1. Nails 2. Hair 3. Sweat glands (both apocrine and sebaceous) 85.What papillae send their senses via chorda tympani of CN VII? m papillae 86.What is the area of mitotic activity in the GI tract? eberkuhn Fungifor The crypts of Li

87.What cells of the GI tract secrete HCl and intrinsic factor? Parietal cells o f the stomach 88.What type of sweat gland is under cholinergic stimulation? Eccrine gland

89.What gland produces a serous secretion that is approximately 20% of the total saliva produced? Parotid gland 90.What papillae are touch receptors on the tongue and send their sensations via CN V3 (mandibular division)? Filiform papillae 91.Secretin and CCK are produced in what portion of the GI tract? testine Small in

92.What cells of the stomach secrete pepsinogen?

Chief cells

93.What cell type produces dentin of the teeth? Odontoblasts (neural crest) 94.What cell type produces enamel of the teeth? Ameloblasts (ectoderm) 95.What gland produces 70% of the total salivary secretions and is a mixture of serous (predominantly) and mucous alveoli and secretory units? Submandibular gl and 96.What type of cells of the respiratory system secrete surfactant? pneumocytes Type II

97.What zone of the liver is the first to be affected in times of hypoxia? Central region (around the central vein) 98.In what region of the respiratory system do you first see Clara cells? Terminal bronchioles 99.What substance does the juxtaglomerular cells of the kidney secrete in respon se to low blood pressure? Renin 100.In what region of the liver is fat stored? Space of Disse -{Ito cells and V it A also present there} 101.In what region of the GI tract does exfoliation take place? At the tip of th e villi 102.What are the mucus secreting cells in the respiratory tract above the level of the terminal bronchioles? Goblet cells 103.What cells of the distal convoluted tubule are sensitive to the low ion cont ent of the urine? Macula densa 104.What cells comprise 95% of the alveolar surface and are responsible for gas exchange? Type I pneumocytes 105.What are the two hormones secreted by the posterior pituitary gland via the para- ventricular supranuclei? ADH and oxytocin 106.What area of the nephron is impermeable to water? op of Henle 107.What region of the kidney is affected by ADH? r reabsorption- but the DCT is also affected by ADH Ascending limb of the lo Collecting duct for wate Thick as

108.What portion of the kidney actively pumps Cl- out of the tubule? cending limb of the loop of Henle

109.What region of the kidney has a countercurrent multi- plier producing a grad ient of hypertonicity in the tubule lumen? Loop of Henle 110.What region of the liver is first affected in toxic doses of drugs? Peripher al zone (because extraction of substances occurs there first) 111.What are the two acidophilic hormones secreted by the adenohypophysis? GH and prolactin " 112.In what region of the kidney does the greatest extraction of nutrients occur

Proximal convoluted tubule (-66% of nutrient extraction occurs here)

113.What cells of the thyroid gland secrete calcitonin? Parafollicular C cells 114.What cells of the adrenal gland are neural crest derivatives? in cells (adrenal medulla) 115.What cells secrete glucagon? Chromaff

Alpha cells of the islet of Langerhans

116.What hormone inhibits glucagon release and pancreatic exocrine secretions? Somatostatin 117.What hormone causes milk letdown? Oxytocin

118.Low levels of what hormone stimulates the uterus to go into its proliferativ e stage? Estrogen 119.What cells of the genito- urinary system secrete testosterone? ells (stimulated by LH) Leydig c

120.What is the mucus-secreting gland in the male reproductive system? Bulboure thral glands (Cowper's glands) 121.Elevated levels of what hormone cause the endometrium to enter the secretory phase of the female cycle? Progesterone 122.What are the cells of the parathyroid gland that produce parathyroid hormone (PTH)? Chief cells 123.What hormone produced during the night causes a decrease in gonadal function ? Melatonin 124.Where is melatonin produced? Pineal gland

125.What are the four basophilic hormones released from the adenohypophysis? 1. Adrenocorticotrophic hormone (ACTH) 2. Thyroid-stimulating hormone (TSH) 3. L uteinizing hormone (LH) 4. Follicle-stimulating hormone (FSH) 126.What cells form the blood-testis barrier? Sertoli's cells

What is the chromosome number of-G1? 46 (2n) What is the chromosome number of-S phase? 46 (4n) What is the chromosome number of-G2? 46 (4n) What is the chromosome number of-Mitosis? 46 (4n) to 46 (2n) What is the chromosome number of a primary spermatocyte? 46 (4n) In females, meiosis is arrested twice - when and at what stages of meiosis? 1. First, in utero at prophase I, 2. Second, at ovulation in metaphase II What must occur for an egg to complete ovulation? It needs to be fertilize d by a sperm. If it is not, the egg is released in metaphase II and meiosis is i ncomplete. What is the chromosome number at the end of meiosis I? 23 (2n)-it is the reduct ive phase of meiosis. 127.What cell is under control of FSH and testosterone; secretes inhibin, MIF, a nd androgen-binding protein; and phagocytizes the excess cytoplasm of the sperma tid? Sertoli cell 128.What is the major androgen released from the zona reticularis? piandrosterone (DHEA) Dehydroe

129.What hormone causes an increase in the accumulation of adipose and collageno us tissue of the breast and an increase in the branching of the ducts of the bre ast? Estrogen 130.What promotes further prolactin and oxytocin release? 131.What part of the placenta is derived from the mother? 132.What is the most common site of fertilization? tube Suckling Decidua basalis

Ampulla of the fallopian

133.What is the only cranial nerve that comes off the dorsal surface of the brai n stem? CN IV 134.What type of fiber is carried in the dorsal root? y only 135.How would a lower motor neuron (LMN) lesion present? siculations and flaccid paralysis (always ipsilateral) Sensory or motor. Sensor Hyporeflexia, fa

136.What is the name of the brain stem tract in which the dorsal columns run? Medial lemniscus 137.What is the ability to tell what something is without looking at it and usin g only your hands? Stereognosis 138.In what tract do pain and temperature fibers run? Spinothalamic tract

139.What gyrus in the cerebral cortex receives information from fibers of the do rsal column tract? Postcentral gyrus 140.What area of the brain is responsible for contralateral gaze? eye field (Brodmann area 8) 141.What is the thalamic relay nucleus for the visual system? te body (LGB) 142.What is the function of the ossicles? ound Frontal

Lateral genicula

They increase the intensity of s Ciliary muscle Fovea (it is mad Cones (Cones and color)

143.What muscle in the eye is responsible for accommodation? 144.What area of the eye has the greatest visual acuity? e up soley of cones) 145.What cell type in the eye is for color vision?

146.If there is macula sparing in a visual deficit, where is the lesion? In the occipital lobe of the cerebral cortex (optic radiations) ... 147.Which way do the eyes drift in a frontal eye field lesion?To the side of the lesion 148.What is the thalamic relay nucleus that CN V needs to "speak" to in order to pass its information on to the cerebral cortex?Ventroposteromedial(VPM) 149.Cell bodies of what fibers are found in the mesencephalic nucleus of CN V?Pr oprioception of the face (CN V) and motor (jaw jerk reflex) 150.If a patient presented with an LMN lesion in CN V, CN VII, or CN XII, what w

ould you see? Ipsilateral paresis 151.What is the motor relay nucleus of the thalamus?Ventrolateral (VL) nucleus o f thalamus 152.What is the only cell type to leave the cerebellum?Purkinje (inhibitory) - G ABA 153.If a patient presented with a right-sided cerebellar lesion, which way would the patient fall if he closed his eyes? To the right 154.What is the function of the superior olivary nucleus? To localize and detemi ne the nature of sounds (Sound and superior start with S.) 155.If a patient presents with a left nystagmus, where is the lesion? On the rig ht, because the nvstagmus is named for the fast component, and the fast componen t is to the unaffected side. 156.What region of the cerebellum is responsible for the planning of movements? Cerebellar hemisphere 157.What is the thalamic relay nucleus for the limbic system? Anterior nucleus 158.What fluid is found in the anterior chamber of the eye? Aqueous humor 159.What is the dividing line between the anterior and posterior chambers of the eye? The lens 160.If there is a total anopsia of the left eye, where is the lesion? Optic nerv e of the left eye 161.What is the center for ipsilateral gaze? The paramedian pontine reticular fo rmation (PPRF) 162.What fluid of the inner ear has an electrolyte content like that of the extr acellular fluid compartment (ECF)?Perilymph 163.What is the thalamic relay nucleus for the auditory system? MGB 164.What region of the cerebellum is responsible for balance and eye movement? Flocculonodular lobe 165.What is the only cell in the cerebellum to have an excitatory neurotransmitt er? Granule cell 166.What does the nystagmus look like if cold water is placed in the right ear? Slow drift to the right, fast drift to the left COWS = Cold Opposite - Warm Same (named in reference to the fast component) 167.Information from the cerebellum leaves via what? ncle Superior cerebellar pedu

168.In what portion of the internal capsule are you if you can see the caudate n ucleus? Anterior limb 169.What type of memory is lost in a hippocampal lesion? Long-term memory

170.In what region of the brain stem does the corticospinal tract cross over? Medullary decussation

171.From what gyrus of the brain does the corticospinal tract originate? Precentral gyrus 172.What type of fibers are carried in the ventral rami? Both sensory and motor (from the spinal nerve on both sensory and motor fibers) 173.What are the hallmark signs of an upper motor neuron (UMN) lesion? 1. Hyper reflexia 2. Spastic paralysis 3. Positive Babinski sign 174.What tract carriers fibers for voluntary refined movements of the distal ext remities? Corticospinal tract 175.What is the name of the tract in which the dorsal columns from the lower ext remities run? Fasciculus gracilis (It is medial of the two tracts on a cross-s ection of the spinal cord; the lateral tract is the fasciculus cuneatus.) Rememb er: Lower extremities dancing-graceful-gracilis. 176.What is the function of the superior colliculi? Cell bodies that are to be relayed to the thalamus for sight are found there. (Sight and superior start with S.) 177.In order for sensory information from the dorsal columns and the spinothalam ic tract to get to the cerebral cortex, they must use what thalamic relay nucleu s? Ventroposterolateral (VPL) 178.In which region of the spinal cord does the spinothalamic tract cross over? Ventral white commissure (VWC) 179.Sensory information from the spinothalamic tract sends its information to wh at region of the cerebral cortex? Postcentral gyrus 180.In which region of the brain stem do the dorsal columns cross over? Lower me dulla (synapse on nucleus gracilis or cuneatus) 181.What tract carries conscious proprioception,fine touch, two-point discrimina tion, and vibratory sense? Dorsal column tract (all senses except pain and temperature) 182.What tract of the spinal cord carries dorsal column information from the upp er extremities? Fasciculus cuneatus 183.If the right side of the corticobulbar tract to the muscles of facial expres sion were damaged, where would the deficit be seen? In the contralateral low er face (left) 184.If the corticobulbar tract for CN V and CN XII were cut on the right side, w here would the lesion be? There would be no deficit, because the corticobu lbar tract receives bilateral input. 185.What type of fibers are carried in the ventral root? 186.What peduncle(s) carry information into the cerebellum? dle cerebellar peduncles Motor only Inferior and mid Touch, p

187.Cell bodies of what fibers are found in the trigeminal ganglion? ain, and temperature

188.When the head moves, what causes the eyes to move in the opposite direction? Vestibular system

189.Unconscious proprioception, body sense, and motor execution are functions of what part of the cerebellum? Vermis and intermediate lobe 190.What three structures contain perilymph? uli 3. Semicircular canals 1. Scala tympani 2. Scala vestib

191.The gravity receptors for changes in the position of the head are located in what part of the inner ear? Saccule and utricle 192.What cells are for black and white vision (night vision)? Rods

193.What is the fluid of the posterior compartment of the eye? Vitreous humor 194.What type of fluid in the inner ear has the consistency of intracellular flu id (ICF)? Endolymph (high levels of K+) 195.Name three lesions that can cause left homonymous hemianopsia? 1. Lesio n of the right optic tract 2. Lesion of the lateral geniculate body (LGB) 3. Les ion of the optic radiation 196.What lesion produces a tremor upon movement? A cerebellar lesion

197.What part of the inner ear is sensitive to angular acceleration and decelera tion? Semicircular canals 198.What is the normal volume of CSF? ly} Approximately 140 ml{135ml most accurate Sphincter pupill CN VI (o

199.What muscle of the eye is under parasympathetic control? ae (part of iris)

200.What cranial nerve receives sensory information from the cornea? phthalmic division)

201.What artery supplies blood to the trunk and the lower extremities on a homun culus map of the cerebral cortex? Anterior cerebral artery 202.What structures of the inner ear contain endolymph? 1. Scala media 2. Semici rcular ducts 3. Saccule Utricle 203.With what type of lesions do you see tremors at rest? asal ganglia 204.What muscle of the eye is under sympathetic control? (part of the iris) Lesions of the b Dilator pupillae

205.Where is the lesion if the patient presents with a right nasal hemianopsia? Right internal carotid artery compression on the optic chiasm 206.What part of the inner ear functions in head movement? ts Semicircular duc

207.What part of the internal capsule are you in if you see the thalamus? Posterior limb of the internal capsule 208.What region of the basal ganglia is affected in Parkinson's disease? Substantia nigra (degeneration) 209.Hemorrhagic destruction of the contralateral subthalamic nuclei results in w hat disorder? Herniballismus (wild flailing movements)

210.Slow writhing movements (athetosis) are caused by what? Hypermyelinizati on of the corpus striatum and the thalamus (seen in cerebral palsy) 211.Atrophy of the striatum of the basal ganglia results in what? involuntary quick movements) Chorea (

212.What tracts are found in the genu of the internal capsule? Corticobulbar tr acts 213.What tracts are found in the posterior limb of the IC? inothalamic Dorsal column Thalamocortical 215.What tracts are found in the anterior limb of the IC? tracts Corticospinal Sp Thalamocortical

216.If warm water is placed in the right ear, what does the nystagmus look like? Slow drift to the left and fast drift to the right (COWS = Cold Opposite & Warm Same) 217.What lesion causes a bitemporal hemianopsia? Optic chiasm lesion Broca's area

218.What area of the brain is known as the motor speech area? 219.What does Meyer's loop lesion cause? r quadrantopia

Contralateral homonymous superio

220.Blood supply to the head/neck area and the upper limb on a homunculus map in the cerebral cortex comes from what artery? Middle cerebral artery 221.What area of the brain is known for language comprehension? Wernicke's area 222.Where is the lesion if the patient presents with a right homonymous inferior quadrantanopia? Left upper loop lesion 223.What region of the cerebellum is affected if a patient has dystaxia of the l egs and trunk during walking? Anterior vermis (It is most commonly caused by c hronic alcohol abuse.) 224.Where is the lesion in a patient who presents with a broad-based gait, hypot onia, intention tremors, nystagmus, and ataxia? Cerebellum 225.What are the functions of the external auditory meatus? and protection of the tympanic membrane 226.What is the function of the inferior colliculi? itory input and arrange the input tonotopically Sound collection

To receive bilateral aud

227.If a patient presents with nystagmus, dystaxia, and hypotonia of the ipsilat eral limbs, what area of the cerebellum is affected? Hemisphere 228.What lesion of the cerebellum is usually caused by an ependymoma or medullob lastoma, resulting in dystaxia of the trunk with an inability to maintain an upr ight posture? Posterior vermis lesion 229.What spinal cord injury results in flaccid paralysis and muscle atrophy? Polio (bilateral ventral horn lesion) 230.What spinal cord lesion results in a lower motor neuron (LMN) lesion at the level of the lesion and an upper motor neuron (UMN) lesion below the level of th e lesion? Amyotrophic lateral sclerosis (ALS)-Lou Gehrig's disease

231.What arterial occlusion would result in a loss of all tracts in the spinal c ord except the dorsal columns? Anterior spinal artery occlusion (posterior spin al artery supplies the dorsal columns) 232.What spinal cord lesion results in a bilateral loss of pain and temperature at the level of the lesion? Syringomyelia (VWC lesion) 233.What spinal cord lesion causes a bilateral dorsal column loss below the leve l of the lesion? Tabes dorsalis (neurosyphilis) 234.What disease is associated with demyelination of the dorsal column, spinocer ebellar tract, and corticospinal tract? Subacute combined degeneration 235.What arterial occlusion results in contralateral spastic hemiparesis, contra lateral spastic lower face, and ipsilateral oculomotor palsy (dilated, ptosis, e ye down and out)? Posterior cerebral artery occlusion (ventral midbrain sy ndrome) 236.What syndrome is associated with an ipsilateral UMN lesion below the level o f the lesion, ipsilateral dorsal column loss at and below the level of the lesio n, an LMN lesion at the level of the lesion, bilateral loss of pain and temperat ure at the level an Brown-Sequard syndrome (heimisection of the spinal cord) What arterial occlusion results in the following syndromes (Name artery and spec ific region.): Contralateral spastic hemiparesis of the body? Vertebral artery -pyramid What arterial occlusion results in the following syndromes (Name artery and spec ific region.): Contralateral loss of position and vibration? Vertebral artery -medial lemniscus What arterial occlusion results in the following syndromes (Name artery and spec ific region.): Ipsilateral paralysis of the tongue? Vertebral artery-CN XII What arterial occlusion results in the following syndromes (Name artery and spec ific region.): Ipsilateral limb ataxia? Anterior inferior cerebellar artery-infe rior cerebellar peduncle What arterial occlusion results in the following syndromes (Name artery and spec ific region.): Ipsilateral pain and temperature loss of the face? Anterior inferior cerebellar artery-spinal nucleus of CN V What arterial occlusion results in the following syndromes (Name artery and spec ific region.): Contralateral pain and temperature of the body? Anterior inferio r cerebellar artery- spinotbalamic tract What arterial occlusion results in the following syndromes (Name artery and spec ific region.): Nystagmus away from the lesion? Anterior inferior cerebellar art ery- vestibular nuclei What arterial occlusion results in the following syndromes (Name artery and spec ific region.): Ipsilateral Horner's syndrome? Anterior inferior cerebellar art ery- descending autonomics What arterial occlusion results in the following syndromes (Name artery and spec ific region.): Ipsilateral facial paralysis? Anterior inferior cerebellar art ery-CN Vll What arterial occlusion results in the following syndromes (Name artery and spec ific region.): Deafness? Anterior inferior cerebellar artery-CN VIll 1.What is the name for the most prominent spinous process in the spine? Vertebra prominens (C7 in 70% of cases, C6 in 20%, T1 in 10%) 2.What portion of the intervertebral disk is a remnant of the notochord? Nucleus pulposus

3.What three muscles comprise the erector spinae? ssimus 3. Spinalis

1. Iliocostalis 2. Longi

4.What are the names given to the first and second cervical vertebrae? C1-atlas C2-axis 5.To what vertebral level does the spinal cord extend? LI to L2 6.What is the name of the extension of the dura mater that attaches at the level of S2? External filum terminale 7.How many pairs of spinal nerves exit from the spinal cord? 31 pairs

8.What is the name of the region where the manubrium and the body of the sternum articulate? Sternal angle of Louis 9.What muscle originates from the third to the fifth ribs and inserts into the c oracoid process? Pectoralis minor .10.Damage to what nerve will give you "winged scapula'."? Long thoracic ne rve To avoid confusing long thoracic nerve and lateral thoracic artery: Long has an "n" for nerve; lateral 3 & has an "a" for artery. 11.The ventral rami of what regions of the spinal cord make up the brachial plex us? C5-TI 12.What bone houses the ulnar groove? nd the trochlea) Humerus (between the medial epicondyle a

13.What muscle initiates Abduction of the arm? Supraspinakis 14.What muscle acts in all ranges of motion of the arm? Deltoid ` 15.What nerve is damaged if a patient presents with "wrist drop"? erve 16.What forms the anatomic snuff box? cis longus, extensor pollicis brevis

Radial n

Extensor pollicis longus, abductor polli

17.What vein, in the antecubital fossa, forms the communica- tion between the ba silic vein and the cephalic vein? Median cubital vein (most common site fo r venipuncture) 18.What two muscles are inner- vated by the axillary nerve? s minor Deltoid and tere

19.What nerve is compromised in carpal tunnel syndrome? Median nerve 20.In what compartment of the thigh is the profundus femoris artery found? Anterior compartment (it's the blood supply to the posterior compartment) 21.Foot drop is caused by a compromise in what nerve? Common peroneal nerve Obturato

22.What nerve is damaged if the patient cannot ADduct the thigh? r nerve (nerve to the medial compartment of the thigh) 23.What is the longest muscle of the body? Sartorius

24.What two nerves innervate the pectineus muscle? rves

Femoral and obturator ne

25.What superficial vein empties into the popliteal fossa? vein 26.What is the artery of the anterior compartment of the leg? artery 27.What nerve supplies the lateral compartment of the leg? neal nerve

Short saphenous Anterior tibia] Superficial pero

28.What sensory nerve are you testing when you touch the first web space of the toes? Deep peroneal nerve 29.The peroneal artery is a branch of what artery? Posterior tibial

30.Inflammation of the pre- patellar bursa is often referred to as what? Housemaid's knee 31.What is the prominent "bump" on the lateral aspect of the knee? the fibula 32.How many ribs articulate with the sternum? ate with the costal cartilage of rib 7.) Head of

Seven (Ribs 8, 9, and 10 articul

33.What is the part of the lung that extends above the level of the first rib? The cupula 34.What type of pleura is adherent to the surface of the organ? Visceral pleura 35.How many lobes does the right lung have? 36.How are they separated? Three

By the oblique and the transverse fissures

37.Into what chamber of the heart do the pulmonary veins empty? Left atrium (Rem ember-the pulmonary veins carry oxygenated blood.) 38.What is the only valve in the heart that has two cusps? ) valve 39.What vein travels with the right coronary artery? Mitral (bicuspid

Small cardiac vein

40.At what vertebral level does the trachea bifurcate? T4-T5 (It is known as th e carina.) 41.What attaches the cusps of the valves to the papillary muscles in the heart? Chordae tendineae 42.Around what thoracic structure does the right recurrent laryngeal nerve loop before ascending into the larynx? Right subclavian artery 43.At what vertebral level does the esophagus originate? C6

44.At what level does the abdominal aorta bifurcate into the common iliac arteri es? L4-L5 45.The obturator artery is a branch of what major artery? rtery 46.What is the first branch off the abdominal artery? Internal iliac a

Inferior phrenic artery

47.Into what vessel does the right gonadal vein drain? The inferior vena cava 48.Into what vessel does the left gonadal vein drain? The left renal vein C4

49.At what vertebral level does the common carotid artery bifurcate? 50.At what vertebral level is the hyoid bone found? 51.The ophthalmic artery is a branch of what vessel? C3

Internal carotid artery

52.What forms the portal vein? The union of the superior mesenteric and the spl enic veins 53.Where does the inferior mesenteric vein drain? The splenic vein

54.What vein is formed by the union of the right and left brachiocephalic veins? Superior vena cava 55.What is the only muscle in the larynx that is not inner- vated by the recurre nt laryn- geal nerve? Cricothyroid (It's innervated by the external laryngeal nerve.) 56.The folds of the mucosa of the stomach are known as what? 57.What is the artery of the embryonic foregut? Celiac artery 58.What comprises the portal triad? Portal vein 1. Common bile duct 2. Hepatic artery 3. Rugae

59.What structures differentiate the anatomic right and left lobes of the liver? Ligamentum teres and ligamentum venosum 60.What structure "runs" along the transverse processes of the lumbar vertebrae? Ureters 61.To enter into the lesser peri-toneal sac, you must traverse through what fora men? Foramen of Winslow 62.What is another name for the rectouterine pouch? Pouch of Douglas

63.What bones comprise the acetabulum? Pubis, ilium, and ischium 64.What two ligaments of the uterus are remnants of the gubernaculum? d ovarian ligaments Bound an

65.What muscles comprise the deep perineal space (the urogenital diaphragm)? Deep transverse perineal and sphincter urethrae 66.What three ligaments com- prise the broad ligament of the uterus? alpinx 2. Mesovarium 3. Mesometrium 67.What structure traverses the diaphragm at the level of T8? 68.What are the components of the pudendal canal? nal pudendal artery and vein IVC 1. Mesos

Pudendal nerve and inter

69.What range of movements can be performed at the metacarpal/phalangealjoint? Flexion/extension, ABduction, and ADduction 70.A fracture of the surgical neck of the humerus will most likely damage what n

erve?

Axillary nerve

71.What compartment of the lower extremity allows flexion of the hip and extensi on of the knee? Anterior compartment of the thigh 72.What nerve roots comprise the lumbosacral plexus? 73.What is the function of gray rami communicans? sympathetic axons. L4 to S4 They are postganglionic

74.What compartment of the lower extremities allows ADduction of the thigh and f lexion of the hip? Medial compartment of the thigh 75.What are the only splanchnics in the body that carry preganglionic parasympat hetic fibers? Pelvic splanchnics (P begins preganglionic, parasympathetic, and pelvic.) 76.What postganglionic parasympathetic ganglion is associated with CN III? Ciliary ganglion 77.What is the name of the ganglion that houses the cell bodies for the postgang lionic sympathetic fibers to the head and neck? Superior cervical ganglion 78.What two muscles do you test to see if CN XI is intact? ernocleidoinastoid Trapezius and st Motor as

79.What component of the corneal reflex is lost in a CN VII deficit? pect

80.Toward what side would the uvula point if the right CN X were damaged? The left (points to the unaffected side) 81.What is the name of the urinary bladder where the ureters enter and the ureth ra exits? Urinary trigone 82.What is the only organ in the body supplied by preganglionic sympathetic fibe rs? Adrenal rnedulla 83.The pudendal canal is formed by splitting the fascia of what muscle? Obturato r internus 84.What is the name of the duct formed by the union of the vas deferens and the duct of the seminal vesicle? Ejaculatory duct 85.What are the fingerlike projections at the end of the fallopian (uterine) tub es? Fimbriae 86.Where is the seminal vesicle located? rinary bladder On the posterior aspect of the u

87.What vessel can be found atop the scalene anterior? Subclavian vein 88.What muscle divides the anterior from the posterior triangles of the neck? Sternocleidomastoid 89.Where does the parotid (Stenson's) duct enter the oral cavity? the second upper molar tooth Opposite

90.What is the function of the arachnoid granulations? Resorb CSF into the bloo d

91.What muscle is the most superiorly situated muscle in the orbit? palpebrae superioris 92.What is the triad of Horner's syndrome?

Levator

Miosis, ptosis, and anhydrosis

93.What bone of the middle ear articulates with the tympanic membrane? Malleus 94.What chamber of the eye is located between the iris and the lens? r chamber 95.What bone houses the ear? Temporal bone Posterio

96.What is the only muscle of the tongue not innervated by the hypoglossal nerve ? Palatoglossus 97.Where does the nasolacrimal duct terminate? Inferior meatus of the nasal cav ity 98.What gland is found in the muscular triangle of the neck? Thyroid gland

99.What two regions of the vertebral column are con- sidered primary curvatures? Thoracic and sacral 100.What are the only muscles in the body innervated by dorsal rami? Intrinsi c (deep) muscles of the back (All other muscles are innervated by ventral rami.) 101.What is the portion of the second cervical vertebra that projects superiorly to act as the body for C1? Odontoid (dens) process 102.What is the actual space that contains CSF? Subarachnoid space 103.What is the protective covering that is adherent to the spinal cord and CNS tissue? Pia mater 104.What is the name of the spinal cord that passes within the subarachnoid spac e that forms the spinal nerves that exit the lumbar and sacral foramina? Cauda equina 105.What are the names ligaments that would pierced, in order, by a lumbar punct ure? 1. Supraspinous ligament 2. Interspinous ligament 3. Ligamentum flavum 106.What is the inferiormost segment of the sternum? Xiphoid process True; it

107.True or false-the pectoralis major medially rotates the arm? also ADducts and flexes the arm. 108.What are the borders of the axillary artery? rst rib to the inferior border of the teres major

Lateral border of the fi

109.What vessels arise from the three segments of the axillary artery? 1. Super ior thoracic artery 2. Lateral thoracic artery and thoracoacromial trunk 3. Subs capular artery, and the anterior and posterior humeral circumflex One artery fro m the first segment, two arteries from the second segment, and three arteries fr om the 110.What muscle is the main lateral rotator of the arm? Infraspinatus muscle 111.What innervates the flexor compartment of the arm? Musculocutaneus nerve

112.What nerve is most commonly affected when there is a fracture of the midshaf t of the humerus? Radial nerve C deer "(Int4.1 a . 113.What vein courses along the medial aspect of the forearm? Basilic vein

114.What is the blood vessel in the upper extremity most commonly palpated while taking a pulse? Radial artery 115.What is the nerve supply to the forearm? Median nerve (except for the fle xor carpi ulnaris and flexor digiti profundus muscles of the pinkie and ring fin ger, which are supplied by the ulnar) 116.What are the "LOAF" muscles of the hand? LOAF stands for the muscles of t he hand innervated by the median nerve: Lumbricales, Opponens pollicis, Abductor pollicis brevis, and Flexor pollicis brevis; All other intrinsic muscles in the hand are innervated by the ulnar nerve 117.What muscles in the hand ADduct the fingers? The palmer interosseus a dducts, whereas the dorsal interosseus abducts (PAD and DAB) 118.In order to pronate the hand, what bones need to cross? ver the ulna Radius crosses o

119.At what point does the femoral artery become the popliteal artery? When it traverses the adductor hiatus 120.Loss of ABduction of the lower limbs results in Trendelenburg gait; what ner ve is compromised to cause this? Superior gluteal nerve 121.What two arteries join together to form the super- ficial and deep palmar ar ches of the hand? Uhiar and radial arteries (ulnar is the main supplier) 122.What muscle "fills" the greater sciatic foramen? Piriformis

123.What nerve is affected when a patient has difficulty rising from a sitting p osition? Inferior gluteal nerve (nerve to the gluteus maximus) 124.Why are IM injections in the gluteal mass given in the upper outer quadrant? To avoid damage to the sciatic nerve 125.What two nerves innervate the adductor magnus? ves 126.What two nerves innervate the biceps femoris? al nerves Obturator and tibial ner Common peroneal and tibi

127.Going from lateral to medial, what structures pass deep to the inguinal liga ment? NAVEL-Nerve, Arterv, Vein, Empty space, and Lacunar ligament or Lymphati cs 128.What artery turns into the dorsalis pedis when it crosses the extensor retin aculum? Anterior tibial artery 129.What is the nerve for the anterior compartment of the leg? Deep peroneal ne rve 130.What is the artery for the posterior compartment of the leg? r tibial arterv Posterio

131.Where is the "magical plane" that divides the superior from the inferior med

ia- stinum?

A horizontal line from T4-T5 to the sternal angle of Louis Hemiazygous vein

132.What vein drains the lower third of the thoracic wall?

133.If you were to do a pleural tap, what region of the intercostal space would your needle enter? The superior border of the rib Why? Because the neurovascular bundle is located on the inner surface of the inferior border of the rib 134.What muscles of the foot are supplied by the medial plantar nerve (Hint: Thi nk about the median nerve distribution in the hand.)? LAFF-Lumbricalis (1st), Abductor hallucis, Flexor 3X hallucis brevis, Flexor digitorum brevis. All other intrinsic muscles in the foot are supplied by the lateral plantar nerve. 135.What remnant of the middle lobe of the lung is found on the left side? The lingula 136.The ventral rami of what cervical vertebrae innervate diaphragm? and C5 keep the diaphragm alive! C3, C4,

137.At the level of rib 6, the internal thoracic artery divides into what two ar teries? Musculophrenic and superior epigastric arteries 138.What portion of the peri- cardium is adherent to the tunica adventitia of th e great vessels? Fibrous pericardium 139.The left anterior descending artery of the heart travels with what vein? Great cardiac vein 140.What is the largest muscle in the body? Gluteus maximus Posterio

141.The middle cardiac vein of the heart travels with what artery? r intraventricular artery

142.What is the ratio of the myocardial thickness of the atria: right ventricle: left ventricle? 1:03:09 143.What chamber of the heart comprises the: Sternal surface? Diaphragmatic surface? Right ventricle and left ventricle Left margin? Left ventricle and left atrium Right ventricle

Right margin? Right atria Base? Left atria 144.What structure does the left recurrent laryngeal nerve loop around before it ascends into the larynx? The arch of the aorta 145.At what point does the axillary artery become the brachial artery? When it crosses the teres major 146.What is the anatomic posi- tioning of the right and left gastric nerve plexu s of the esophagus as they pass through the diaphragm? LARP147.Left goes Anterior and Right goes Posterior (because of the rotation of the gut -Remember your embryology!) 148.What muscles comprise the rotator cuff? s, Teres minor, & Supraspinatus SITS-Subscapularis, Infraspinatu

149.What are the five branches off the median cord of the brachial plexus? Four Ms and a U- 1. Median 2. Medial antebrachial 3. Medial pectoral 4. Medial b rachial cutaneus 3X 5. Ulnar 150.What are the five branches off the posterior cord of the brachial plexus? STARS- 1. Upper Subscapularis 2. Thoracodorsal 3. Axillary ; & 4. Radial 5. Lowe r Subscapularis 151.What are the three branches off the lateral cord of the brachial plexus? 1. Lateral pectoral 2. Lateral head of the median 3. Musculocutaneus 152.What are the four branches off the brachial plexus that arise prior to the f irst rib? 1. Dorsal scapular 2. Suprascapular 3. Long thoracic 4. Nerve to subclavius 153.What nerve innervates the extensor compartment of the arm? Radial nerve (It also innervates the extensor compartment of the forearm.) 154.What muscles insert in/on the intertubercular groove of the humerus? Lady between two Majors- latissimus dorsi, pectoralis major, and teres major 155.What artery is found in the lateral compartment of the leg? None. The perone al artery is in the posterior compartment of the leg. 156.What muscle laterally rotates the femur to "unlock" the knee? s 157.What bursa is inflamed in "clergyman's knee"? 158.Where does the great saphenous vein terminate? Popliteu

Infrapatellar bursa In the femoral vein

159.What comprises the "unhappy triad" of the knee? 1. Medial collateral lig ament 2. Medial meniscus 3. Anterior cruciate ligament (ACL) The severity of inj ury to these ligaments is ranked from bad to worst in relation to how many of th em are damaged, and they are usually damaged in the order listed. 160.What are the two branches off the external iliac artery before it becomes th e femoral artery? Circumflex iliac and inferior epigastric arteries 161.From which three sources does the adrenal gland get its blood supply? 1. Superior suprarenal artery (off inferior phrenic artery) 2. Directly off the abdominal aorta as the middle suprarenal artery 3. Inferior suprarenal artery of f the renal artery 162.What are the three branches off the celiac artery? 1. Common hepatic artery 2. Splenic artery 3. Left gastric artery 163.What are the three main branches off the inferior mesenteric artery? 1. Left colic artery 2. Superior rectal artery 3. Sigmoid artery 164.The inferior thyroid artery is a branch of what vessel? unk Thyrocervical tr Carotid

165.What is the area of the carotid artery that is an 02 receptor? body (The carotid sinus is a pressure receptor.) 166.What arteries join together forming the basilar artery? ertebral arteries

Left and right v

167.What is the major difference between the veins in the face and the veins in

the rest of the body? in the face

No valves and no smooth muscle in the walls of the veins

168.At what point does the sig- moid sinus become the internal jugular vein? When it crosses the jugular foramina 169.What connects the lateral ventricles to the third ventricle? of Monro Foramen

170.What connects the third and the fourth ventricles together? Cerebral aqueduc t 171.How does cerebrospinal fluid leave the fourth ventricle? Through the fora mina of Magendie (medial) and Luschka (lateral) M in Magendie = medial; L in Lus chka = lateral. 172.What is the lymphatic drainage of the gonads? hatic drainage follows blood supply.) 173.What is the lymphatic drainage of the pelvic organs? odes Lumbar trunk nodes (Lymp Internal iliac n

174.What are the five clinical signs of portal hypertension? 1. Caput medusae 2. Hemorrhoids 3. Retroperitoneal varices 4. Splenomegaly 5. Esophageal varices 175.What is the region of the body where all tonsillar tissue can be found? Waldeyer's ring 176.What are the three functions of the nasal cavity? filters inspired air. It warms, moistens, and

177.What region of the pharynx does the eustachian tube enter? Nasopharynx 178.In which segment of the duodenum is the ampulla of Vater located? egment Second s

179.The duodenal/jejunal flexure is suspended from the posterior abdominal wall by what? Ligament of Treitz 180.What are the three anatomic characteristics that differen- tiate the large b owel from the small bowel and the rectum? 1. Tinea coli 2. Haustra 3. Epip loic appendages 181.What is the artery of the embryonic midgut? Superior mesenteric artery 182.What two ligaments together comprise the lesser omentum? d hepatoduodenal Gastrohepatic an

183.The quadrate and the caudate lobes are part of what side of the anatomic liv er? The right 184.What is the artery of the embryonic hindgut? ry Inferior mesenteric arte Common b

185.The hepatic duct and the cystic duct come together to form what? ile duct 186.What muscle forms the "bed" for the kidney? Quadratus lumborum 187.What ligament of the uterus houses the ovarian vessels?

Suspensory ligam

ent of the ovary 188.What three muscles comprise the pes anserinus? 3. Semitendinous 1. Sartorius 2. Gracilis

189.What are the 10 retro- peritoneal organs? 1. Duodenum 2. Ascending Colon 3 . Ureters 4. Pancreas 5. Suprarenals 6. Descending colon 7. Aorta 8. Kidneys 9. Rectum 10. Inferior vena cava 190.What is the only cranial nerve with the ability to regenerate? CN I

191.What duct transmits secretions from the sub- mandibular gland to the oral ca vity? Wharton's duct 192.What are the boundaries of the posterior triangle of the neck? n ocleidomastoid, the trapezius, and the clavicle 193.What are the contents of the adductor canal? and saphenous nerve The ster

Femoral artery and vein Submandi

194.What are the contents of the submandibular triangle of the neck? bular gland, facial artery and vein, nerve to the mylohyoid 195.What carpal bones articulate with the radius?

Scaphoid and lunate

196.What are the borders of the anterior triangle of the neck? Sternocleidomast oid, mandible, and midline of the neck 197.What are the six cranial nerves that innervate structures in the orbit? 1. CN II-vision 2. CN Vl-sensory (ophthalmic division) 3. CN VII-lacrimal gland LR6 (S04)3 (to remember nerves below): 4. CN VI-lateral rectus 5. CN IV-superior oblique 6. CN III-all other muscles of the eye 198.What muscle keeps the stapes taut against the oval window? Stapedius muscle 199.What area of the posterior aspect of the eye has no photo receptors? The optic disk is the blind spot. 200.What muscles are found in the superficial perineal pouch? sverse perineal, ischiocavernous, and bulbocavernous muscles 201.What gland is found in the deep perineal pouch in men? and; no gland is found in this pouch in women. Superficial tran Bulbourethral gl

202.What pouch of the peri- neum houses the superficial fascia and the inferior fascia? The deep perineal pouch (same as the urogenital diaphragm) 203.What gland is found in the superficial perineal pouch in men and women? None in men, the greater vestibular gland in women 204.What are the five structures that traverse the spermatic cord? 1. Pampi niform plexus of veins 2. Vas deferens 3. Testicular artery 4. Nerves 5. Lymphat ics 205.What are the borders of Hesselbach's triangle? Rectus abdominis mediall y, Inferior epigastric vessels laterally, Inguinal ligament as the base 206.Which type of hernia goes through both the deep inguinal and superficial ing uinal ring? Indirect; a direct hernia goes directly through Hesselbach's tri angle

207.What structure(s) traverse the diaphragm at the level of T10? s and gastric plexus of nerves

Esophagu

208.What structure(s) traverse the diaphragm at the level of T12? Aorta, a zygous vein, and thoracic duct Remember: One at Ts, two at Tlo,three at T,Z. 209.What compartment of the lower extremity allows flexion of the toe, inversion of the foot, and plantar flexion of the foot? Posterior compartment of the leg 210.How are the arm and the forearm positioned in "waiter's tip hand"? The arm is medially rotated, and the forearm is extended and pronated. 211.What is the function of white rami communicans? They are preganglionic s ympathetic axons. They are white because they are myelinated. 212.What compartment of the lower extremity allows extension of the hip and flex ion of the knee? Posterior compartment of the thigh 213.What compartment of the lower extremity allows dorsiflexion, extension of th e toes, and inversion of the foot? Anterior compartment of the leg 214.What cervical nerves comprise the cervical plexus? C1 to C4 215.What compartment of the lower extremity allows eversion and plantar flexion of the foot? Lateral compartment of the leg 216.If the left hypoglossal nerve was damaged, which way would the tongue point? To the left (The uvula points to affected side.) 217.In women, what is the name of the pouch between the bladder and the uterus c alled? Uterovesical pouch 218.What component of the pelvic diaphragm forms the rectal sling (muscle of con tinence)? Puborectalis 219.What is the name of the comma-shaped structure that is attached to the poste rior aspect of the testes? Epididymis 220.What is the structure of the uterus that projects above the opening of the u terine tubes? Fundus 221.What is the region of the fallopian tube where fertilization most commonly o ccurs? Ampulla 222.How can you access the lowermost point in the peritoneal cavity in women? Via the posterior fomix of the vagina 223.Which type of hemorrhoids are painful? emorrhoids lack pain fibers. External hemorrhoids; internal h

224.What are the five terminal branches of the facial nerve? 1. Temporal 2. Z ygomatic 3. Buccal 4. Mandibular 5. Cervical (Two Zebras Bit My Clavicle.) 225.What two vessels come together to form the external jugular vein? rior auricular vein 2. Posterior division of the retromandibular vein 226.What is the position of the eyeball if CN VI is lost? Adducted 1. Infer 1. Poste

227.The thyroid gland receives blood from what two different sources?

ior thyroid off the thyrocervical trunk 2. Superior thyroid artery off the exter nal carotid artery, and sometimes off the arch of the aorta as the thyroid ima a rtery 228.What postganglionic para-sympathetic ganglia is associated with: CN VII? COPS Submandibular ganglion 229.What postganglionic para-sympathetic ganglia is associated with: CN IX? Pterygopalatine and otic ganglion 230.What postganglionic para-sympathetic ganglia is associated with: CN X? Terminal ganglion 231.What type of fibers are carried in the thoracic and lumbar splanchnics? Preganglionic sympathetic fibers 232.How are preganglionic parasympathetic fibers carried to the embryonic hindgu t? Via pelvic splanchnics 233.What is the only portion of CN V that carries motor fibers? Mandibular divis ion (V3) 234.What portion of CN V is affected if the corneal reflex is lost? ic division (VI) Ophthalm

235.What cranial nerve is affected if you have a laterally deviated eye that is dilated with a ptosed eyelid? CN III 236.Which cranial nerves are found in the midline of the brain stem? CN I, 11 , 111, VI, and XII ARN Add 1 + 1 = 2, 1 + 2 = VXF 3,1+2+3=6,1+2+ 3+6=12 237.What are the four muscles of mastication? ial pterygoid 4. Lateral pterygoid 1. Masseter 2. Temporalis 3. Med

1.What happens to prevalence as: Incidence increases? 2.What happens to prevalence as: Duration increases? 3.What is the formula for IQ? ge)

Increases Increases

MA/CA x 100 (MA = mental age, CA = chronologic a Crude ra

4.In statistics, what is the measured rate for: A whole population? te

5.What stage of sleep is associated with high pulse, blood pressure, and respira tion rates, and is characterized by increased brain oxygen use, penile erection in males, and total paralysis of the skeletal muscles? REM sleep (Remember as: Awake brain & paralyzed body.) 6.What stage of sleep is associated with slow pulse and respiratory rates, a dec rease in blood pressure, and involuntary skeletal muscle contractions? Non-rapi d eye movement (NREM) sleep (Remember as: Idle brain in an awake body.) 7.What type of study is prospective-that is, it "looks to see who gets sick" by defining a population at risk of being exposed to a disease? Cohort study (al so called prospective, follow-up, longitudinal, or incidence study) 8.What hormone is inhibited by sleep? TSH

9.With what stage of sleep are nightmares associated?

REM=`remember them" ; Stage 4-they are

10.With what stage of sleep are night terrors associated? not remembered

11.What is the triad of normal pressure hydrocephalus? Dementia, gait apraxia, and urinary incontinence 12.What category of symptoms of schizophrenia associated with muscarinic recepto rs include affective flattening, social withdrawal, apathy, anhedonia, poverty o f thought and of content of speech, and lack of interest? Negative symptom s (type II) 13.In biostatistics, what type of error is due to chance? Random error

14.Dementia is associated with a decrease in what neurotransmitter in the amygda la, hippocampus, and temporal neocortex? Acetylcholine (ACh) 15.What symptoms of schizophrenia associated with dopamine receptors include del usions, hallucinations, and agitation? Positive symptoms (type I) 16.In biostatistics, what type of error has unanticipated factors that obscure t he relationship and cause a bias? Confounding error 17.What is the most frequently occurring value in a set? Mode

18.What is the difference between the highest and the lowest scores in a set? Range 19/What type of study has diffusional effects if you separate the groups and tes t the entire population? Community trial 20.What type of study is under the greatest possible degree of control of the in vestigator? Experimental study 21.In what type of skew is the tail to the right and the mean greater than the m edian? Positive skew 22.In what type of error is the null hypothesis rejected when it is true? Type I error (alpha error) 23.If the P-value is less than or equal to .05, what do you do to the null hypot hesis? Reject it What drug is used to prevent alcohol consumption by blocking aldehyde dehydrogen ase? Disulfiram Which drug is used to treat opiate withdrawal, attention deficit hyperactive dis order (ADHD), and sometimes Tourette's syndrome? Clonidine Which drug is used to treat the respiratory depression associated with an overdo se of opioids? Naloxone or naltrexone Which opioid agonist, more addictive than heroin, is used in the treatment of he roin dependence? Methadone What type of reinforcement strengthens each response and involves fast learning and fast extinction? Continuous reinforcement According to operant conditioning theory, what type of reinforcement is occurrin g in avoidance behaviors such as phobias and compulsive rituals? Negative reinforcement What are the pharmacologic effects seen sexually with: alpha-Blockers? Impaired

ejaculation What are the pharmacologic effects d orgasm What are the pharmacologic effects e What are the pharmacologic effects What are the pharmacologic effects Increased erection and libido What are the pharmacologic effects dysfunction In a classic gaussian deviations (SDs)? In a classic gaussian nd 1 SD? 34% In a classic gaussian SDs? 13.50% In a classic gaussian 3 SDs? 2.40% In a classic gaussian 0.15%

seen sexually with: Serotonin? seen sexually with: beta-Blockers?

Inhibite Impotenc

seen sexually with: Trazodone? Priapism seen sexually with: Dopamine agonists? seen sexually with: Neuroleptics? Erectile

curve, what percentage of the curve is between: 3 standard 99.70% curve, what percentage of the curve is between: The mean a curve, what percentage of the curve is between: 1 SD and 2 curve, what percentage of the curve is between: 2 SDs and curve, what percentage of the curve is between: 3 SDs?

1.What is the rate-limiting step of: Glycolysis? PFK-1 What is the rate-limiting step of: Gluconeogenesis? Pyruvate carboxylase What is the rate-limiting step of: TCA (Krebs) cycle? Isocitrate dehydrogenase What is the rate-limiting step of: Glycogenesis (glycogen synthesis)? Glycogen synthase What is the rate-limiting step of: Glycogenolysis? Glycogen phosphorylase What is the rate-limiting step of: Hexose monophosphate (HMP) shunt? Glucose6-phosphate dehydrogenase (G-6-PD) What is the rate-limiting step of: Fatty acid synthesis? Acetyl CoA carbo xylase What is the rate-limiting step of: -Oxidation? Carnithine acyltransferase I What is the rate-limiting step of: Ketogenolysis? HMG CoA synthase What is the rate-limiting step of: Cholesterol synthesis? HMG CoA reductas e What is the rate-limiting step of: Urea cycle? Carbamoyl phosphate synthetase I What is the rate-limiting step of: Heme synthesis? Delta-Aminolevulinic aci d (ALA) synthase What is the rate-limiting step of: Pyrimidine synthesis? Aspartate transc arbomylase What is the rate-limiting step of: Purine synthesis? Phosphoribosyl pyrophosp hate (PRPP) glutamyl amido transferase What is the only fatty acid that is gluconeogenic? Propionic acid Aldose reductase converts galactose to what? Galactitol 2.How many ATPs are generated from glycolysis? 8 3.In the mitochondria, what complex is needed in order for pyruvate carboxylase to catalyze the reaction from pyruvate to OAA? Biotin, ATP, and CO2 4.What is the enzyme for the oxidative reaction in glycolysis? Glyceraldehyde d ehydrogenase 5.What three substrates control the enzyme phosphoenolpyruvate carboxykinase (PE PCK) for the conversion of OAA to pyruvate in the cytoplasm? 1. Cortisol (sti mulates PEPCK) 2. Glucagon 3. Guanine triphosphate (GTP)

6.What is released from the reaction of phosphoenolpyruvate carboxykinase (PEPCK ) for the conversion of OAA to pyruvate? C02 7.What enzyme deficiency causes cataracts, galactosemia, and galactosuria? Galactokinase deficiency 8.The addition of D-2,3-bisphosphoglycerate (D-2,3-BPG) to HbA does what to the 02 saturation curve? Shifts it to the right 9.In what benign condition do you see excretion of large amounts of fructose aft er ingestion? Essential fructosuria (fructokinase deficiency) 10.What is the glycolysis enzyme found only in the liver? Glucokinase

11.How many ATPs are generated per acetyl CoA? 12 (Not 15-that would be the ans wer if you included the pyruvate to acetyl CoA step.) 12.What enzyme is associated with the substrate-level phosphorylation in the TCA cycle? Succinate thiokinase 13.The availability of OAA and acetyl CoA regulates what pathway? e TCA cycl

14.What complex of the electron transport chain (ETC) is inhibited by malonate? Complex II 15.What drug blocks the FO portion of the adenosine triphosphatase (ATPase) syst em of the electron transport chain (ETC)? Oligomycin 16.In what two places is glycogen made and stored? 1. Liver 2. Muscle: Live r stores are for blood glucose; muscle stores are for energy reserves. 17.What drug blocks the ETC by attaching itself to K+ for passage through the me mbrane, negating the charge gradient? Valinomycin 18.At what step of the TCA cycle is FADH2 generated? (inhibited by malonate) 19.What inhibits complex III of the ETC? Succinate dehydrogenase

Antimycin A Atractyloside

20.What inhibits the ATP/ADP translocase of the ETC?

21.Thiamine pyrophosphate (TPP) is associated with what three enzymes? 1. a-ket oglutarate dehydrogenase 2. Pynivate dehydrogenase 3. Transketolase 22.What hormone stimulates glycogen synthesis? Insulin 23.Deficiency in what enzyme leads to insoluble glycogen formation? ansferase A-1,6 tr

24.The reduced nicotinamide adenine dinucleotide phosphate (NADPH) generated fro m the G-6-PD reaction is used exclusively for what? Fatty acid synthesis 25.What enzyme requires selenium (Se) to function? 26.What are the two essential fatty acids? id Glutathione peroxidase

1. Linoleic acid 2. Linolenic ac

27.What intermediate of the HMP pathway is used to generate nucleotides? Ribose-5-phosphate

28.A deficiency in what enzyme causes a decrease in oxidoreductase activity in n eutrophils? G-6-PD 29.What are the nonoxidative enzymes of the HMP shunt? Transketolase and transa ldolase 30.Are the reactions they catalyze reversible or irreversible? Reversible 31.A patient who presents with cardiomegaly and hepatomegaly has what glycogen s torage disease? Pompe's disease (lysosomal glucosidase deficiency) 32.Very low density lipoprotein (VLDL) remnants are known as what? iate-density lipoproteins (IDLs) Intermed

33.What carries triacylglycerols (TAGS) and cholesterol from the diet? Chylomic rons 34.What protein is required for the uptake of low-density lipoproteins (LDLs) in the peripheral tissue? Apoprotein B-100 35.What 3 apoproteins are on the surface of chylomicrons? C-II, and E 36.What protein carries free fatty acids to the liver? Albumin 37.What hormone is activated in adipose tissue when blood glucose levels decreas e? Hormone-sensitive lipase 38.In the P-oxidation pathway, what enzyme generates the FADH2? Acyl CoA dehydro genase 39.How many ATPs are generated per acetyl CoA in [3-oxidation? 5 40.How many ATPs are generated per acetyl CoA from [3-oxidation if it is run thr ough the TCA cycle? 12 41.What is the only organ in the body that can produce ketone bodies? r (in the mitochondria) The live Apoprotein B-48,

42.What two tissues prefer ketone bodies over glucose? 1. Heart muscle 2. Renal cortex 43.What enzyme is absent in the liver so that ketogenolysis cannot occur? Thiophorase 44.What pathway utilizes HMG-CoA synthetase in the cytoplasm? ynthesis Cholesterol bios

45.What two vitamins are inactivated when they come in contact with acetaldehyde ? 1. Thiamine 2. Folate 46.What is the precursor of all sphingolipids? Ceramide 47.What two sugars can be used to produce cerebrosides? 1. Glucose 2. Galactose 48.Where does the energy for the urea cycle come from? Fat metabolism 49.What are the two major carriers of nitrogen from tissues? st tissues) 2. Alanine (muscle) 1. Glutamine (mo

50.What are the ketogenic amino acids? Leucine and lysine 51.What is the storage form of folate? N-methyl folate 52.What disease is produced by a deficiency in the enzyme tyrosinase? (Tyrosine is converted to melanin by the enzyme tyrosinase.) 53.What two enzymes are blocked by lead? tase Albinism

1. ALA dehydratase 2. Ferrochela Reticular endothelial sy Indirect or unco Uridine monophos PH of 2 and 9

54.Where in the body is heme converted to bilirubin? stem (RES) 55.What type of bilirubin is found in neonatal jaundice? njugated

56.What is the primary end product of pyrimidine synthesis? phate (UMP) 57.All amino acids have titration plateaus at what pH values? 58.What amino acid is a good buffer at a pH of 7?

Histidine Increase enzyme

59.What is the only way to increase maximum velocity (VmaY)? concentrations

60.What happens to affinity if you increase K ? Affinity decreases (they are inv ersely proportional) 61.What two amino acids disrupt an a-helix? 62.What amino acid is a phenol? Tyrosine 63.What enzyme requires molybdenum (Mo) as a cofactor? Xanthine oxidase 64.What determines the rate of a reaction? The energy of activation (Ea) Km 1. Glycine 2. Proline

65.What substrate concentration is required to produce 1/2 Vmax? 66.What enzyme is blocked by allopurinol? itor")

Xanthine oxidase ("suicide inhib

67.What enzyme is stimulated by PTH to produce 1,25 vitamin D3? 1-a-Hydroxylase 68.What three organs are used to produce vitamin D? ey 1. Skin 2. Liver 3. Kidn

69.What vitamin is an important component of rhodopsin? Vitamin A 70.What G protein is stimulated by activated rhodopsin? Gt (transducin), which d ecreases cyclic guanosine monophosphate (cGMP) and closes the Na+ channels, caus ing nerve transmission 71.What are the vitamin K-dependent clotting factors? 2, 7, 9, and 10 Vitamin E

72.What vitamin is connected to selenium (Se) metabolism? 73.What is the activated form of vitamin E? Alpha-tocopherol

74.What elements make up a nucleoside? A base and a sugar

75.What is the most common methylated base?

Cytosine S phase

76.DNA is replicated at what phase of the cell cycle? 77.At which end of DNA are new bases added? 3' end

75.What keeps single-strand DNA (ssDNA) from re-annealing during DNA replication ? Single-strand (ss) binding protein 76.What enzyme is responsible for producing a single-strand (ss) cut in the DNA to relieve the coil tension? Topoisomerase I (relaxase) 77.What two amino acids are found in high concentrations in the nucleosome? 1. Arginine 2. Lysine 78.What three bases are pyrimidines? hymidine 1. Cytosine 2. Uracil (only in RNA) 3. T

79.What enzyme creates a short sequence of RNA to start DNA replication? Primase 80.What type of enzyme is reverse transcriptase? rise 81.What is the direction of transcription? RNA-dependent DNA polyme 5' to 3' direction

82.Where is the TATA box in located eukaryotes? 25 bases downstream (-25)(promot er) 83.What causes transcription to stop in eukaryotes? DNA The poly(A) site on the

84.What protein binds to the promoter region in eukaryotes to initiate transcrip tion? TF II D (transcription factor) 85.What part of the 30S ribosome binds to the Shine-Dalgarno sequence? 16S subu nit 86.What is the start codon for translation? AUG Aminoacy

87.What is the enzyme that activates the amino acids for the tRNA? l-tRNA synthetase 88.What is needed to direct enzymes to a lysosome? se residues 89.What cofactor is needed for lysyl oxidase? Cu2+

Phosphorylation of manno

90.What part of the 50S and 60S ribosomal subunit is needed for elongation? Peptidyl transferase 91.In the lac operon: At which site is the repressor gene encoded? I gene

92.To which site does the repressor protein bind in order to inhibit transcripti on? Operator 93.What amplification technique is used to generate a larger amount of DNA? Polymerase chain reaction (PCR)

94.What test is used to determine whether a gene is expressed? Northern blottin g 95.At what organ in the body is urea produced? Liver 96.What regulatory proteins work during fetal development to ensure that cells b ecome a specific cell type (If there is a defect here, there can be profound str uctural mutations.)? Homeobox genes 97.What is the mode of inheritance in which a trait is seen in every generation and is passed on only by females? Mitochondrial inheritance 98.What is the name for the process of going from mRNA to proteins? ion 99.What are the components of a nucleotide? Translat

A base, a sugar, and a phosphate

100.What enzymes hydrolyze 3'-5' phosphodiesterase bonds from the outside of the strand in? Exonucleases 101.What type of organisms have monocistronic mRNA? Eukaryotes Glycine

102.In collagen, every third amino acid is this amino acid.

103.What form of continuous DNA, used in cloning, has no introns or regulatory e lements? c-DNA, when it is made from mRNA 104.What proteins stimulate a cell to enter the S phase? Growth factors

105.At what pH is there no net charge on the structure? pI (isoelectric point) 106.What complex of the ETC contains Cue+? Complex 4 Malate/a

107.What two shuttles are needed to keep NAD+ in the reduced state? spartate and glycerol-3-phosphate shuttles

108.What vitamin is required for y-carboxylation of many Cat+-binding proteins? Vitamin K 109.From where is the energy for gluconeogenesis derived? ty acids -Oxidation of fat

110.What amino acid is broken down into N20, causing an increase in eGMP of smoo th muscle resulting in vasodilatation? Arginine 111.What hormone phosphorylates enzymes to decrease their activity? Glucagon

112.Lack of what enzyme can lead to Wernicke-Korsakoff syndrome through lack of activity in the HMP shunt? Thiamine pyrophospate (TPP) 113.What enzyme found in the liver catalyzes glycerol to glycerol-3-phosphate? Glycerol kinase 114.Which shuttle is used to bring fatty acyl CoA from the cytoplasm for ketogen esis? Carnitine acyl CoA transferase II 115.Which enzyme is deficient in phenylketonuria (PKU)? Phenylalanine hydroxylas e 116.After approximately how many days of a prolonged fast does death occur in hu

mans? 60 days What is the cause of death? eart and brain

The breakdown of the essential proteins of the h Cytoplas

117.All the carbons in a fatty acid are derived from what source? mic acetyl CoA that left the mitochondria as citrate 118.What enzyme is deficient in alcaptonuria? Homogentisic acid

119.In a diabetic patient, glucose is converted by aldose reductase to what? Sorbitol (resulting in cataracts) 120.What glycolytic intermediate can be used to synthesize triglycerides and pho spholipids? DHAP 121.What glycolytic enzyme has a high Vmax high Km and low affinity for glucose? Glucokinase 122.What is the main inhibitor of pyrnvate dehydrogenase? vate to acetyl CoA) Acetyl CoA (pyru 1. Pyruv

123.What are the two substrate-level phosphorylations in glycolysis? ate kinase 2. Phosphoglycerate kinase

124.What are the eight liver-specific enzymes? 1. Fructokinase 2. Glucokinase 3 . Glycerol kinase 4. PEPCK 5. Pyruvate carboxylase 6. Galactokinase 7. Fructose1,6-biphosphate 8. Glucose-6-phosphate 125.In what cycle does glucose go to the muscle, where it is converted to pyruva te and then into alanine before being taken back to the liver? Alanine cycle 126.In what cycle does glucose go to the muscle, where it is converted to lactat e, and then returned to the liver? Cori cycle 127.What four substances increase the rate of gluconeogenesis? 1. Glucagon 2. N ADH 3. Acetyl CoA 4. ATP 128.What enzyme is deficient in a patient who presents with:Q A.Liver damage and severe hypoglycemia? Aldolase B (hereditary fructose intolerance) QB..Jaundice, vomiting, lethargy, cat, galactosemia, and galactosuria? Galactos e 1-phosphate uridyltransferase 129.What three substances stimulate glycogenolysis? 2. Epinephrine 3. Glucagon 130.What are the two inhibitors of complex I of the ETC? mytai (barbiturates) 1. CaL+:calmodulin ratio 1. Rotenone 2. A

131.What are the five factors that constitute the pyruvate dehydrogenase complex ? 1. TPP 2. Lipoic acid 3. CoASH 4. FAD 5. NAD 132.What attaches to protons and allows them to enter into the mitochondria with out going through the ATP-generating system? 24-Dinitrophenol 133.What are the two decarboxylation steps of the TCA cycle? hydrogenase 2. a-Ketoglutarate dehydrogenase 134.What are the three inhibitors of complex IV of the ETC? 3. Azide 1. Isocitrate de 1. Cyanide 2. CO

135.What three steps of the TCA cycle generate NADH? 1. Malate dehydrogenase 2. Isocitrate dehydrogenase 3. a-Ketoglutarate dehydrogenase 136.What disease presents with weakness and cramps on exercise without an increa se in blood lactate levels? McArdle's disease (muscle glycogen phosphorylase deficiency) 137.NADPH generated from the HMP shunt is used for what? esis, nucleotide synthesis, and glutathione reductase 138.Is linolenic acid an omega-3 or -6 fatty acid? ga-6 Fatty acid synth Omega-3; linoleic is ome

139.Is the oxidative reaction of the HMP shunt reversible or irreversible? Irreversible (G-6-PD and 6-phosphogluconate dehydrogenase) 140.What disease presents with an enlarged liver and kidneys, dwarfism, hypoglyc emia, acidosis, and hyperlipidemia? Von Gierke's disease (glucose 6-phosphat ase deficiency) 141.At what three sites can the HMP shunt enter into glycolysis? 1. Fruct ose-6-phosphate 2. Glucose-6-phosphate 3. Glyceraldehyde-3-phosphate 1 142.Deficiency in the liver glycogen phosphorylase enzyme is known as what? Hers disease 143.What causes the lysis of red blood cells by oxidizing agents in a glucose-6phosphate dehydrogenase deficiency? The lack of glutathione peroxidase activ ity results in a decrease in NADPH production, leaving glutathione in the reduce d state. 144.What disease presents with hepatomegaly and a normal EKG? disease, type III (Forbes disease, Cori disease) 145.What apoprotein is on the surface of LDL? Apoprotein B-100 HDLs Apoprote Glycogen storage

146.What carries cholesterol from the tissues back to the liver? 147.What apoprotein mediates the uptake of remnants by the liver? in E 148.What is the complex needed for acetyl CoA carboxylase? G02 (acetyl CoA to malonyl CoA) 149.are the three tissues where TAGS are produced? ipose tissue 150.What delivers cholesterol to the tissues? LDLs

Biotin, ATP, and

1. Liver 2. Muscle 3. Ad

151.What apoprotein is produced by the intestinal epithelium? 152.What carries TAGS to the peripheral tissues? VLDLs

Apoprotein B-48

153.How many NADPHs are used per addition of each of acetyl CoA into a fatty aci d chain? 2 NADPHs/acetyl CoA 154.What apoprotein activates lipoprotein lipase in the capillary epithelium to hydrolyze TAGS? Apoprotein C-II 155.What apoproteins are on the surface of IDL? Apoproteins B-100 and E

156.From which two substances are phospholipids made? acid

1. DAGs 2. Phosphatidic

157.What apoproteins are on the surface of HDL? Apoprotein A-I, C-II, and E 158.What is needed to produce a double bond in a fatty acid chain in the endopla smic reticulum? NADPH, O2, and cytochrome b5 159.What apoprotein activates lecithin cholesterol acyl transferase to esterify cholesterol from tissue? Apoprotein A-I 160.What apoproteins are on the surface of VLDL? and E Apoproteins B-100, C-II,

161.At the end of each round of -oxidation, what is released? Acetyl CoA, FADH , and NADH 162.What two enzymes are vitamin B12 dependent? 1. Homocysteine methyl transfera se 2. Methylmalonyl CoA transferase 163.What enzyme is blocked by disulfiram? Aldehyde dehydrogenase Hormone-

164.What hormone hydrolyzes TAGS to free fatty acids and glycerol? sensitive lipase

165.What enzyme is deficient in a patient 2 years of age or younger who presents with vomiting, lethargy, coma, hypoketosis, and hypoglycemia following a fast o f more than 12 hours? Medium-chain acyl dehydrogenase 166.What form of alcohol did the patient drink if he became blind as a result? Methanol (wood alcohol) 167.What regulates the rate of ketone body formation? The rate of -oxidation

168.What intermediate enables propionyl CoA to enter into the TCA cycle? Succinyl CoA 169.What sphingolipid is formed by the union of serine and palmitoyl CoA? Sphingosine 170.What intermediate of cholesterol synthesis anchors proteins in the membranes and forms CoQ? Farnesyl pyrophosphate (FPP) 171.What is the complex needed for propionyl CoA carboxylase? C02 172.What are the three ketone bodies? yrate Biotin, ATP, and

1. Acetoacetate 2. Acetone 3. -hydroxybut

173.What type of damage to the kidneys is caused by drinking ethylene glycol (an tifreeze)? Nephrotoxic oxylate stones 174.What is the only sphingolipid that contains choline and p04? yelin (lecithin also, but it is not a sphingolipid) Sphingom

175.What is the order of fuel utilization in a prolonged fast? 1. Glucose from liver glycogen 2. Glucose from gluconeogenesis 3. Body protein 4. Body fat 176.Sialic acid and amino sugars are needed to produce what sphingolipid?

Ganglioside 177.What vitamin is needed as a cofactor for decarboxylation and transaminase re actions? Vitamin B6 178.What are the two ways that nitrogen can enter into the urea cycle? 1. Aspar tate 2. Carbomoyl PO4 179.What is the only enzyme in the body that uses N5 methyl folate? eine methyl transferase Homocyst

180.What enzyme deficiency will result in an increase in blood ammonia and an in crease in uracil and orotate concentrations in both the blood and urine? Ornithine transcarbamoylase (OTC) deficiency-also called ornithine carbamoyltran sverase (OCT) deficiency 181.What amino acid is a precursor of Tryptophan What amino acid is a precursor of the e What amino acid is a precursor of the e What amino acid is a precursor of the arginine What amino acid is a precursor of the an What amino acid is a precursor of the the following substances: Serotonin? following substances: GABA? Glutamat

following substances: Histamine? Histidin following substances: Creatine? following substances: NAD? following substances: N20? Glycine/ Tryptoph Arginine

182.What enzyme deficiency will result in an increase in blood ammonia, but no i ncrease in uracil concentrations? Carbamoyl-phosphate synthetase 183.What are the glucogenic and ketogenic amino acids? Phenylalanine, Tyrosine, Tryptophan, Isoleucine, Threonine 184.What are the three diseases of sphingolipids? 2. Gaucher's disease 3. Tay-Sachs disease 1. Niemann-Pick disease

185.What type of jaundice is seen in Rotor's syndrome? Conjugated (direct) hype rbilirubinemia 186.What is the pyrimidine intermediate that joins PRPP? Orotic acid 1. Glyci

187What three amino acids are used to synthesize the purine ring? ne 2. Aspartate 3. Glutamine 188.What enzyme is blocked by hydroxyurea? Ribonucleotide reductase IMP

189.What is the primary end product of purine synthesis? 190.What enzyme is deficient in hereditary protoporphyria? 191.What are the two precursors of heme? 192.What enzyme is blocked by 5-FU?

Ferrochelatase

1. Glycine 2. Succinyl-CoA Thymidylate synthetase

193.What disease has a genetically low level of UDPglucuronate transferase, resu lting in elevated free unconjugated bilirubin? Gilbert's syndrome 194.What form of bilirubin can cross the blood-brain barrier? e bilirubin Unconjugated fre

195.What substrates are needed to produce carbamoyl P04 (de novo pyrimidine synt hesis)? Glutamine, CO2, and ATP via carbamoyl PO4 synthetase II 196.What enzyme is blocked by methotrexate/trimethoprim? ductase 197.What type of jaundice is seen in Dubin-Johnson syndrome? ct) hyperbilirubinemia (It is a transport defect.) 198.What form of bilirubin is carried on albumin? Dihydrofolate re Conjugated (dire

Unconjugated (indirect)

199.In what disease is there a genetic absence of UDPglucuronate transferase, re sulting in an increase in free unconjugated bilirubin? Crigler-Najjar syndrome 200.What enzyme is deficient in acute intermittent porphyria? I synthetase Uroporphyrinogen Uroporph

201.What enzyme is deficient in congenital erythropoietic porphyria? yrinogen III cosynthase 201.What amino acid has a pKa of 13? Arginine 1. Lysine 2. Tyrosine

203.What two amino acids have a pKa of 10? 204.What two amino acids have a pKa of 4? d

1. Aspartic acid 2. Glutamic aci Severe c

205.What disease has a genetic deficiency in adenosine deaminase? ombined immunodeficiency (SCID)

206.What type of charge does the molecule have if the pH is greater than the pI (isoelectric point)? A net negative charge 207.In what disease is there a deficiency in hypoxanthine guanine phosphoribosyl transferase (HGPRT)? Lesch-Nyhan syndrome 208.What is the end product of purine catabolism? Uric acid Purine n

209.What enzyme is deficient in selective T cell immunodeficiency? ucleoside phosphorylase 210.What substrate builds up in Tay-Sachs disease? Ganglioside GM2

211.On a Lineweaver-Burke plot, what type of binding has both plots crossing the y-axis in the same spot? Competitive, reversible inhibition (Vmax is the same, increase Km) 212.What toxin ADP-ribosylates via GS protein to increase CAMP? Cholera toxin 213.What vitamin derivatives are used for growth and differentiation of epitheli um for reproductive and embryonic development? Vitamin A 214.Light causes isomerization of what in the eyes? retinal (activated rhodopsin) 215.What are the two actions of calcitonin? the kidney 2. Increases bone mineralization 11-cis-retinal to traps-

1. Increases Ca2+ excretion from

216.What causes an increase in bone mineralization and Cat+, as well as P04 abso

rption from the GI tract and kidney tubules?

Vitamin D

217.On the Lineweaver-Burke plot, what type of binding has both plots crossing t he x-axis in the same spot? Noncompetitive, reversible binding (decrease in Vmax; Km is same) 218.What is the maximum rate possible with a given amount of enzyme? Vmax

219.To what are intracellular glucose levels inversely related? cAMP levels 220.Does a saturated fatty acid have double bonds? ids have double bonds No; unsaturated fatty ac

221.What two factors cause PTH to be secreted? 1. A decrease in blood Ca2+ 2. A decrease in PO4 concentrations 222.What in the human genome differs in each individual that can serve as an ide ntification marker? RFLP-restriction fragment length polymorphism 223.What test is used to identify HIV-positive patients? ked immunosorbent assay ELISA-enzyme-lin

224.What toxin ADP-ribosylates via G to increase cAMP? Pertussis toxin 225.What vitamin is essential for normal Ca2+ and P04 metabolism? D Vitamin

226.What vitamin is deficient in a person who has impaired taste, night blindnes s, and increased risk for having an abortion? Vitamin A 227.What bond does an endonuclease cleave? e bond 3', 5' internal phosphodiesteras

228.What protein separates base pairs and unwinds the DNA at the replication for k? Helicase (It is an ATP-dependent enzyme.) 229.What vitamin deficiency would cause liver necrosis and red blood cell fragil ity? Vitamin E deficiency 230.What protein catalyzes the formation of the last phosphodiester bond (PDE) b etween the Okazaki fragments to produce a continuous strand? DNA ligase 231.What is the hypochromic effect? uble stranded to single stranded Increased absorption as DNA goes from do Southern blotting

232.What technique uses DNA for analysis?

233.In what direction is a new DNA strand made? 5' to 3' 234.What enzyme has a 5' to 3' synthesis activity and a 3' exonuclease activity? DNA polymerase III 235.What enzyme makes a double-stranded cut through DNA, needs ATP, and introduc es negative supercoiling? Topoisomerase II 236.What eukaryotic DNA polymerase is used for: DNA replication? elta-polymerase a- and d

237.What eukaryotic DNA polymerase is used for: Replication in mitochondria? gamma-Polymerase

238.What eukaryotic DNA polymerase is used for: DNA repair? 239.What is the orientation of the gene sequence strand? s RNA) 240.What is the location of the TATA box in procaryotes? eam 241.What binds to the promoter region in procaryotes?

-Polymerase 5' to 3' (same a 10 bases downstr

Sigma factor

242.What enzyme has a 5' to 3' synthesis of the Okazaki fragments, 3' exonucleas e activity, and 5' exonuclease activity? DNA polymerase I 243.What type of mutation has the same amino acid coded for, but with a differen t codonsequence? Silent mutation 244.At what position of the anticodon does the "wobble" hypothesis occur? Position 1 of the anticodon (the 5' end)- which is the same as position 3 of the codon (the 3' end) 245.What are the three "stop" codons? 1. UAA 2. UAG 3. UGA Rho factor or a hairpin loop

246.What stops transcription in procaryotes?

247.What are the three post-transcriptional modifications? 1. 7-methyl guan ine cap on the 5' end 2. Addition of the poly(A) tail to the 3' end 3. Removal o f introns 248.In what type of mutation is a different codon added, resulting in formation of a different amino acid? Missense mutation 249.What amino acid is attached to the 3' end of the tRNA in eukaryotes? Methionine 250.What enzyme makes tRNA and the SsRNA? RNA polymerase III

251.What structure of a protein describes the interaction among subunits? Quaternary structure 252.What two amino acids require vitamin C for hydroxylation? sine 1. Proline 2. Ly The fold

253.What is determined by the secondary structure of an amino acid? ing of an amino acid chain

254.Which mutation has a stop codon put in place of the previous codon? Nonsense mutation 255.What amino acid is attached to the 3' end of the tRNA in prokaryotes? f-Methionine 256.What enzyme makes rRNA (barring the 5s subunit)? 257.What is the site of action of cycloheximide? S) RNA polymerase I Peptidyl transferase (60

258.What translation factor is blocked by erythromycin? Elongation factore G (EF -G)

259.What is the charge of the molecule if the pH is less than the pI (isoelectri c point)? A net positive charge 260.What technique uses the separation of proteins on a gel electrophoresis? Western blot 261.Lack of what vitamin causes multiple carboxylase deficiency? 262.What enzyme makes hnRNA/mRNA? RNA polymerase II Cystathionase Branched-chain k Frameshi Biotin

263.What enzyme is deficient in cystathioninuria?

264.What enzyme is deficient in maple syrup urine disease? eto acid dehydrogenase

265.What type of mutation has the addition or deletion of a base? ft 266.What is the site of action of puromycin? Aminoacyl tRNA (A site)

267.What translational factor is blocked by both diphtheria and Pseudomonas toxi ns? Elongation factor 2 (EF-2) 268.What substrate gets built up in Gaucher's disease? Glucosyl cerebroside 269.What enzyme is deficient in homocystinuria? Homocysteine methyl transferase or cystathionine synthetase 270.What substrate is built up in Niemann-Pick disease? Tyrosine kinase 271.What is the second messenger system used by growth factors? Sphingomyelin 272.What is the name of the sequence on mRNA that precedes the start codon in pr okaryotes? Shine-Dalgarno sequence 273.What amino acid undergoes N-glycosylation? Asparagine 274.What translational factor is blocked by tetracycline? r Tu (EF-Tu) 275.What translational factor is blocked by streptomycin? 276.Which organisms have polycistronic mRNA? okaryotes both start with P. Elongation facto IF-2

Prokaryotes: Polycistonic and pr

277.In what disease are lysosomal enzymes released into the extracellular space where an accumulation of inclusion bodies compromises the cell's function? I-cell disease 278.What is the site of action of chloramphenicol? s) 279.What is needed to initiate translation? Peptidyl transferase (50

IF and GTP (OF for eukaryotes)

280.What is the name of the process of going from DNA to mRNA? Transcription 281.Is the hydroxyl (-OH) end of DNA and RNA at the 3' or the 5' end? Phosphate (P04) is at the 5' end. 3' end.

282.What is the only factor of enzyme kinetics that the enzyme affects? Ea (acti

vation energy) 283.What tumor suppressor gene prevents a cell from entering S phase when no gro wth factors are present? Rb gene 284.What tumor suppressor gene prevents a cell with damaged DNA from entering th e S phase? p53 gene 285.What factors are needed for elongation in prokaryotes? nd GTP 286.What factors are needed for translocation in eukaryotes? EF-Tu or EF-is a EF-2 and GTP

287.What type of inheritance involves carriers, affects only males, and skips ge nerations? X-linked recessive 288.What type of separation technique uses RNA on an electrophoresis? blotting 289.What factors are needed for elongation in eukaryotes? 290.What factors are needed for translation in prokaryotes? Northern

EF-1 and GTP EF-G and GTP

291.What type of inheritance has no male-to-male transmission, and every daughte r is affected from the father in every generation? X-linked recessive 1.What virus is associated with intranuclear inclusions known as "owl's eye" inc lusions? Cytomegalovirus (CMV) 2.What virus is associated with Negri bodies? Rabies virus Variola virus -7-ox vtrc

3.What virus is associated with Guarnieri bodies? s

4.What virus causes small, pink, benign wart-like tumors and is associated with HIV-positive patients? Molluscum contagiosum 5.What viruses are associated with cervical carcinoma? Human papilloma viruses (HPVs) 16 and 18 6.What virus is associated with erythema infectiosum or fifth disease? Parvovir us B-19 7.What virus binds to: CD4? Human immunodeficiency virus (HIV) Cytomegalovirus (CMV) Epstein-Barr virus (EBV) - Qp7_1

8.What virus binds to: 2-Microglobulin? 9.What virus binds to: Complement factor C3?

10.What virus binds to: ACh receptors? Rabies virus 11.What is the only dsRNA virus? Reovirus 1. Picornavirns 2. Calic

12.What are the three non-enveloped RNA viruses? ivirns 3. Reovirns (PCR)

13.What viruses are associated with Cowdry type A intranuclear inclusions? Herpes virus I and II

14.What virus is associated with the Norwalk agent?

Calicivirns Poliovirus rotavirus

15.What virus affects the motor neurons in the anterior horn? 16.What is the most common cause of diarrhea in children? 17.What virus lies dormant in the: Trigeminal ganglia? Herpes I

18.What virus lies dormant in the: Dorsal root ganglia? Varicella 19.What virus lies dormant in the: Sensory ganglia of S2 and S3? I 20.With what virus are Downey type II cells associated? EBV 21.What are the four segmented RNA viruses? 3. Reovirns 4. Arenavirus (BORA) 1. Bunyavirns 2. Orthomyxovirus Herpes I

22.Koilocytic cells on a Papanicolaou (Pap) smear are indicative of what virus? HPV 23.What bacteria constitute the most common cause of nosocomial infections in bu rn patients and in patients with cystic fibrosis? Pseudomonas 24.What organism is associated with pneumonia acquired from air conditioners? Legionella 25.What species of bacteria is associated with whooping cough? Bordetella pertu s,sis 26.What two bacteria are associated with drinking unpasteurized milk? lla 2. Listeria (has tumbling motility) 27.What organism is associated with gastritis and ulcers? ori 1. Bruce

Helicobacter pyl Escheric

28.What species of bacteria is associated with traveler's diarrhea? hia coli (enterotoxic) 29.With what organism is "currant jelly" sputum associated? oniae 30.What is the most common cause of enterocolitis?

Klebsiella pneum

Salmonella enteritidis

31.What organism is so infective that it takes only 1 to 10 organisms to cause a n infection? Shigella 32.What organism stains bipolar and causes buboes? 33.Rice water stools are indicative of what organism? Yersinia pesos Vibrio cholerae

34.With what organism are intracellular gram-negative inclusions in neutrophils associated? Neisseria gonorrhoeae 35.What organism is most likely to cause an infection if you are bitten by a dog or a cat? Pasteurella 36.What organism needs factor X and NAD in order to grow on growth medium? Haemophilu.s influenzae type B

37.What organism is associated with a rigid belly and rose spots on the belly? Salnwnella typhi 38.What organism will cause an infection if undercooked or raw seafood is eaten? Vibrio parahaemolyticus 39.What organism is likely to infect you if you get cut by a shell at the beach? Vibrio vulnificus 40.What infective bacteria are found in undercooked hamburgers? Escherichia coli strain 0157:H7ss 41.What organism is said to have a "spaghetti and meatball" arrangement under a microscope? Malassezia furfur 42.What fungus is associated with rose gardener's disease? ckii Sporothrix schen

43.What fungus is seen as colored cauliflower lesions? Chromomycosis 44.What fungus is found in soil with bird or bat feces? Histoplasma cap.sulatum 45.Which organism causes San Joaquin fever? Coccidioides immitis Candida albicans

46.What fungus causes endocarditis in IV drug users? 47.What fungus is found in pigeon droppings?

Cryptococcus neoformans

48.What fungus is seen as a yeast with broad-based buds and a double refractile cell wall? Blastomyces dermatitidis 49.What fungus is stained positive with India ink? Cryptococcus neoforrnans

50.What virus is the most common causative agent of the common cold in the summe r and the fall? Rhiuovirus 51.To what family of viruses do dengue, St. Louis, and yellow fever belong? Flavivirus 52.What is the name of the bullet-shaped virus? Rhabdovirus 53.What virus is responsible for causing the croup and also the common cold in t he young and the old? Paraiuflueuza virus 54.What is the causative agent of orchitis, parotids, and pancreatitis? Mumps vi rus 55.What virus causes hoof-and-mouth disease? Vesicular stomatitis virus

56.What is the most common cause of pneumonia in children 1 year old or younger? Respiratory syncytial virus (RSV) 57.What virus is the most common cause of the common cold in the winter and earl y spring? Coronavinzs 58.What yeast is urease positive? Cryptococcus neoformans

59.What fungus is a facultative intracellular parasite of the reticular endothel ial system? Histoplasma capsulatum

60.What virus is helical and has HN and F glycoprotein spikes? Paramyxovirus 61.What is the most common cause of pneumonia in persons with underlying health problems? Klebsiella pneumoniae 62.What is the most common cause of pneumonia in young children? ma 63.What virus causes epidemic keratoconjunetivitis? 64.What is the most common cold virus? Rhinovirus 65.What two viruses have neuraminidase activity? 66.What is the most common cause of diarrhea in infants? 67.What is the reservoir for the togavirus? Birds 1. Influenza 2. Mumps Rotavirus Adenovirus Mycoplas

68.What two viruses cause pancreatitis? 1. Mumps 2. Coxsackie B40 69.With what two viruses are Reye's syndrome associated? us 2. Influenza virus 1. Varicella vir

70.What is the most common cause of meningitis in: Children younger than 3 month s of age? Streptococcus agalactiae or Escherichia Coli 71.What is the most common cause of meningitis in: Non-immunized children 12 mon ths to 6 years old? Haemophilus influenzae type b 72.What is the most common cause of meningitis in: Immunized children 12 months to 6 years old? Streptococcus pneumoniae 73.What is the most common cause of meningitis in: Military recruits? a meningitidis Neisseri

74.What is the most common cause of meningitis in: HIV+/immunocompromised person s? Cryptococcus neoformans 75.What is the most common cause of meningitis in: Adults? eunwniae Streptococcus pn

76.What is the most common cause of bronchiolitis in children? RSV 77.What is the most common cause of urinary tract infections? Escherichia coli 1. CMV 2

78.Which three organisms cause heterophilic negative mononucleosis? . Toxoplasma gondii 3. Listeria 79.What two genera are spore formers? 1. Clostridia 2. Bacillus

80.What bacteria are responsi- ble for woolsorters' disease? is 81.What is the most common cause of bacterial pneumonia? eumoniae

Bacillus anthrac Streptococcus pn

82.What bacteria cause subacute endocarditis and dental caries? Streptococcus vi ridans 83.Which two organisms can cause sulfur granules in the pus? 1. Actinomyces 2

. Nocardia 84.What species of bacteria is responsible for causing endocarditis in IV drug u sers? Staphylococcus epidermidis 85.What bacteria are responsible for food poisoning from rice, fried rice, and r eheated foods? Bacillus cereus 86.Which bacteria present as a common cause of meningitis in renal transplant pa tients? Listeria 87.What bacteria get inoculated into the body by a puncture wound in the skin an d also inhibit glycine and GABA? Clostridium tetani 88.What bacteria are found in poorly preserved canned food and cause flaccid par alysis? Clostridium botulinum 89.What bacteria cause myonecrosis? Clostridium perfringens Clostridium difficile

90.What bacteria cause pseudomembranous colitis?

91.What bacteria are associated with food poisoning from ham, potato salad, and custards? Staphylococcus aureus 92.What three bacteria are quellung reactive test positive? 1. Neisseria men ingitidis 2. Haemophilus influenzae 3. Streptococcus pneumoniae 93.Which organism causes a painful chancre? Haemophilus ducreyi RSV Bacillus fragilis

94.What is the most common cause of viral pneumonia? 95.What is the predominant anaerobe in the colon? 96.Which organism causes trench mouth? Fusobacterium

97.Which organism causes Lyme disease? Borrelia burgdorferi 98.Which organism causes Weil's disease? 99.What organism causes Q fever? Leptospira Coxiella burnetii Mycoplas

100.agent causes pneumonia in college students and military recruits? ma pneumoniae

101.What is the tetrad of Jarisch-Herxheimer reaction? Rigors, leukopenia, decr ease in blood pressure, and increase in temperature 102.Which spirochete causes Rocky Mountain spotted fever? ttsii (wrist to trunk rash) 103.Which organism causes trench fever? Rochalimaea quintana 104.Which organism causes epidemic typhus? periphery rash) Rickettsia prowazekii (trunk to Chlamydia psittaci Borrelia Rickettsia ricke

105.Which organism causes pneumonia in bird owners?

106.Which organism causes multiple infections by antigen switching? recurrentis

107.Which organism has protein A for an anti-opsonization defense? coccus aureus 108.Which organism releases endotoxins PRIOR to cell death? itidis 109.What is the only ssDNA virus? Parvovirus Poxvirus

Staphylo

Neisseria mening

110.What is the only DNA virus to replicate in the cytoplasm? 111.What are the three naked DNA viruses? apovavirus (PAP)

1. Parvovirus 2. Adenovirus 3. P

112.What is the only DNA virus that has the reverse transcriptase enzyme? Hepadnavirus 113.Which hepatitis virus is an RNA viroid-like virus that needs hepatitis B to be infective? Hepatitis D 114.Which hepatitis virus is an enveloped RNA flavivirus, which is known for pos tinfusional hepatitis? Hepatitis C 115.What antigen is needed to diagnose an infectious patient with hepatitis B? Hepatitis Be antigen 116.Which type of hepatitis can cause hepatocellular carcinoma? Hepatitis B 4- C 117.Which type of hepatitis is a calicivirus? Hepatitis E (enteric)

118.Which type of hepatitis is a picornavirus? Hepatitis A (infectious) 119.What two antigens need to be positive for a patient to have chronic active h epatitis? 1. Hepatitis Bs 2. Hepatitis A antigen 120.In the window phase of a hepatitis B infection, which antibodies do you see? Hepatitis Be and c antibodies; You see the antibodies c and e. 121.Which virus is found in the urine of rodents? Arenavirus HTLV-I and HTLV-

122.Which virus is associated with hairy T cell leukemia? II 123.What are the components of the rubella triad? sus (PDA) 2. Cataracts 3. Mental retardation

1. Patent ductus arterio Vidarabine

124.What is the drug of choice to treat: HSV encephalitis?

125.What is the drug of choice to treat: RSV pneumonia? Ribavirin 126.What is the drug of choice to treat: CMV retinitis/infection? vir 127.What is the drug of choice to treat: Influenza A? 128.What is the drug of choice to treat: HSV? 129.What is the drug of choice to treat: HPV? Amantadine Ganciclo

Acyclovir Interferon alpha

130.What does Candida albicana do that distinguishes it from other fungi? It forms a germinal tube at 37C.

131.Which organism, transmitted by sexual contact, is almost diagnostic by the f oul-smelling, green discharge from the vagina and its associated itch? Trichomo nas vaginalis 132.Which organism is associated with a diffuse bilateral interstitial pneumonia and with HIV-positive patients with CD4 counts of 200 or lower? Pneumocy sti.s carinii 133.What three organs can be affected by Trypanoaoma cruzi? 1. Heart 2. Esop hagus 3. Colon Remember-you get "megas:" 1. Cardiomegaly 2. Megaesophagus 3. Meg acolon 134.What organism causes kala-azar, which is associated with hyperpigmentation o f the skin, enlargement of the spleen, and decreased bone marrow activity? Leishmania donovani 135.Which type of malaria is associated with dark urine? parum (malignant) 136.What people are "protected" from malaria? cell trait Plasmodium falci

People with heterogenous sickle

137.What type of Plasmodium is banana or crescent shaped when stained with Giems a stain? Plasmodium falciparum 138.What is the only Plasmodium that is quartan? others are tertian. Plasmodium malarize; the

139.What types of Plasmodium produce latent hypnozoites in the liver, which can cause a relapse? Plasmodium vivax and Plasmodium ovale 140.What organism is associated with liver abscess, t ulcers, and perforated dia phragms? Entamoeba histolytica 141.What type of Plasmodium affects: Only mature RBCs? Plasmodium malariae 142.What type of Plasmodium affects: Only redculocytes? Plasmodium vivax 143.What type of Plasmodium affects: RBCs of all ages? Plasmodium falciparum 144.What three carcinomas are associated with EBV? Nasopharvngeal 3. Thymic 1. Burkitt's lymphoma 2.

145.What is the direction of the strand if a virus has infectious +RNA? 5' to 3' RNA 146.What two viruses do not get their envelope from budding but actually from co ding? HIV and poxvirus 147.What glycoprotein in the HIV virus is used for fusion? 148.What glycoprotein in the HIV virus attaches to CD4? GP120 149.What protein of the HIV virus is used to detect if a patient is HIV-positive by ELISA? P24 150.What two viruses cause progressive multifocal encephalitis? 1. Simian (SV40) virus 2. JC virus GP41

151.With what virus do you see Koplik's spots and Worthin- Finkeldy cells, and p ossibly subacute sclerosing panencephalitis? Rubeola (measles) 152.What are known as jumping genes? Transposons

153.What is the most common viral cause of myocarditis? Coxsackie B 154.What virus is associated with heterophil-positive mononucleosis? 155.What is the only virus to be eradicated? Smallpox virus EBV

156.What virus, which creates painful vesicular lesions, is a cause of aseptic m eningitis? Herpes simplex II 157.What virus attaches to fibroblastic growth factor? Herpes simplex I 158.What is the most prevalent coral infection in the USA? virus 159.What is the only herpes virus to cross the placenta? 160.Of what virus are Guarnieri bodies diagnostic? Smallpox Varicella-zoster CMV

161.What cells are atypical on a peripheral blood smear in a heterophil-positive mononucleosis? T cells not the B cells 162.What type of hepatitis has the highest mortality rate among pregnant women? Hepatitis E 163.What does hepatitis D virus need from hepatitis B virus to be infective? Hepatitis Bs antigen as its envelope 164.What are the two hepatitis viruses that can be chronic and can lead eventual ly to hepatocellular carcinoma? 1. Hepatitis B 2. Hepatitis C 165.What are the only two viruses where naked dsDNA is NOT infectious? 1. Poxvi rus 2. Hepatitis B virus 166.What is the only diploid virus? Retrovirus

167.What types) of immune response is the body capable of making when presented with a live vaccine? Humoral and cell mediated 168.What HPV is the causative agent of anogenital warts? HPV 6 and 11

169.What types) of immune response is the body capable of making when presented with a killed vaccine? Humoral only 170.Who are the "typical" women who present with endometrial carcinoma? Older, n on-sexually active women (whereas young, sexually active women present with cerv ical carcinoma) 171.What type of vaccine is the MMR vaccine? Live, attenuated vaccine Measles

172.With what virus is postauricular lymphadenopathy associated? (rubella) virus

173.In what trimester is the fetus most vulnerable to congenital rubella syndrom e? The first trimester

174.What is the order of the antibodies, from first to last, in an infected pati ent with hepatitis? Hepatitis Be, e, s 175.What is the first antigen seen in an individual with hepatitis? s Bs antigen (incubation period) Hepatiti

176.What antibody is an indication of low transmissibility for hepatitis? Hepatitis Be antibody 177.What antibody is an indication of recurrent disease for hepatitis? Hepatiti s Be antibody 178.Antibodies to what hepatitis B antigen provide immunity? patitis Bs antigen Antibodies to he

179.What are the three C's of measles? 1. Cough 2. Coryza 3. Conjunctivitis 180.What vector is associated with malaria? 181.What is the vector for yellow fever? Anopheles mosquito Aedes mosquito

182.What are the only two picornaviruses that do NOT lead to aseptic meningitis? 1. Rhinovirus 2. Hepatitis A virus 183.Are antibiotics helpful in treating a disease caused by a prion? ns are infectious proteins; thus, antibiotics are useless. No. Prio

184.What is the only part of the virus that is "detectable" during the eclipse p eriod of the viral growth cycle? The viral nucleic acid . 185.What is the only virus to carry its own ribosomes? Arenavirus 186.What is the term for the period from the onset of an infection to the appear ance of the virus extracellularly? Latent period 187.What is the leading cause of diarrhea in the USA? IA----ROTAVIRUS} Campylobacter jejuni{IND

188.What organism would cause a patient to present with constant diarrhea after drinking mountain stream water on a camping trip? Giardia lamblia 189.What parasite can cause vitamin B12 deficiencies? 190.What viral infection is associated with black vomit? avivirus) Diphyllobothrium latum Yellow Fever (fl

191.What are the two nonspecific chemical defenses of the body? Acidic pH and ly sozymes 192.What are the two nonspecific physical defenses of the body? Skin and mucus 193.What immunoglobulin is the first antibody in an immune response? 194.What is the major antibody of external secretions? IgA 195.What is the major antibody of internal secretions (blood, CSF, lymph)? IgG 196.What is the valence of an immunoglobulin molecule equal to? The number of an tigens that the antibody can bind IgM

197.What immunoglobulin is a marker for mature B cells and is the antigen recept or for B cells? IgD 198.What is the only IgG that cannot bind to Staphylococcus protein A? IgG3 199.By which process do antibodies make microorganisms more easily ingested via phagocytosis? Opsonization 200.Which immunoglobulin is found as a pentamer and activates complement? IgM 201.What is synthesized by epithelial cells, protects IgA from degradation, and transports IgA across epithelial barriers? Secretory component of IgA 202.Which IgG cannot activate complement? IgG4

203.What cell surface marker is used to distinguish different stages in the matu ration of T cells? CD3 204.What form of graft involves tissue or organ transplantation between genetica lly identical twins? Isograft (isogenic graft, syngraft) 205.True or false: antigen. antibody binding is irreversible? False; it is rev ersible because the antigens and antibodies are not linked covalently. 206.What is the term for a single isolated antigenic determinant? not immunogenic) Hapten (

207.Which chromosome is associated with major histocompatibility complex (MHC) g enes? Chromosome 6 208.Which region of the variable domain comprises the antigen-binding site of th e antibody? Hypervariable region (three per light chain; three per heavy cha in) 209.What form of transplantation crosses the species barrier? 210.What subdivision of MHC is found on all nucleated cells? ee subtypes: -A,-B,-C) Xenograft MHC class I (thr

211.What protein is used to differentiate MHC class I from MHC class II, and on what chromosome is it found? 2-Microglobulin, on chromosome 15 212.What cell type recognizes MHC class I? Cytotoxic T cell (CD8)

213.What is the predominant antibody of a secondary immune response in the mucos al route? IgA 214.What substance is secreted by activated helper T cells to induce T and B cel l division? IL-2 215.What type of antigen do B cells recognize? Free, unprocessed antigen 216.What type of antigen do T cells recognize? Processed antigenic peptides bou nd in the groove of the MHC molecule 217.Which protein prevents internal binding of self proteins within an MHC class II cell? Invariant chain

218.In a binding of helper T cells and an antigen-presenting cell (APC), which i s the first cell to secrete activating signals? Helper T cells (cytokines to act ivate the APCs) 219.Which type of cell is responsible for immunologic memory? Memory B cell

220.What region of the immunoglobulin does not change with class switching? Hypervariable region 221.What type of cell does an antigen-stimulated B cell turn into if there is a continuous supply of antigen? Plasma cell 222.What three cells are essential for T cell differentiation in the thymus? 1. Dendritic cells 2. Macrophages 3. Thymic epithelial cells 223.What type of T cell leaves the bone marrow? Pre-T cell (unable to recognize antigen) 224.What cytokine stimulates stem cell differentiation? IL-3 225.What form of T cell binds to mature B cells? Activated helper T cell

226.What co-stimulatory molecules are necessary for effective T cell and B cell signaling? B7 and CD28 (stimulatory signal for the T cell) 227.At which site of the lymph node are B cells found? The germinal centers (fo llicles) 228.What are the two primary lymphoid organs? 1. Bone marrow 2. Thymus

229.What cytokine, produced by stromal cells of the bone marrow, is important in myeloid development? IL-3 A " 3" on its side looks like an m for myeloid 230.What immunoglobulins are found in an infant at birth? fetal IgM 231.What cell surface marker is found on blood B cells? CD19 232.What cell surface marker do all T cells have? CD3 The lymp Maternal IgG and

233.What is the major antigen-trapping site in the immune system? h node

234.What cells are the first to come into contact with soluble antigen in the ly mph node? Macrophages or dendritic cells 235.In which region of the lymph node do plasma cells spend their lives secretin g antibodies? Medulla 236.What cell surface marker is found on activated helper T cells? 237.In which region of the lymph node are T cells found? CD40

Paracortex

238.Myeloperoxidase uses H202 and what to generate additional oxidants? Halide c ofactor (Cl-, I-) 239.What is the main cell type of chronic inflammation? Macrophages 240.What aspect of the complement system is deficient if there are repeated gono coccal infections and recurrent episodes of meningococcal meningitis? C5, 6 ,7

, or 8 241.A deficiency in C1 esterase (C1-INH) results in what disease? ry angioedema Heredita

242.What form of immunity kills the host cell in order to recover from intracell ular infections? Cell-mediated immunity 243.With what area of the spleen are the T cells associated? ymphatic Sheath (PALS) 244.What are the three secondary lymphoid tissues? 3. Mucosal-Associated Lymphoid Tissue (MALT) 245.What type of cell can never leave the lymph node? 246.What is the B cell-dependent area of the spleen? Periarteriolar L

1. Lymph nodes 2. Spleen Plasma cell The marginal zone RBCs-tha

247.Which major cell type is found in the red pulp of the spleen? t is why it is called red pulp.

248.At what stage of the immune response do you see an increase in serum specifi c antibody levels? Log phase 249.What cytokines do APCs secrete to activate helper T cells? IL-1, IL-6, TNFa 250.What immunoglobulin is responsible for antibody- dependent cell-mediated cyt otoxicity (ADCC) of parasites, has a high-affinity Fc receptor on mast cells and basophils, and is responsible for the allergic response? IgE 251.What immunoglobulin is responsible for activation of complement, opsonizatio n, and ADCC, and is actively transported across the placenta? IgG 252.What immunoglobulin activates the alternate pathway, neutralizes bacterial e ndotoxins and viruses, and prevents bacterial adherence? IgA 253.What are defined by antigen-binding specificity? Idiotypes

254.What are the genetic variants of a molecule within members of the same speci es? Allotypes 255.What are different classes and subclasses of the same gene products known as ? Isotypes 256.What are the four phases of immune system defense? 1. Recognition 2. Amplif ication 3. Regulation 4. Elimination-Rare 257.What is the term for a molecule that will trigger an immune response? Immunogen (It must be foreign and have at least two antigenic determinants.) 258.What MHC class does not participate in immune cell communication by direct c ontact? MHC class III 259.What types of T cell receptors (TCRs) comprise 95% of all TCRs? d beta chains 260.Which TCRs are found on the skin and mucosal surfaces? TCRs Alpha an

Gamma and delta

261.What type of graft transplants from one individual to another with a differe nt genetic makeup (within the same species)? Allograft 262.What type of graft transplants from one site to another on the same person? Autograft 263.What is the term for the strength of association between multiple antibody-b inding sites and multiple antigenic determinants? Avidity (greater than on e binding site) 264.How is IgA found in secretions? . As a dimer; it is a monomer in the blood

265.What is the limited portion of a large antigen that will actually be recogni zed and bound to an antibody and contains approximately five to six amino acids or four to five hexose units? Antigenic determinant (epitope) (Idiotypes bind to epitopes.) 266.What would be the result if an antibody were cleaved with papain? uld be two Fab and Fc regions. There wo

267.What would be the result if an antibody were cleaved with pepsin? There wo uld be a Fab' region; thus, it would still be able to participate in precipitati on and agglutination. 268.What type of binding is involved when there is binding of one Fab or one idi otype of IgG? Affinity 269.What cytokines are secreted by helper T cells to activate the APC? INF-y an d IL-4 270.What acts as a target for elimination of abnormal host cells? s I MHC clas

271.At what stage of an immune response do we see stable levels of antibody in t he serum? Plateau phase 272.At what stage of an immune response do we see catabolism without synthesis o f antibody, causing a decline in the levels? Antigen processing 273.What stage of the immune response is involved from the time when we are firs t presented with an antigen to the first time that there are detectable levels o f antibody in the serum? Lag phase 274.What response involves elevated levels of antibody and a short lag phase, an d requires low levels of antigen to precipitate? Secondary response 275.Whose function is it to present exogenous peptides to helper T cells? MHC class II 276.What process is involved when an antigen is in the endocytic vacuole and the re is fusion with lysosomes, which contain proteases that cleave the protein ant igens into peptide fragments? Antigen processing 277.Which four helper T cell cytokines are involved in differentiation? I. IL-4 2. IL-5 3. IL-6 4. IL-10 278.Which co-stimulatory molecules are necessary for B cell differentiation (cla ss switching)? CD40 and CD40L (gp39)

279.Which cytokine is chemotactic for neutrophils?

IL-8

280.Which nondividing cells synthesize immunoglobulins in great amounts? Plasma cells 281.Which process is involved in rearranging the DNA that encodes for the consta nt region of the heavy chain? Class switching 282.What are the two functions of the thymus in T cell differentiation? 1. Hormo ne secretion for T cell differentiation 2. T cell education to recognize self fr om nonself 283.Which process is involved in rearranging one heavy chain gene to produce a f unctional gene product, while it shuts off the rearrangement and expression of t he other alleles to ensure that one antibody type is made? Allelic exclusio n 284.What are the three rules of clonal selection? 1. One cell type 2. One antibody type 3. Random selection of hypervariable regions, and only cells with bound antigen undergo clonal expansion 285.When is the last time that maternal IgG is seen in circulation? 9 and 15 months Between

286.What cytokine, produced by stromal cells of the bone marrow, is important in lymphoid development? IL-7 A "7" upside down looks like an "L" (L for Lymphoid ). 287.Which region of the thymus contains the mature T cells? y mature in the cortex.) 288.What are the cell surface markers of helper T cells? 289.What are the cell surface markers of cytotoxic T cells? 290.Which complement fragments cause lysis of cells? C5b-9 The medulla (The CD28 and CD4 CD28 and CD8

291.Which complement fragment is deficient if a patient presents with repeated i nfections, fever, rash, and arthralgia? C3 292.What three factors cause opsonization? otein IgG, C3b, and mannose binding pr CD21

293.Which cell surface marker binds to C3d fragments?

294.What disease is associated with an inherited deficiency in NADPH oxidase, in which the individual is likely to develop infections with catalase-positive org anisms? Chronic granulomatous disease 295.What cells are antigen-specific and have MHC restricted killing? c T cells Cytotoxi

296.What large granular lymphocytes have CD16 and CD56 as cell surface markers, do not secrete immunoglobulins, and are not antigen specific? NK cells 297.What three complement fragments are also anaphylatoxins? C5a 1. C3a 2. C4a 3.

298.What pathology is associated with low levels of immunoglobulin that persist in children for up to 2 years? Transient hypogammaglobulinemia of childhood

299.What cell surface marker is found on pre-T cells?

Tdt CD40

300.What cell surface marker is required for class switching?

301.What large granular lymphocytes are stimulated by IL-2 and INF-y and are act ivated by natural killer (NK) cells? Lymphokine-activated killer (LAK) cells 302.Which cells recognize the Fc region of IgG and carry out ADCC? ells 303.What are the four chemotactic agents? 4. Bacterial peptides Killer c

1. C5a 2. Leukotriene B4 3. IL-8

304.What are the five main oxidizing reactions that are used to kill ingested or ganisms? 1. H202 2. Superoxide 3. Hydroxyl radical 4. Myeloperoxidase 5. Hypochlorous acid 305.True or false-it is perfectly normal to have low levels of IgG from 3 to 12 months of age? True; it is called physiologic hypogammaglobulinemia of infancy. 1.What valve of the heart is most commonly affected in IV drug abusers? Tricuspi d valve 2.What is the first sign of reversible cellular damage? Ballooning or hydropic c hanges secondary to mitochondrial injury 3.What is another name for isolated right-sided heartfailure? Cor pulmonale

4.What region of the aorta is affected in syphilitic (luetic) aneurysms? Ascending arch or the root 5.What organism is associated with hyaline membrane formation and cold agglutini ns? Mycoplasma 6.What cell type is commonly elevated in asthma? 7.What enzyme level is commonly elevated in sarcoidosis? erting enzyme (ACE) Eosinophils Angiotensin conv Malignan

8.What form of lung cancer is commonly associated with asbestosis? t mesothelioma

9.With what protein-losing enteropathy do you see hypertrophied gastric rugal fo lds? Menetrier's syndrome 10.What region of the GI tract does Giardia lamblia most commonly affect? Duodenum 11.What organism is associated with "flask-shaped" ulcers? ytica Entamoeba histol

12.What hepatic pathology is associated with ingestion of oral contraceptives? Liver adenoma 13.True or false alpha-1-antitrypsin deficiency can lead to cirrhosis of the liv er? True; it is most commonly associated with panacinar emphysema of the low er lobes.

14.What organism is associated with cholangiocarcinoma? Clonorchis sinensis 15.True or false-polycythemia is associated with renal cell carcinoma? True, be cause there is an increase in erythropoietin 16.What cancer of the male genitourinary system is associated with osteoblastic bony metastasis? Prostatic carcinoma 17.What organism is commonly associated with IUD infections? Actinomyces

18.What female genitourinary pathology is associated with elevated levels of hum an chorionic gonadotropin (hCG) and has a "snowstorm" appearance on an ultrasoun d? Hydatidiform mole 19.What subset of leukemia involves gingival hypertrophy? leukemia (AML)-M5, also called acute myelogenous leukemia Acute myelocytic

20.What chemical pathogen is associated with squamous cell carcinoma of the skin ? Arsenic 21.With what form of cancer is diethylstilbestrol (DES) associated? clear cell adenoma 22.What form of cancer is most commonly associated with nitrosamines? cancer Vaginal Gastric

23.What cellular process defines irreversible cellular injury? Vacuolation of t he mitochondria 24.Antinuclear antibodies (ANA), anti-dSDNA, and smooth muscle antigen (Sm ag) a re all used to diagnose what disease? Systemic lupus erythematosus (SLE) 25.SS-A(Ro), SS-B(La), and R-ANA are diagnostic markers of what disease? Sjogren's disease 26.What disease is characterized by decreased bone resorption due to defective o steoclast function? Osteopetrosis (Albers- Schonberg disease) 27.What metaplastic cellular change (from what cell type to what cell type) occu rs in Barren's esophagus? Squamous to columnar cell (adenocarcinoma) 28.Antinuclear antibodies (ANAs) and anti-SCL-70 antibodies are diagnostic of wh at disease? Scleroderma 29.What T cell defect is found in Graves' disease? ic suppressor T cells Defect in antigen-specif

30.A gastric mucosal erosion caused by extensive burns to the body surface is kn own as what? Curling's ulcer (Burns = hot = curling iron) VXP 31.A superficial gastric mucosal erosion causing an excessive secretion of pepsi n, due to an increase in intracranial pressure from trauma or surgical injury to the CNS, is known as what? Cushing's ulcer 32.What two antibodies are used to diagnose Hashimoto's thyroiditis? oglobulin and antimicrosomal antibodies Antithyr

33.What is the term for the copper corneal deposits found in Wilson's disease? Kayser-Fleischer rings

34.What is the triad of Meig's syndrome? cites 3. Ovarian fibroma 35.What is the triad of Plummer-Vinson syndrome? Esophageal webs 3. Iron-deficiency anemia

1. Right-sided hydrothorax 2. As 1. Atrophic glossitis 2.

36.In multiple myeloma, what are the immunoglobulin protein droplets within plas ma cells called? Russell bodies 37.Obstetric hemorrhage or shock to the anterior pituitary leads to what syndrom e? Sheehan's syndrome (approximately 90% destruction of the pituitary) 38.What is the term for a twisting of a loop of bowel leading to an obstruction? Volvulus 39.In what condition is a strawberry gallbladder seen? Cholesterolosis 40.What is the term for a palpable left supraclavicular lymph node? s node (due to metastatic gastric carcinoma) 41.What pathology is associated with Budd-Chiari syndrome? hrombus 42.Cell fragments of hemolysis are known as what? es) Virchow'

Hepatic venous t

Helmet cells (schistocyt

43.What is the name given to the red blood cell that contains a peripheral rim o f hemoglobin along with a dark central area containing hemoglobin? Target c ell 44.What are the three causes of a microvesicular fatty change in the liver? 1. Tetracycline toxicity 2. Reye's syndrome 3. Fatty liver of pregnancy 45.Abnormal, unstable RNA within a developing RBC is known as what? ic stippling Basophil

46.What is the term for an RBC with inorganic iron-containing granules? Siderocy te 47.A patient presenting with dark urine, pale stools, and itchy skin has what fo rm of jaundice? Obstructive (direct) jaundice 48.With what form of hepatitis do you see elevated levels of AST (SGOT) and GGT liver enzymes? In both alcoholic 2nd viral hepatitis, AST and ALT are elevated. SGOT > SGPT in alcoholic hepatitis, and SGPT > SGOT in viral hepatitis. 49.What region of the liver is affected during shock? Pericentral (necrotic) reg ion 50.What drug can cause amyloidosis and focal segmental glomerulosclerosis in the kidney? Heroin 51.What type of metal poisoning causes basophilic stippling? Lead 52.What drug causes a six- fold increase in schizophrenia, can impair motor acti vity, and can cause lung problems? Marijuana ... 53.With what disease do you see dust-containing macrophages within a reticulin m esh? Coal workers' pneumoconiosis

54.With what two pathologies is a honeycomb lung associated? Silicosis 55.What is Caplan's syndrome?

1. Asbestosis 2.

Rheumatoid arthritis with silicosis

56.What chemical can be potentially dangerous if you work in the aerospace indus try or in nuclear plants? Beryllium 57.What are the three causes of angiosarcoma of the liver? e 2. Thorium dioxide 3. Arsenic 58.What causes acellular fibrosis in the upper zone of the lung? s 1. Vinyl chlorid Silicosi

59.What dye is a major cause of transitional cell carcinoma of the urinary bladd er? Naphthalene 60.Interstitial fibrosis of the lower lobe is pathognomonic of what? is 61.With what is cherry red intoxication associated? isoning 62.With what disease do you see Brushfield's spots? Asbestos

Acute carbon monoxide po Down syndrome

63.What is the causative organism associated with: Acute infectious endocarditis ? Staphylococcus aureus; Aureus begins with an "a" just like acute. 64.Subacute infectious endocarditis? Streptococcus viridans

65.What percentage of a vessel is stenosed in order to cause sudden cardiac deat h? Greater than 75% of the vessel 66.What foci of fibrinoid necrosis are surrounded by lymphocytes and macrophages throughout all the layers of the heart? Aschoff's bodies of rheumatic fe ver 67.What cardiac pathology are children with pharyngeal infections more likely to develop? Rheumatic heart disease 68.Where is the embolism site for a right-sided heart lesion? The lungs Roth's s

69.What is the term for white retinal spots surrounded by hemorrhage? pots 70.In what condition are they seen? In bacterial endocarditis

71.The painless hemorrhagic areas on the palms and soles in a patient with bacte rial endocarditis are known as what? Janeway lesions 72.What pathology involves a diastolic blood pressure greater than 90 mmHg or a systolic blood pressure greater than 140 mmHg? Hypertension 73.What are the four cardiac abnormalities associated with tetralogy of Fallot? 1. Shifting of the aorta 2. Hypertrophy of the right ventricle 3. 74.Interventricular septal defect 4. Pulmonary stenosis 3X (SHIP) 75.What term applies when a single vessel receives blood from both ventricles? Truncus arteriosus

76.How do you diagnose a patient who presents with an underdeveloped right ventr icle, atrial septal defect, and no tricuspid valve? Tricuspid atresia 77.What are the three left-to-right shunts? 1. Ventricular septa] defect (VS D) 2. Atrial septal defect (ASD) 3. Patent ductus arteriosus (PDA) 78.What type of coarctation of the aorta is associated with Turner's syndrome? Preductal (infantile) type 79.Which leaflet is most commonly affected in mitral valve prolapse? r leaflet due to the long chordae tendineae Posterio

80.What disease affects medium to small arteries, occurs most commonly in men, i s associated with elevated perinuclear anti-neutrophilic cytoplasmic antibody (P -ANCA) levels to myeloperoxidase, and involves 30% of patients with hepatitis Bs antigen? Polyarteritis nodosa (PAN) Remember by P in P-ANCA and P in PAN. 81.With what disease do you see IgA deposits in small vessels of the skin and th e kidneys? Henoch-Schonlein purpura 82.What type of vasculitis presents with headache, facial pain, and sometimes lo ss of vision? Temporal arteritis (usually in women older than 80 years of age) 83.What type of vasculitis affects children, has conjunctival involvement associ ated with skin rash and lymphadenopathy, and involves 70% of persons with corona ry artery aneurysms? Kawasaki disease; Think kids, koronary, konjunctiva-Kawa saki 84.What is the difference between a true aneurysm and a false aneurysm? A true a neurysm is the bulging of an arterial wall that is intact, whereas a false aneur ysm is a rupture where a sac is formed by the tissue adjacent to the artery. 85.With what lung pathology do you see a collapsed leathery lung? spiratory distress syndrome CARDS) Acute re

86.In which region of the lung are 75% of the pulmonary infarcts seen? Lower lo bes 87.What is it called when the cervical lymph node is involved in Tb? Scrofula

88.What is the difference between Raynaud's disease and Raynaud's phenomenon? Disease occurs when there is vasospasm in a small artery without any underlying pathology, whereas a phenomenon has some underlying pathology associated with it . 89.What is the difference between a Ghon focus and a Ghon complex? A Ghon f ocus is a TB tubercle, whereas a Ghon complex is a focus with hilar lymph node i nvolvement. 90.What is the term for laminated concretions of Ca2+ and protein found in granu lomas, especially sarcoidosis? Schaumann's bodies 91.What condition is manifested by bilateral sarcoidosis of the parotid glands, submaxillary gland, and submandibular gland with posterior uveal tract involveme nt? Mikulicz's syndrome 92.In what autosomal recessive (AR) disease do you see a decrease in adenine tri phosphatase (ATPase) activity of dynein arms in cilia, also known as immotile ci lia syndrome? Kartagener's syndrome

93.What disease involves a decrease in al-antitrypsin activity, causing all the alveoli to be affected? Panacinar emphysema 94.What lung pathology involves columnar to squamous cell metaplasia and cannot be diagnosed unless the patient has 3 months of productive sputum for 2 years or more? Chronic bronchitis 95.What bronchogenic carcinoma: Is found in the peripheral aspect of the lung an d is known as the "scar" carcinoma? Adenocarcinoma 96.Has a poor prognosis because 50% of the cases have metastasized to the brain by diagnosis? Undifferentiated cell carcinoma 97.What hematoma involves lucid intervals and affects the middle meningeal arter y? Epidural hematoma 98.What renal pathology involves uniform thickening of the glomerular capillary wall, "granular" appearance under the microscope, and effacement of foot process es? Membranous glomerulonephritis (MGN) 99.What syndrome has massive proteinuria, hypoalbuminemia, hyperlipidemia, and a nasarca as its components? Nephrotic syndrome 100.What renal pathology is associated with an alteration of the basement membra ne and mesangial cell proliferation, along with a tram-track appearance under th e microscope? Membranoproliferative glomerulonephritis (MPGN) 101.What is the triad of renal cell carcinoma? 1. Hematuria 2. Costovertebral p ain 3. A palpable mass 102.What is the most common type of kidney stone? Calcium oxalate Pyelonep

103.In what condition do you see "dimpling" on the kidney's surface? hritis

104.What are the four most common metastatic sites for renal cell carcinoma? 1. Lung 2. Liver 3. Brain 4. Bone 105.What is the causative organism involved in squamous cell carcinoma of the bl adder? Schistosomiasis haematobia 106.Is cigarette smoking associated with transitional cell carcinoma of the blad der? Yes. It is also a cause of cancers of the lung, esophagus, ureter, and k idney, just to name a few. 107.What type of peptic ulcers is associated with elevated gastric secretions an d blood group O and responds well to cimetidine? Duodenal ulcer, whereas gastric ulcers are associated with blood group A and low gastric secretions 108.What GI pathology can be caused by a patient taking clindamycin or lincomyci n or by Clostridium diffcile, ischemia, Staphylococcus, Shigella, or Candida inf ections? Pseudomembranous colitis 109.How is Pseudomembranous colitis treated? With vancomycin or metronidazole

110.What regions of the GI tract are the most common sites of peptic ulcers? The lesser curvature of the stomach and the first part of the duodenum 111.What three criteria allow you to differentiate an ulcer from an erosion or c

arcinoma? cosa

1. Less than 3 cm 2. Clean base 3. Level with the surrounding mu

112.What is the watershed area of the GI tract? It is the most common site of is chemic bowel disease (splenic flexure of large bowel). 113.What is the term for hypoperfusion of an area involving only the inner layer s? Mural infarct 114.Failure of neural crest cells to migrate to the rectum and sigmoid colon is known as what? Hirschsprung's disease (congenital megacolon) 115.What is the term for the irregular linear lacerations in the long axis of th e esophagus seen in chronic alcoholics? Mallory-Weiss tears 116.What is the condition in which progressive dysphagia and regurgitation occur owing to failure of the lower esophageal sphincter to relax when swallowing is initiated? Achalasia (can also be seen in Chagas' disease) 117.With what autosomal dominant disease do you see polyps in the GI tract that can be large and pedunculated, along with melanin pigmentation of the oral mucos a, lips, and palms? Peutz-Jeghers syndrome (rarely predisposes to colon canc er) 118.What is the most common hemolytic anemia that has an elevated reticulocyte c ount and a positive result on a Coombs' test? Autoimmune/immune hemolytic anem ia 119.What is the first sign of megaloblasdc anemia on a blood smear? mented polymorphonucleocytes (more than five lobes) Hyperseg

120.With what autosomal dominant disease do you see osteomas in the mandible and maxilla, epidermoid cysts, and edematous polyps in the GI tract? Gardner' s syndrome 121.In what autosomal recessive disease do you see a decrease in ceruloplasmin, micronodular cirrhosis, and Kayser-Fleischer rings? Wilson's disease 122.What is the term for the smooth, round fragments of nuclear chromatin seen i n RBCs? Howell-Jolly bodies 123.What form of hepatitis involves Kupffer's cell hyperplasia, ballooning hepat ocytes, councilman bodies, and an increase in ALT? Viral hepatitis 124.What X-linked recessive disease involves a decrease in hypoxanthine guanine phosphoribosyl transferase (HGPRT), mental retardation, self-mutilation, choreoa thetosis, and spasticity and an increase in uricemia? Lesch-Nyhan syndrome 125.In what X-linked recessive disease is there a decrease in the hexose monopho sphate (HMP) shunt, along with Heinz body formation? G-6-PD deficiency (gluco se-6-phosphate dehydrogenase deficiency) 126.What illegal drug can cause rhabdomyolysis, myocardial infarction, cerebral infarct, and lethal cardiac arrhythmias? Cocaine 127.What type of metal poisoning causes mental retardation, somnolence, convulsi ons, and encephalopathy? Lead 128.What type of acute metal poisoning involves stomach and colon erosion and ac ute tubular necrosis? Mercury

129.What enzyme level increases in hours and falls 24 to 48 hours after a myocar dial infarction? Creatinine phospholanase (CPK) 130.What form of infectious endocarditis is most likely to metastasize? Acute in fectious endocarditis 131.What is the terminology for the secondary disease of benign hypertension in the kidney? Benign nephrosclerosis 132.What is the term for the appearance of the kidney in malignant hypertension (it has petechiae on its surface)? Flea-bitten kidney (can also be seen in pyelonephritis) 133.What is the term for fibrinoid necrosis of the arterioles in the kidney seco ndary to malignant hypertension? Onion skinning 134.What are the three most common sites for left-sided heart embolisms to metas tasize? 1. Brain 2. Spleen 3. Kidney 135.What enzyme levels increase in 6 to 8 hours, peak in 24 to 48 hours, and fal l 4 to 8 days after an MI? SGOT 136.What are the four right-to-left shunts? 1. Tricuspid atresia 2. Truncus arteriosus 3. Transposition of the great vessels 4. Tetralogy of Fallot; All of the Ts 137.What form of angina pectoris is classically brought on by exercise or by an elevated heart rate? Stable angina (S-T depressions are seen on EKG.) 138.What form of angina pectoris is the most serious and is referred to as "prei nfarction angina"? Unstable angina 139.What marker increases in 12 hours, peaks in 48 to 72 hours, and falls in 1 t o 2 weeks after an MI? Lactate dehydrogenase (LDH)-1 > 2 140.What form of angina pectoris is caused by coronary artery vasospasms? Prinzmetal's angina (variant). S-T elevations are seen on EKG. 141.What is the condition that involves focal accumulations of basophilic, mutin ous extracellular substance in the media of the aorta? Cystic medial necrosis 142.When do you see an elevated blood pressure in the upper extremities with hyp otension in the lower extremities? Coarctation of the aorta (postductal), a dult type 143.What is the characteristic radiographic finding in postductal coarctation of the aorta? Notching of the ribs due to dilated internal mammary arteries 144.What test uses p24 protein when diagnosing HIV? d immunosorbent assay test) ELISA test (enzyme-linke

145.What type of vasculitis occurs in male heavy smokers, typically younger than 35 years of age, that can cause gangrene in the lower extremities? Buerger' s disease 146.Low to absent levels of pulmonary surfactant along with hyaline membrane for mation is the hallmark of what? NRDS (neonatal respiratory distress syndrome) 147.What form of emphysema affects the upper lobes, increases in smokers, and ha

s carbon deposits distal to the affected areas? Centriacinar emphysema 148.Is right ventricular hypertrophy (RVH) in utero a sign of coaretation of the aorta? Yes, if you are referring to preductal or infantile coarctation of the a orta; congestive heart failure is also a sign of the infantile form. 149.What tumor involves an increase in 5-hydroxyindoleacetic acid; is associated with skin flushing, cramps, diarrhea, and nausea; and affects the valves on the right side of the heart along with the endocardium? Carcinoid tumor of the h eart 150.What is the consolidation around the small bronchi known as? neumonia Bronchop

151.What disease usually occurs in men in their 40s who present with persistent pneumonia, chronic sinusitis, renal disease, and mucosal alterations, and show a n increase in cytoplasmic antineutrophilic cytoplasmic antibodies (C-ANCAs)? Wegener's granulomatosis 152.What disease involves a weakened pulse and hypotension in the upper extremit ies, visual problems, and dizzy spells and occurs in Asian females who are 15 to 45 years of age? Takayasu's arteritis (pulseless disease) 153.What bronchogenic carcinoma occurs most commonly in men and smokers; is foun d centrally; and can have eetopic production of adrenocorticotrophic hormone (AC TH) or antidiuretic hormone (ADH)? Small-cell carcinoma (oat cell) 154.What bronchogenic carcinoma is commonly associated with Addison's disease? Squamous cell carcinoma 155.What disease, diagnosed by exclusion, involves bilateral hilar lymphadenopat hy and noncaseating granulomas in many organs and is most commonly seen in black women? Sarcoidosis 157.What causes a nutmeg liver? Right-sided heart failure 158.What bronchogenic carcinoma is associated with an elevated level of Cat+, in volves keratin pearls, occurs in men more than women, is associated with smoking , occurs in the major bronchi, and is seen in the central areas of the lung? Squamous cell carcinoma 159.What GI pathology involves a loss of villi and a decrease in the absorptive area due to gluten sensitivity? Celiac disease 160.What tumor spreads via the lymphatic system into the peritoneum, rectal shel f, and both ovaries, secondary to metastatic gastric carcinoma? Krukenberg's tum or 161.What is necrosis and distortion of liver architecture with nodular regenerat ion and fibrosis known as? Cirrhosis 162.What disease is due to increased iron deposits causing congestive heart fail ure (CHF), bronze diabetes, and micronodular cirrhosis? Hemochromatosis 163.What autosomal dominant syndrome involves 1000 or more edematous polyps, mos t commonly affects the colorectal area, and is associated with chromosome 5q21? Familial poly posis coli 164.What autosomal dominant disease involves hyperkeratosis of the palms and sol es in association with esophageal carcinoma? Tylosis

165.What primary carcinoma is associated with signet ring cells? carcinoma

Gastric

166.What liver pathology involves an increase in AST, GGT, Mallory bodies, fatty change, and micronodular cirrhosis? Chronic alcoholism 167.What is the term for a reversible change in which one adult cell type is rep laced by another adult cell type? Metaplasia 168.What is a decrease in the size of cells because of the loss of structural co mponents known as? Atrophy 169.What hemoglobin-derived pigment contains iron? Hemosiderin

170.What brown-black pigment is derived from the oxidation of tyrosine? Melanin 171.What form of necrosis is seen in the lower extremity or the bowel due to vas cular occlusion? Gangrenous necrosis 172.What is the term for depositions of Ca2+ in normal tissue due to hypercalcem ia? Metastatic calcifications 173.What is the most common type of necrosis? Coagulative necrosis Hyperplasia

174.What is the increase in the number of cells known as?

175.What form of necrosis is caused by the actions of lipases on adipose tissue, seen in acute hemorrhagic pancreatitis? Fat necrosis 176.What is the term for cells that have undergone proliferation and atypical cy tologic alterations involving all sizes, shapes, and orientations? Dysplasi a 177.What pigment, called the "aging pigment," is due to free radical injury and lipid peroxidation and is seen in brown atrophy? Lipofuscin 178.The increase in the size of cells due to synthesis, causing an increase in t he size of the organ, is known as what? Hypertrophy 179.What is the term for a collection of epithelioid cells surrounded by a rim o f lymphocytes? Granuloma 180.What form of necrosis is caused by immune-mediated vascular damage? Fibrinoi d necrosis 181.What is the term for excess amounts of granulation tissue that can block reepithelialization and wound healing? Proud flesh 182.In what rare autosomal recessive disorder do you see neutropenia, defective degranulation, and delayed microbial killing due to a problem in chemotaxis and migration? Chediak-Higashi syndrome 183.What is the main cell type of chronic inflammation? Macrophages (from blood monocytes) 184.What are the two main components of granulation tissue? . Neovascularization 1. Fibroblasts 2

185.What causes platelets to bind to the collagen of the basement membrane?

von Willebrand's factor (factor VIII) 186.What is the term for ischemia to the CNS causing dissolution of the tissue d ue to the actions of hydrolytic enzymes? Liquefactive necrosis 187.What is a sudden loss of oxygen to tissue, causing death of the cells becaus e of a cessation of blood flowing to a particular area, known as? Coagulat ive necrosis (most common in heart and kidneys) 188.What is the term for depositions of Ca2+ in nonviable or dying tissue, when the calcium level in the blood is normal? Dystrophic calcifications 189.What type of necrosis is seen as a part of granulomatous inflammation? Gaseous necrosis 190.What are the three causes of transudate? 1. CHF 2. Cirrhosis 3. Nephrosis

191.What is the type of healing that occurs in a clean surgical incision? Primary intention 192.What protein causes clot retraction? Thrombosthenin

193.What protein acts as a binding factor in wound healing and embryogenesis? Fibronectin 194.What IgE-mediated cell secretes major basic protein and has elevated levels in the blood during asthma and parasitic infections? Eosinophils 195.What are the three platelet aggregating factors? (ADP) 2. Prostaglandin 3. Thromboxane A2 (TXA2) 1. Adenosine diphosphate

196.What is the triad of fat embolism? 1. Petechiae 2. Hyperactive mental statu s 3. Occurs within 24 to 48 hours of the initial insult (e.g., long bone fractur e) 197.An intense inflammatory reaction, an increase in the amounts of granulation tissue and wound contraction by myofibroblasts are the characteristics of what? Healing by secondary intention 198.What are the two most common origins of pulmonary embolism? 1. Deep veins of the legs 2. The prostatic plexus of veins in the pelvis 199.What is a localized area of necrosis caused by circulatory insufficiency kno wn as? Infarction 200.What is the term for an abnormal amount of collagen type III that produces a large bulging scar, seen primarily in blacks? Keloid 201.What component of the basement membrane binds to collagen type IV and hepari n sulfate and is a cell surface receptor? Laminin 202.What is the term for an excessive production of collagen that flattens out a nd does not extend beyond the site of the injury? Hypertrophic scar 203.What are the three Bs of adult polycystic kidneys? 1. Big 2. Bilateral 3. B erry aneurysm 204.What pathway in the coagulation cascade is activated after tissue injury? Extrinsic pathway

205.What is an intravascular mass that is carried from its point of origin to a distant site known as? Embolism 206.In what disease do you see horseshoe kidneys, rockerbottom feet, low-set ear s, micrognathia, and mental retardation? Edward's syndrome (trisomy 18) 207.What genetic disease has an increased risk for developing carcinoma of the b reast? Klinefelter's syndrome 208.What is the predominant cell type of humoral immunity? 209.What protein causes fibrinolysis? Plasmin B lymphocytes

210.What factors in the coagulation cascade need Ca2+ to be activated? Factors II and X 211.What tumor comprises 40% of all testicular tumors in children? Teratoma

212.What most common germ cell tumor can be detected by elevated hCG levels and metastasizes to the bone, lung, and liver? Choriocarcinoma 213.What pathway in the coagulation cascade is activated by making contact with foreign substances? Intrinsic pathway 214.What protein is a common activator of the coagulation, 6brinolytic, and infl ammatory systems? Hageman factor (XII) 215.What factors in the coagulation cascade need factor Ila (activated) to becom e activated? Factors V and VIII 216.What lobe of the brain is most commonly affected by herpes virus? lobe Temporal

217.What disease involves microcephaly, mental retardation, cleft lip or palate, and dextrocardia? Patau's syndrome (trisomy 13) 218.What disease, caused by decompression sickness, leads to multiple foci of is chemic necrosis that affect the head of the femur, humerus, and tibia? Caisson' s disease 219.What type of hemostasis occurs in an intravascular space and consists of fib rin, platelets, and red and white blood cells? Thrombus 220.What disease involves mental retardation, flat face, muscle hypotonia, and a "double-bubble" sign on an x-ray and poses an increased risk of 221.Alzheimer's disease and acute lymphocytic leukemia (ALL)? Down syndrome (t risomy 21) 222.What are the three components of amyloid? protein 3. Glycosaminoglycans 1. Fibrillary protein 2. Amyloid Type I hypersensitivity

223.What type of hypersensitivity is mediated by IgE?

224.What type of hypersensitivity involves the antigen-antibody complex and acti vates complement, leading to tissue injury? Type III hypersensitivity 225.What type of hypersensitivity involves serum sickness, SLE, Arthus reaction, and acute glomerulonephritis? Type III hypersensitivity 226.What test uses gp120 protein when diagnosing HIV? Western blot test

227.What disease involves a patient with dry eyes and dry mouth; is associated w ith other collagen vascular diseases and B cell dysfunction; occurs in women mor e than men; and poses an increased risk of the patient developing a high-grade B cell lymphoma? Sjogren's disease 228.What is the most common fungal infection in HIV? 229.What is the most common infectious agent in HIV? Candida Pneumocystis carinii 1. CMV retinitis

230.What are the two most common viral infections in HIV? 2. HSV-2

231.What is the most common opportunistic infection of the CNS in HIV? Toxoplas mosis 232.What is the term for a hospital-acquired infection? Nosocomial infection 233.What are the four DNA oncogenic viruses? 1. HPV (human papilloma virus) 2 . EBV (Epstein-Barr virus) 3. Hepatitis B 4. Kaposi's sarcoma- HSI/? 234.What disease is X-linked recessive, presents with eczema thrombocytopenia an d an increased chance of developing recurrent infections, involves a decrease in serum IgM and in the T cell-dependent paracortical areas of the lymph nodes, an d means that the pa Wiskott-Aldrich syndrome 235.What is the most common pathway for carcinomas to spread? Via lymphatics

236.With what are the following microscopic changes associated: loss of polarity , anaplasia, pleomorphism, discohesiveness, increase in the nuclear:cytoplasmic ratio, hyperchromasia, and increase in the rate of mitosis? Malignancy 237.What is the most common pathway for sarcoma to spread? hway 238.What is the most common complement deficiency? Hematogenous pat

C2 deficiency Erythema nodosum

239.What type of erythema do you see in: Ulcerative colitis? Rheumatic fever? Erythema marginatum Stevens-Johnson syndrome? Erythema multiforme

240.What is the triad of meningitis? 1. Fever 2. Headache 3. Stiff neck (Naus ea, photophobia, and irritability are also commonly seen in patients with mening itis.) 241.What carcinoma in the thyroid involves stromal amyloidosis and an excessive release of calcitonin? Medullary carcinoma 242.What is the name for an adenoma in the adrenal glands that causes elevated l evels of aldosterone? Conn's syndrome-page230 243.What does prepubertal hypersecretion of growth hormone lead to? m 244.What is an ACTH-secreting tumor in the pituitary gland known as? s disease Gigantis Cushing'

245.What is a beta cell tumor that secretes an excess of gastrin, which causes m ultiple peptic ulcers in aberrant locations, known as? Zollinger-Ellison syndro me

246.What type of pancreatic tumor has the following common signs: hypoglycemia, hunger, sweating, tremors, seizures, and coma? Insulinoma 247.The most common causes of osteomyelitis: Overall? Staphylococcus aureus In neonates? Streptococcus agalactiae In patients with sickle cell disease? Staphylococcus aureu.s (but they are mor e prone to developing salmonella infections) In drug addicts? P.seudomonas 248.What renal disease in diabetic patients is seen as a halo of capillaries aro und the mesangial nodules? Kimmelstiel-Wilson disease 249.With what pancreatic tumor do you see watery diarrhea, hypokalemia, and achl orhydria? Vasoactive intestinal peptide (VIP) tumor of the pancreas 250.What are the four most common causes of femoral head necrosis? ids 2. Alcohol 3. Scuba diving 4. Sickle cell anemia 1. Stero

251.What bone pathology is associated with teenagers, occurrence in men more tha n women, hematogenous spread to the lungs, and Codman's triangle on an x-ray? Osteosarcoma 252.What is the collapse of the vertebral body due to Tb known as? isease Pott's d

253.What pathology is associated with cartilage formation in the bone of the jaw , shoulder, and pelvic girdle and presents in middle age? Chondrosarcoma 254.What disease is found in persons in the 20- to 40-year-old age range; affect s women more than men; occurs at the epiphysis of the knee; and is seen as a "so ap bubble" appearance on an x-ray? Giant cell tumor of the bone 255.What disease is seen in the first or second generations; is associated with chromosome 11,22; affects men more than women; and has pseudorosettes and an oni on- skin layering formation? Ewing's sarcoma 256.What joint is affected causing Heberden's nodes in osteoarthritis? Distal i nterphalangeal (DIP) joint (Bouchard's nodes are in the proximal interphalangeal [PIP] joint) 257.What vitamin D deficiency leads to a softening of the bone causing demineral ization? Osteomalacia 258.Podagra, tophi in the ear, and polymorphonucleocytes (PMNs) with monosodium urate crystals are associated with what pathology? Gout 259.With what pathology are bamboo spine on an x-ray and the haplotype HLA-B27 a ssociated? Ankylosis spondylitis 260.With what pathology is deposition of calcium pyrophosphate in patients older than 50 years of age associated? Pseudogout 261.What childhood pathology involves anterior bowing of the tibia, epiphyseal e nlargements, and costochondral widening with the endochondral bones being affect ed? Rickets 262.What is the triad of Reiter's syndrome? nctivitis 3. Nongonococcal urethritis 1. Peripheral arthritis 2. Conju

263.What are the five chemotactic mediators? a, 5. N-formyl-methionine 264.What are the three opsonins? nding protein

1. LB4, 2. IL-8, 3. C5a, 4. TNF-

1. C3b 2. Fc region of IgG 3. Mannose-bi

265.What enzyme is lacking in the autosomal recessive type of severe combined im munodeficiency? Adenosine deaminase 266.What two components of the complement cascade are anaphylatoxins? . C5a (causes mast cells to secrete histamine) 267.What causes vasodilatation, pain, and smooth muscle contraction? ins 1. C3a 2 Bradykin

268.What is the most potent vasoconstrictor and can cause platelet aggregation? TXA2 269.What prostaglandin is associated with vasodilatation (edema) iand nhibits pl atelet aggregation? Prostaglandin I2, (PGI2) 270.What prostaglandin is associated with vasodilatation, fever, and pain? PGE2 271.What are the three characteristics in Virchow's triad of thrombosis? 1. Injury to the endothelium 2. Change in laminar flow 3. Hypercoagulation 272.What is collagen and fibrin surrounded by macrophages in rheumatic fever kno wn as? Aschoff bodies 273.What leukotriene is a major chemotactic factor that causes WBCs to adhere to the endothelium? LB4 274.What two substances, produced by the body, cause fever? (IL-1) 2. Tumor necrosis factor (TNF) 1. Interleukin 1

275.What three leukotrienes are associated with bronchospasms and an increase in vessel permeability and vasoconstriction? LC4, LD4, and LE4, 276.What substance, derived from IgE-sensitized basophils, causes increased vasc ular permeability, leukocyte adhesion, chemotaxis, and aggregation along with be ing one of the most potent platelet stimulators? Platelet-activating fact or (PAF) 277.What is the most common organ involved in amyloidosis? Kidney

278.What is the term that describes when treatment for one disease leads to anot her disease presenting itself? Iatrogenic infection 279.What disease involves a failure of the third and fourth pharyngeal pouches t o develop, with a lack of T cell immunity causing a poor defense response to cer tain fungal and viral infections and tetany? DiGeorge's syndrome 280.What disease involves a lack of both T cell-mediated and Immoral immune resp onses that can be either X-linked or autosomal recessive? Severe combined immunodeficiency 281.What pathology involves excessive fibrosis throughout the body via increased fibro- blast activity, occurs in women more than men, and is most commonly seen in the third to the fifth decade? Scleroderma

282.What type of hypersensitivity involves autoimmune hemolytic anemia, erythrob lastosis fetalis, Goodpasture's syndrome, parasitic killing, Graves' disease, an d myasthenia gravis? Type II hypersensitivity 283.What type of hypersensitivity involves systemic anaphylaxis and skin and foo d allergies? Type I hypersensitivity 284.What joints in the hand are most commonly affected by rheumatoid arthritis? Proximal interphalangeal and metacarpal phalangeal joints 285.What disease is seen in the 20- to 40-year-old age group, is more prevalent in women than men, involves diarrhea with or without bloody stools, starts in th e rectum and ascends without skipping areas, includes pseudopolyps, and has a th ickness of the bowe Ulcerative colitis 286.What disease arises from the adrenal medulla, displaces and crosses the midl ine, metastasizes early, is the most common solid tumor, and is seen in the 2- t o 4-year-old age group? Neuroblastoma 287.In which disease do you find a decrease in the bone density and thickness of the cortex, occurring most commonly in postmenopausal women but can be induced by steroids, old age, or idiopathic causes? Osteoporosis 288.What is cortical hypersecretion of the adrenal gland known as? s syndrome 289.What does postpubertal hypersecretion of growth hormone cause? ly Cushing' Acromega

290.What tumor is seen in the 2- to 4-year-old age group; does not cross the mid line; has immature glomeruli, tubules, and stroma; and metastasizes late to the lungs? Wilms' tumor 291.What GI pathology is associated with the following: a positive string sign, an increase in the number of bloody stools, RLQ pain, skip lesions, terminal ile um most commonly affected, occurrence in women more than men, and an increased t hickness of the bow Crohn's disease 292.What is associated with deposition of an extracellular amorphous substance i n the blood vessel walls and connective tissue and green birefringence under pol arized light? Amvloidosis 293.What disease is characterized by a mosaic pattern of bone marrow replacement ; involves high-output cardiac failure; and occurs in women more than men and in people older than 40 years of age? Paget's disease of the bone 294.What type of hypersensitivity involves a Tb test, viral infections, graft re jections, and tumor-associated antigen? Type IV hypersensitivity 295.What form of hypersensitivity involves T cell-mediated cytotoxicity? Type IV hypersensitivity 296.What disease has the following characteristics: occurrence in women more tha n men; involvement of the second or third generation; positive ANAs; joint pain; skin rash in a malar distribution; diffuse proliferate GN; Libman- Sacks endoca rditis; and neurolo SLE : MD SOAP N HAIR (malar rash; discoid lesion; serolo gic; oligoarthritis; ANA positive; pleuritis/pericarditis; neurologic complicati ons; hematologic [leukopenia/thrombocytopenia]; anticardiolipin antibodies; 3 & immunologic; and renal)

297.What form of hypersensitivity includes IgG- or IgM-activating phagocytosis, complement, or antibody-dependent cell-mediated cytotoxicity (ADCC)? Type II hypersensitivity 298.What X-linked recessive disease involves failure of maturation of pre-B cell s with no surface immunoglobulins, leading to recurrent pyogenic infections in i nfants? X-linked agammaglobulinemia 299.What syndrome comprises small-cell carcinoma of the lung and myasthenia grav is? Lambert-Eaton syndrome 300.What pathologic process and pattern of involvement in the brain consist of n ecrosis, fibrosis, edema, and gliosis going from the center out? Abscess of the brain on a CT scan 301.What is the most common site of a contrecoup contusion? of the frontal lobe Orbital surface

302.What artery is the most common site of infarctions in the cerebral circulati on? Middle cerebral artery 303.What is the most common cause of a cerebral infarct? arteriosclerosis Atherosclerosis/

304.What event involves the following precipitating factors: hypertension, diabe tes, aneurysms, atherosclerosis, and occurrence in blacks more than in whites? Intracerebral hemorrhage 305.What artery is affected in a subarachnoid hematoma? Middle cerebral artery 306.What is the most common cause of meningitis or group B streptococci What is the most common cause of meningitis in: occus pneumoniae What is the most common cause of meningitis in: lus influenzae What is the most common cause of meningitis in: itidis What is the most common cause of meningitis in: lus influenzae in: Neonates? Escherichia coli Streptoc

Immunized children?

Non-immunized children? Haemophi Young adults? Neisseria mening

The base of the brain? Haemophi Phenylke

307.What AR disease involves a lack of phenylalanine hydroxylase? tonuria (PKU)

308.What type of head trauma is characteristic of a fractured temporal or pariet al bone, lucid intervals, headache, and confusion? Epidural hematoma 309.What disease is seen in children younger than 5 years of age; X-linked reces sive, cardiac myopathies; calf pseudohypertrophy; lordosis; protuberant bellies; an increase then a decrease in CPK; and death in the second generation of life? Duchenne's muscular dystrophy 310.What breast pathology involves malignant cells with "halos" invading the epi dermis of the skin? Paget's disease of the breast 311.What tumor comprises 40% of all testicular tumors in children? Teratoma

312.What disease has the following characteristics: has autoantibodies to IgG; o ccurs in women more than men; and includes exophthalmos, pretibial myxedema, ner

vousness, heart palpitations, and fatigue?

Graves' disease

313.What germ cell tumor is seen in the 15- to 35-year-old age group, peaks when the person is 35 years of age, and is a bulky mass that spreads via the lymphat ic system? Seminoma 314.What sex cell tumor causes precocious puberty, masculinization, gynecomastia in adults, and crystalloids of Reinke? Leydig cell tumor 315.What thyroid pathology causes dwarfism, retarded bone maturation, myxedema, mental retardation, and decreased T4 with increased thyroid-stimulating hormone (TSH) levels? Cretinism 316.What are the following risk factors characteristic of: late menopause, early menarche, obesity, nulliparity, excessive estrogen, genetic factor p53, and brc -abl? Breast cancer 317.What adenocarcinoma presents with elevated levels of acid phosphatase, dihyd rotestosterone, prostate-specific antigen (PSA), and bone pain? Prostatic carcin oma 318.What myopathy, due to autoantibodies to ACh receptors, can present with thym ic abnormalities, red cell aplasia, and muscle weakness? Myasthenia gravi s 319.What two factors of the complement cascade are deficient in the person with SLE? C2 and C4 320.What aspect of the complement cascade is defective if a patient constantly p resents with recurrent infections with Neisseria gonorrhoeae or N. meningitidis? Membrane attack complex-MAC (C5 to C9) 321.What disease involves "cold" skin abscesses due to a defect in neutrophil ch emotaxis and a serum IgE level higher than 2000? Job's syndrome 322.What X-linked recessive disease, deficient in NADPH oxidase, presents with c atalase- positive infections? Chronic granulomatous disease (30% are AR) 323.When does fetal IgM first appear in the fetal circulation? Third trimester (6 to 9 months) 324.Hereditary angioneurotic edema (AD) presents with local edema in which organ s? GI, skin, respiratory tract 325.What enzyme deficiency causes increased capillary permeability due to a rele ase of vasoactive peptides? C1 esterase inhibitor (C1INH) 326.What AR disease contains the most common neutrophil defect? Myeloperoxidase deficiency 327.How can a deficiency in adenosine deaminase be a bone marrow suppressor? It causes a buildup of dATP, which inhibits ribonucleotide reductase and leads t o a decrease in deoxynucleoside triphosphate (a precursor of DNA), resulting in overall bone marrow suppression. 328.What disease involves an adenosine deaminase deficiency, B and T cell defici ency, and defective IL-2 receptors? Severe combined immunodeficiency (SCID) 329.What AR disease presents with B and T cell deficiencies, lymphopenia, an IgA deficiency, cerebellar problems, and spider angiomas? Ataxia-telangiectasia

330.What immunoglobulins are present on the surface of mature B cells? IgM and IgD 331.What is the only immunoglobulin found in the fetal circulation in the second trimester? Maternal IgG 332.How does maternal immunoglobulin cross the placenta to get into fetal circul ation? Fc receptor on the heavy chain of the immunoglobulin via active transpor t 333.What disease involves a deficiency in IgM; elevated IgA; normal IgG; and rec urrent pyogenic infections, especially from pneumococci; and thrombocytopenia? Wiskott-Aldrich syndrome 334.What syndrome in children involves hypofunction of the adrenal glands due to bilateral hemorrhagic infarctions, which are most commonly associated with meni ngococcemia? Waterhouse- Friderichsen syndrome 335.What thyroid carcinoma presents with psammoma bodies? oma of die thyroid Papillary carcin

336.In what disease do you see a large, hard, fibrous proliferation of the conne ctive tissue of the thyroid? Riedel's thyroiditis 337.What thyroid pathology presents with a "cold," solitary, discrete nodule? Adenoma of the thyroid 338.What cell type involves Immoral immunity? B lymphocytes

339.What disease involves a goiter, hypothyroidism, and autoimmunity due to ag-s pecific suppressor T cells and occurs in women more commonly than in men? Hashimoto's thyroiditis 340.How is gonadal sex determined? 341.How is ductal sex determined? 342.How is phenotypic sex determined? The gonads' histologic characteristics Presence of mullerian or wolffian ducts Appearance of external genitalia

343.What disease presents with severe mental retardation, VSD, asymmetric face, microcephaly, and chromosome 5p deletion? Cri du chat 344.What is primary hyperparathyroidism most commonly due to? ma (80%) Chief cell adeno

345.What are low levels of Ca2+ and P04 along with neuromuscular irritability si gns of? Hypoparathyroidism 346.What pathology has the following signs: severe headache, palpitations with o r without tachycardia, diaphoresis, anxiety, nervousness, and hypertensive episo des? Pheochromocytoma 347.What pathology has the following signs: elevated levels of Cat+, cardiac arr hythmias, bone resorption, kidney stones, and metastatic calcifications? Primary hyperparathyroidism 348.What adrenal pathology presents with hypotension, hyponatremia, hypoglycemia , and hyperkalemia along with skin pigmentation? Addison's disease

349.What disease, with familial mental retardation, presents with large everted ears and macroorchidism? Fragile X syndrome 350.What ovarian pathology involves psammoma bodies? Serocystadenocarcinoma

351.What cystic swelling of the chorionic villi is the most common precursor of choriocarcinoma? Hydatidiform mole 352.What estrogen- or progesterone-producing tumor is associated with Call-Exner bodies? Granulosa cell tumor 353.What ovarian pathology is associated with Turner's syndrome or malformation of the genitals and comprises 50% of all malignant germ cell tumors? Dysgermi noma 354.What ovarian pathology involves ectoderm, endoderm, and mesoderm in a histol ogic section and is most commonly seen in the early reproductive years? Dermoid cyst (teratoma) 355.What AR disease involves a deficiency in glucocerebrosidase,a huge spleen, a nd engorged phagocytic cells and is associated with chromosome 1? Gaucher' s disease 356.What disease, with an abnormality in collagen type I, presents with "blue sc lera"? Osteogenesis imperfecta 357.What small, subareolar, solitary tumor that affects the lactiferous ducts pr esents with bloody discharge from the nipple? Intraductal papilloma 358.What cell consists of a binucleate giant cell with eosinophilic inclusions? Reed-Sternberg cells 359.What disease involves bilateral, enlarged, pale ovaries and presents with in fertility, hirsutism, obesity, secondary amenorrhea with elevated levels of LH, testosterone, and low levels of FSH? Polycystic ovaries 360.What benign tumor of the breast, seen in the young, is well demarcated and h as a very low risk of cancer? Fibroadenoma 361.What breast pathology commonly occurs bilaterally in the upper outer quadran ts, and includes microcalcifications, hypertrophy of the ducts, apocrine metapla sia, sclerosing adenoma, and blue-domed cysts? Fibrocystic change of the breast 362.What AR disease involves a deficiency in tyrosinase, poses an increased risk of developing basal cell or squamous cell carcinoma, and is associated with chr omosome llp? Albinism 363.What is the most common fatal recessive disease in whites? Cystic fibrosis 364.What AR disease has a deficiency in homogentisic oxidase that causes brittle , fibrillated articular cartilage, blue-black pigmentation of collagen, and urin e that turns black upon standing? Alkaptonuria 365.What AR disease involves a decreased amount of sphingomyelinase, massive org anomegaly, zebra bodies, and foamy histiocytes microscopically and is associated with chromosome llp? Niemann-Pick disease 366.What disease has multiple schwannomas, cafe au lait spots on the skin, and L isch nodules and is associated with chromosome 17q? Neurofibromatosis I (chromosome 22q is with neurofibromatosis II and no Lisch nodules either)

367.Name the AD disease associated with chromosome 15 in which the patient has l ong extremities, lax joints, pigeon chest, and posterior mural leaflet prolapse, and is prone to developing dissecting aortic aneurysm? Marfan's syndrome 368.What AR disease involves a defect in amino acid 508 on chromosome 7, causing a defect in Cl- transportation that leads to recurrent pulmonary infections and an increase in viscid mucoid secretions along with pancreatic insufficiencies? Cystic fibrosis (Parents are usually the first to find out because the baby tast es salty.) 369.What AD disease, associated with chromosome 19, involves a defect in the LDL receptors, leading to skin and tendon xanthomas? Familial hypercholestero lemia 370.In what syndrome does the patient have angiomatosis; renal cell carcinomas; pheochromocytomas; retinal, cerebellar, medulla, or spinal cord hemangioblastoma s; and epidermal cysts? von Hippel-Lindau syndrome 371.What AD disease is associated with chromosome 4p; does not present until the person is in his or her 30s; and involves atrophy of the caudate nucleus, dilat ation of the lateral and third ventricles, and signs of extrapyramidal lesions? Huntington's disease 372.What is the most common cause of death in this disease? Suicide

373.In utero death, caused by a complete lack of an alpha chain, is known as wha t? Hydrops fetalis \ 374.What AR disease involves a substitution of valine for glutamic acid at posit ion 6 on the beta chain? Sickle cell anemia 375.What AD disease involves a defect in spectrin and leads to jaundice, splenom egaly, and cholecystitis? Hereditary spherocytosis 376.What spinal cord pathology is caused by a degeneration of the cortical spina ls, leading to weakness, fasciculations, hyperreflexia, and spasticity? Amyotrop hic lateral sclerosis (ALS)-Lou Gehrig's disease 377.What are the two reasons for megaloblastic anemia with elevated mean corpusc ular volume (MCV)? 1. Vitamin B12 deficiency 2. Folate deficiency 378.What are the five reasons for normochromic normocytic anemia with a normal M CV and an elevated reticulocyte count? 1. Autoimmune hypersplenism 2. Trauma 3. Anemia 4. Spherocytosis 5. Sickle cell anemia 379.What are the four reasons for hypochromic microcytic anemia with a low MCV? 1. Sideroblastic anemias (i.e., porphyrin and heme synthesis disorders) 2. Thala ssemia 3. Iron deficiency 4. Lead poisoning 380.What are the three causes for normochromic normocytic anemia with a normal M CV and a low reticulocyte count? 1. Marrow failure 2. Cancer 3. Leukemia 381.How is iron-deficiency anemia differentiated from the other forms of hypochr omic microcytic anemia? RDW value greater than 15 (indicative of iron-deficiency anemia) 382.What CNS tumor is seen in persons between 40 and 50 years of age; occurs in men more than women; affects the cerebral brain stem; is GFAP-positive; has a po or prognosis; and microscopically displays pseudopalisades,increased cellularity

, pleomorphism, neo

Glioblastoma multiforme

383.In what CNS tumor, arising from arachnoid cells, do you see psammoma bodies? Meningioma occurs in (women more often than in men). 384.What CNS pathology has a protracted history of seizures, occurs in the 30- t o 40-year-age range, and microscopically involves "fried egg" cells? Oligoden droglioma 385.In what disease do you see atrophy of the frontal and temporal lobes ("walnu t brain")? Pick's disease 386.What disease affects the basal ganglia and the substantia nigra, produces co gwheel rigidity, mask-like faces, and resting tremors; and reveals Lewy bodies m icroscopically? Parkinson's disease 387.What pathology has bilateral periventricular plaques, perivascular inflammat ion, and demyelination and is thought to be autoimmune or brought on by a viral infection? Multiple sclerosis 388.What type of vascular pathology, involving damage to the bridging veins that drain into the superior sagittal sinus, is due to blunt trauma and is seen most commonly in old people? Subdural hematoma 389.What pathology is associated with a bloody lumbar puncture, which is caused most commonly by a ruptured berry aneurysm that produces the worst headache that the person has ever experienced? Subarachnoid hemorrhage 390.What is the most common intramedullary spinal cord tumor? Ependymoma

391.What is the most common cause of dementia; occurs in women more than men in the 60- to 90-year-old age group; and is associated with (32 amyloid, senile pla ques, and neuroflbrillary tangles? Alzheimer's disease 392.What common cause of hypogonadism in men involves testicular atrophy, azoosp ermia, gynecomastia, and Barr bodies? Klinefelter's syndrome 393.What is the main type of cell involved in cellular immunity? cyte T lympho

394.What is the syndrome associated with women of short stature with a web neck, a low posterior hairline, streaky ovaries, and preductal coarctation of the aor ta? Turner's syndrome 395.What is the term for a person with ambiguous external genitalia who has both ovarian and testicular tissue (Note: 66% of such persons are 46 XX.)? True her maphrodite (due to an X,Y translocation) 396.What type of Ehlers-Danlos syndrome is X-linked recessive and is caused by a defect in copper metabolism, causing a problem with cross-linking collagen and elastin fibers? Type IX 397.What type of Ehlers-Danlos syndrome involves decreased activity of lysyl hyd roxylase and affects collagen types I and III the most? Type VI (AR) 398.What type of Ehlers-Danlos syndrome involves a deficiency in procollagen- Npeptidase? Type VII (AR) 399.What type of person has testicular tissue with female genitalia and both the Y chromosome and testis present? Male pseudohermaphrodite 3 & "Dude looks

like a lady!" 400.What type of person presents with male external genitalia and is 46XX owing to excess exposure to androgenic steroids during the early stage of gestation? Female pseudohermaphrodite 401.What syndrome has the following components: hypertension, proteinuria, hemat uria, azotemia, and oliguria? Nephritic syndrome 402.What type of glomerular nephritis (GN) occurs most commonly in children afte r a pharyngeal or skin infection; is immune complex mediated; and is seen as "lu mpy-bumpy" subepithelial deposits? Postinfectious glomerular nephritis 403.In what type of GN, seen in the 2- to 6-year-old age group, is there albumin uria and effacement of the visceral epithelial foot process with no deposits? Minimal change disease (lipoid nephrosis) 404.What type of GN, associated with celiac disease and dermatitis herpetiformis , has mesangial deposits of IgA, C3, properdin, IgG, and IgM? Berger's disease (IgA nephropathy) 405.What pathology has pulmonary and renal basement membrane involvement and cre scent formation and involves type II hypersensitivity? Goodpasture's syndrome 406.In what AD pathology is there a derangement of the epiphyseal cartilage grow th, causing a large skull and a normal-sized vertebral column? Achondroplasia 407.What gynecologic pathology occurs in the third and fourth decades, is the ca use of 18 to 25% of all gynecologic laparoscopic procedures, and presents with c hocolate cysts? Endometriosis 408.What form of GN is characteristically associated with crescent formation? Rapidly progressive glomerulonephritis (RPGN) 409.What type of GN has C3, IgG, C1q, C4 along with subendothelial deposits? MPGN type I (two thirds of the MPGN cases) 410.What type of skin carcinoma occurs on sun-exposed sites, has a low level of metastasis, and involves keratin pearls? Squamous cell carcinoma 411.A herniation of the brain through a defect in the skull is known as what? Fungus cerebri 412.What type of skin pathology, located on the central face and on sun-damaged skin, is a precursor to squamous cell carcinoma? Keratoacanthoma 413.True or false-Hodgkin's lymphoma affects Waldeyer's ring? in's disease affects Waldeyer's ring and the periaortic nodes False; non-Hodgk

414.What pathology is due to increased resorption or impaired synthesis of bone, resulting in decreased bone mass, and is associated with postmenopause, inactiv ity, hyperthyroidism, hyperadrenocorticism, and Ca2+ deficiency? Osteopor osis 415.What is a circumscribed, flat, nonpalpable pigmented change up to 1 cm? Macule (e.g., a freckle) 416.What is a palpable, elevated solid mass up to 0.5 cm? Papule

417.What is the most common tumor on sun-exposed sites that rarely metastasizes

but is locally aggressive and has palisade arrangements of the nuclei? Basal ce ll carcinoma 418.What is an elevated, fluid-filled cavity between skin layers up to 0.5 cm? Vesicle (e.g., poison ivy) 419.What is an elevated, fluid-filled cavity between the layers greater than 0.5 cm? Bulla 420.What melanocytic tumor has a neural filament tumor marker and vertical or ra dial growth? Malignant melanoma 421.What benign neoplasm has the total lesion above the level of the skin with a "pasted on appearance"? Seborrheic keratosis 422.What are the gray-black patches of verrucous hyperkeratosis, usually found i n the axillary folds, that can be a sign of an underlying malignancy? Acanthos is nigricans (It is commonly seen in obese patients.) 423.What is it called when the posterior cerebellar mass pushes the tonsils thro ugh the foramen magnum? Tonsillar herniation 424.What form of lymphoma is diffuse in the lymph node, has an IgM spike, slowly evolves to chronic lymphocytic leukemia (CLL), is seen in the old, and also has liver, spleen, and bone marrow involvement? Small-cell lymphoma 425.What lymphoma is a diffusely mixed, diffusely large cell that grows rapidly and consists mainly of B cells along with "null" cells? Diffuse aggressive lymph oma 426.What lymphoma forms 30% of childhood lymphomas, is linked to EBV, is associa ted with chromosome 8,14q translocation, and has a "starry sky" pattern of invol vement on a histologic section? Burldtt's lymphoma 427.What lymphoma arises from germinal follicles and is associated with proto- o ncogene bcl-2 due to translocation of chromosome 14,18? Follicular lymphoma 428.What variant of small lymphocytic lymphoma has a slow course and elevated le vels of IgM that lead to hyperviscosity syndrome and blindness? Waldenstrom's ma croglobulinemia 429.What lymphoma is seen in male adolescents, is associated with a thymic mass, and progresses to ALL? Lymphoblastic lymphoma {verify this} 430.What is the term for the condition in which the brain is pushed under the fa lx cerebri by a one-sided mass lesion? Subfalcial herniation 431.What CNS tumor arises from Rathke's pouch? Craniopharyngioma 432.What variant of Hodgkin's lymphoma occurs least frequently, is seen in peopl e younger than 35 years of age, is localized, has an excellent prognosis, and in volves mainly lymphocytes and a few Reed-Sternberg cells? Lymphocyte predo minant 433.What variant of Hodgkin's lymphoma is the most common type; involves women m ore than men; occurs in adolescence more than in old age; affects the lower cerv ical, supraclavicular, and mediastinal lymph nodes with broad bands of fibrous t issue and "lacunar" Nodular sclerosis 434.What variant of Hodgkin's lymphoma is widespread with extensive fibrosis and

necrosis, occurs in older patients, involves many Reed-Sternberg cells, and has a poor prognosis? Lymphocyte depletion 435.What variant of Hodgkin's lymphoma can be localized or widespread and has an intermediate prognosis with lymphocytes, eosinophils, plasma cells, histiocytes , and Reed-Sternberg cells? Mixed cellularity 436.What AR disease involves a deficiency in hexosaminidase A and cherry red spo ts on the retina; is seen more commonly in Jewish people; and is associated with chromosome 15q? Tay-Sachs disease 437.Name the condition described by the following: Adenomas of the thyroid, para thyroid, and adrenal cortex along with Zollinger-Ellison syndrome? Multiple endocrine neoplasia, type I (MEN I)-Wermer's syndrome 438.Pheochromocytoma, medullary carcinoma of the thyroid, and adenoma of the par athyroid? MEN Ila-Sipple's syndrome 439.What cranial nerve is most commonly affected in a schwannoma? CN VIII

440.What leukemia is associated with a chromosome 9,22 translocation; constitute s 80% of childhood leukemias; and has blasts with PAS-positive material and term inal deoxy-transferase (TdT) marker present? Acute lymphocytic leukemia (ALL) 441.What leukemia is seen in the 15- to 39-year-old age group, has blasts with m yeloperoxidase-positive granules, tends to invade tissues, and is associated wit h a poor prognosis? Acute myelocytic leukemia (AML) 442.What leukemia is seen in the 25- to 60-year-old age group and is associated with chromosome 9,22 translocation, bcr-abl oncogene, and blast crisis? Chronic myeloid leukemia (CML) 443.What leukemia is the most indolent of all leukemias; affects persons older t han 55 years of age; and is associated with trisomy 12 (Hint: 95% are B cell neo plasms.)? Chronic lymphocytic leukemia (CLL) 444.What type of GN has C3 MPGN type II deposits in irregular granular/linear fo ci and intramembranous deposits of unknown material? MPGN Type II 445.What is the term for a News benign melanocytic tumor? Nevus

1.Which muscarinic receptor uses a decrease in adenyl cyclase as its second mess enger? M2 2.What drug is used to differentiate a cholinergic crisis from myasthenia gravis ? Edrophonium 3.What drug causes a gradual loss of choline from the presynaptic nerve terminal by blocking its reuptake? Hemicholium 4.What is the only site in the body that uses Ml receptors? 5.What drug is an Ml-specific antispasmodic? Pirenzepine The stomach

6.What is the most potent neuromuscular junction Mocker (NMJB), and also has no cardiovascular side effects? Doxacurium

7.What antimuscarinic is used as an inhalant for asthma? 8.What is the antidote for organophosphate ingestion? idoxime)

Ipratropium Atropine and 2-PAM (pral

9.What is the drug of choice for atropine or tricyclic antidepressant (TCA) over dose? Physostigmine 10.What is the rate-limiting step for norepinephrine synthesis? Tyrosine hydroxy lase 11.What two enzymes are blocked by disulfiram? Aldehyde dehydrogenase and dopam ine -hydroxylase 12.What is the monoamine oxidase B (MAOB) inhibitor? Selegiline

13.What two drugs inhibit the release of neurotransmitters from storage granules ? 1. Guanethidine 2. Bretylium 14.What drug blocks intragranular uptake of norepinephrine (NE)? e 15.What two drugs, when mixed, can lead to malignant hyperthermia? nylcholine 2. Halothane (Treatment is with dantrolene.) Reserpin 1. Succi

16.In what phase of noncompetitive depolarization does no further depolarization occur, producing a desensitized block? Phase 2 17.What adrenergic receptors use inositol triphosphate (IP3) and diacylglycerol (DAG) for their second messenger system? a1-Receptors 18.What a1-agonist is used to treat paroxysmal atrial tachycardia with hypotensi on? Metaraminol (a1, 1) 19.What a1-agonist, not inactivated by catechol-O- methyl transferase (COMT), is used as a decongestant and also for treatment of paroxysmal atrial tachycardia? Phenylephrine 20.What 2-agonist is used in the prophylactic treatment of asthma? ol 21.What two 2-agonists cause myometrial relaxation? ine Salmeter

1. Ritodrine 2. Terbutal

22.A hypertensive crisis can be caused by the addition of an MAO inhibitor and w hat? Tyramine 23.What group of drugs is known as the "antihypertensive" group? a1-Antag onists Why? Because they decrease total peripheral resistance (TPR) and preload with no change in heart rate or cardiac output 24.What mixed a-antagonists are used for patients with pheochromocytoma? Phentolamine and phenoxybenzamine 25.What a2-antagonist is used to treat impotence and postural hypotension? Yohimbine 26.What drug that penetrates the blood-brain barrier is found in asthma preparat ions and used as a nasal decongestant? Ephedrine

27.What two 2-agonists are used to produce bronchodilatation? 1. Metaprotereno l 2. Albuterol 28.What -blocker is also an a-blocker? Labetalol 29.What are the four cardioselective -blockers? oprolol 4. Acebutolol (BAMA) 30.What -blocker is also a membrane stabilizer? 1. Bisoprolol 2. Atenolol 3. Met Propranolol

31.In what three areas of the body are sympathetics the predominant tone? 1. Sweat glands 2. Arterioles 3. Veins 32.What three -blockers are used in the treatment of glaucoma? 1. Propranolol 2 . Timolol 3. Carteolol 33What two -blockers decrease serum lipids? 1. Pindolol 2. Acebutolol

34.In what area of the brain can an excess of dopamine lead to psychotic symptom s? Mesocortical area 35.What area of the brain is linked to emotion and movement? m Mesolimbic syste

36.What two drugs block dopa-decarboxylase in the periphery to decrease the conv ersion of L-dopa to dopamine? 1. Carbidopa 2. Benserazide 37.What is the drug of choice for early Parkinson's disease? Selegiline

38.What antiviral agent is used in the treatment of drug-induced Parkinson's dis ease? Amantadine 39.What drug which causes Ca2+ independent release of dopamine is used to treat attention deficit hyperactive disorder (ADHD) and narcolepsy? Methylphenidate 40.What dopamine-2 (D2) agonist is used to treat neuroleptic malignant syndrome? Bromocriptine 41.What cofactor of dopa- decarboxylase decreases the efficacy of L-dopa? Vitamin B6 42.What three drugs can cause gingival hyperplasia? rine 3. Nifedipine 1. Phenytoin 2. Cyclospo

43.What antiepileptic agent has syndrome of inappropriate antidiuretic hormone ( SIADH) as a side effect? Carbamazepine 44.What drug is used for partial seizures and Lennox-Gastaut syndrome in childre n? Felbamate 45.What is the drug of choice for trigeminal neuralgia? Carbamazepine 46.What are the first signs of overdose from Phenobarbitals? axia Nystagmus and at

47.What drug, used for partial seizures, inhibits the release of glutamate and c auses rashes in 45% of patients taking it? Lamotrigine 48.What benzodiazepine is used to treat absent mal seizures? Clonazepam

49.What are the two drugs of choice for simple partial seizures? mazepine 2. Phenytoin

1. Carba

50.What is the only neuroleptic agent that does not cause hyperprolactemia? Clozapine 51.What is the drug of choice for status epilepticus? Diazepam

52.What two neuroleptic agents are associated with tardive dyslunesia? 1. Haldo l 2. Fluphenazine 53.What neuroleptic agent causes retinal deposits, hypotension, and torsades de pointes? Thioridazine 54.What neuroleptic has amoxapine as a metabolite? Loxapine

55.What is the only neuroleptic that does not cause an increase in weight or app etite? Molindone 56.What TCA causes sudden cardiac death in children? 57.What TCA is used to treat enuresis? Imipramine 58.What class of antidepressants are associated with insomnia? Serotonin select ive reuptake inhibitors (SSRIs) 59.What monoamine oxidase inhibitor (MAOI) does not cause a hypertensive crisis? Selegiline 60.What occurs if you mix an MAOI and a sympathomimetic? ion, which can lead to subarachnoid hemorrhage Severe hypertens Desipramine

61.What two TCAs are considered to be heavily sedative? 1. Amitriptyline 2. Traz odone 62.What neuroleptic agent is also considered to be an antihistamine? one Risperid

63.What TCA is used to treat obsessive-compulsive disorder and is said to cause aggressive behavior? Clomipramine 64.What neuroleptic agent causes agranulocytosis and also has no tardive dyslcin esia as a side effect? Clozapine 65.What drug decreases mood swings and is used for the manic phase of a bipolar illness? Lithium 66.What group of drugs potentiate the activity of gamma-aminobutyric acid (GABA) ? Benzodiazepines 67.What benzodiazepine is used for: Anxiety/panic attacks? 68.Absence seizures and as an anticonvulsant? Clonazepam Alprazolam

69.Alcohol withdrawal and as an anticonvulsant? Clorazepate 70.Status epilepticus, as a preoperative medication? 71.Preoperative medication? Lorazepam Diazepam

72.What benzodiazepines are activated outside of the liver? pam, and lorazepam (OTL = "outside the liver")

Oxazepam, temaze

73.What benzodiazepine antagonist is used for benzodiazepine overdose? Flumazen il 74.What are the three signs of morphine overdose? creased respiratory rate 3. Coma 75.What is the neurotransmitter (NT) at the mu receptor? 76.What is the NT at the delta receptor? 77.What is the NT at the kappa receptor? Enkephalin Dynorphin 1. Pinpoint pupils 2. De -Endorphin

78.Which type of receptor antagonist is the most clinically efficacious? Competitive antagonist 79.Which type of antagonist acts on the same receptor as the agonist that it blo cks? Pharmacologic antagonist 80.How are water-soluble drugs eliminated primarily? Via the kidneys

81.What class of pharmaceuticals are initially inactive but are then metabolized to their active products? Prodrugs 82.How are drugs that are excreted via the biliary system resorbed by the GI tra ct? Enterohepatic cycling 83.What body fluid preferentially breaks down esters? Blood

84.Which type of antagonist takes out a drug by binding to it? Chemical antagon ist 85.Which type of antagonist directly reverses the action of a drug by working on a different receptor? Physiologic antagonist 86.Quantal dose-response curves indicate what two things about a drug in a patie nt population? 1. Margin of error 2. Relative safety 87.When an agonist drug binds to its receptor, what type of mechanism does it ac tivate? Effector mechanism 88.What are the four types of signaling mechanisms? 1. Intracellular recepto rs 2. Membrane receptors 3. Enzymes 4. Intracellular effectors 89.What are the two factors that influence low oral bioavailability? -pass metabolism 2. Acid lability 90.What do the following values stand for: EDS ? f drug takers (median effective dose) 91.TD50? 92.LD50? 1. First

Effective dose for 50% o

Toxic dose for 50% of drug takers (median toxic dose) Lethal dose for 50% of drug takers (median lethal dose ) LD50 divided by

93.What is the equation for a drug's ther-a e~utic index? ED50

94.What does the FDA regulate? Efficacy and safety of drugs 95.Which phases of drug testing require an investigational new drug exemption? Phases I, II, and III 96.How many years do preclinical animal studies last? 97.How many phases of clinical testing are there? 2 to 5 years Four phases

98.Before which phase of clinical testing is a new drug application applied for? Phase IV 99.How many years does clinical testing last? 100.What antineoplastic drug is a prodrug? 4 to 5 years Flucytosine

101.What carbapenem is resistant to penase and is a partial cell wall inhibitor? Imipenem 102.What monobactam is resistant to -lactamases of some bacteria? m Aztreona

103.What are the drugs used in the triple treatment of Helicobacter pylori? Pepto-Bismol, metronidazole, and erythromycin or amoxicillin 104.What is the commonly IV opioid used in surgery? Fentanyl

105.Which IV agent has the lowest incidence of postoperative emesis and has the fastest rate of recovery? Propofol 106.What is the major pulmonary side effect of mu-activators? ession Respiratory depr

107.Which drug causes dissociative anesthesia and is used mainly in pediatric su rgery? Ketamine 108.Which short-acting benzodiazepine is often used for conscious sedation? Midazolam 109.What is the most commonly used IV opioid in cardiovascular surgery? Morphine sulfate 110.What toxicities are caused by the following agents: Occupational nitrous oxi de exposure? Anemia MethoxyHurane? Nephrotoxicity Halothane? Hepatitis, with or without necrosis 111.What is the only local anesthetic that does not cause vasodilatation? Cocaine (It is vasoconstrictive.) 112.Do opioids increase or decrease uterine smooth muscle tone? Decrease-but the y increase ureter smooth muscle tone 113.What are the two side effects of opioids to which the user will not develop tolerance? 1. Constipation 2. Miosis 114.What two classes of drugs can cause schizoid behavior? ds 2. Amphetamines 115.What is another name for prolactin inhibiting factor? 1. Clucocorticoi Dopamine

116.What is the mechanism behind tardive dyskenesia? lation

Dopamine receptor upregu Diazepam Lipid solubility

117.What sedative-hypnotic is used for alcohol withdrawal? 118.To what is inhalational anesthesia potency proportional?

119.What is the only commonly used inhalant that is not a halogenated hydrocarbo n? Nitrous oxide 120.Which sedative-hypnotic is contraindicated in patients on warfarin therapy? Chloral hydrate 121.What is the triad associated with "serotonin crisis"? thermia, and rigidity Myoclonus, hyper

122.What form of antimicrobial therapy is better to treat an immunocompromised p atient? Bactericidal 123.What drug used to treat alcoholism has a long half-life and is given orally? Naltrexone 124.What is the site of action for carbonic anhydrase inhibitors? tubule 125.What is the site where local anesthetics bind? Proximal

Inactive Na+ channels

126.Which bactericidal agents interfere with cell wall synthesis by inhibiting t ranspeptidation? Penicillins 127.What are the three -lactamase inhibitors? 1. Clavulanic acid 2. Sulbactam 3. Tazobactam 128.What sulfonamide is the drug of choice in treating: UTIs? Toxoplasmosis? Sulfadiazine and pyrimethamine Malaria prophylactically? Sulfadoxine and pyrimethamine Ophthalmic infections? Sulfacetamide Crohn's or ulcerative colitis? Sulfasalazine 129.What drug blocks dihydrofolate reductase? dihydropteroate synthase.) Sulfisoxazole

Trimethoprim (Sulfonamides block 1. Ampicillin 2. Amoxici

130.What are the two "broad-spectrum" penicillins? llin

131.Which penicillin can cause interstitial nephritis? Methicillin 132.What form of penicillin is stable in acid environments? Penicillin-V

133.What form of penicillin is used in the treatment of life-threatening illness es? Penicillin-G (benzylpenicillin) 134.What is the drug of choice in the treatment of Pneumocystis carinii pneumoni a? Sulfonamide/trimethoprim 135.What are the five penicillinase-resistant penicillins? 1. Cloxacillin 2 . Oxacillin 3. Nafcillin 4. Dicloxacillin 5. Methicillin (CONDM) 136.Which antiviral agent is teratogenic? Amantadine

137.Which broad-spectrum antibiotic inhibits the attachment of amino acyl tRNA b y binding to the 30S ribosomal subunit? Tetracycline 138.Which tetracycline is used in the treatment of SIADH? Demeclocycline

139.Which tetracycline is used when there is a decrease in renal function? Doxycycline 140.With which two tetracyclines is phototoxicity associated? . Minocycline 1. Doxycycline 2

141.Which drug inhibits peptidyl transferase enzyme and binds to the 50S ribosom al subunit? Chloramphenicol 142.What two tetracyclines have the highest plasma binding? . Minocycline 1. Doxycycline 2

143.Renal tubular acidosis, nephrosis, and amino aciduria constitute the triad o f what syndrome? Fanconi-like syndrome 144.What is the cephalosporin of choice for Pseudomonas infections? ime Ceftazid

145.Which bacteriostatic drug inhibits translocation of protein synthesis by bin ding to the 50S ribosomal subunit and is a narrow-spectrum antibiotic used for p ulmonary infections? Erythromycin Vestibular toxicity is associated with what tetracycline? Minocycline Hepatotoxicity is associated with what tetracycline? Chlortetracycline 146.Which two cephalosporins cross the blood-brain barrier? Cefaclor 1. Cefuroxime 2. 1. Cefam 1. Cefam 1. Cefam

147.What three cephalosporins are eliminated via biliary mechanisms? andole 2. Cefoperazone 3. Ceftriaxone 148.What three cephalosporins can produce disul6ram-like reactions? andole 2. Cefoperazone 3. Moxalactam 149.What three cephalosporins inhibit vitamin K-dependent factors? andole 2. Cefoperazone 3. Moxalactam

150.What three cephalosporins have good penetration against Bacteroides fragilis ? 1. Cefotetan 2. Cefoxitin 3. Ceftizoxime 151.What cephalosporin-like drug has excellent coverage against gram-positive an d gram-negative bacilli activity and is used in conjunction with the enzyme inhi bitor cilastatin? Imipenem 152.What is the drug of choice for penicillin-resistant gonococcalinfections? Spectinomycin 153.Which drugs block the enzyme DNA gyrase? Quinolones/nalidixic acid

154.What aminoglycoside is used before surgery to sterilize the bowel? Neomycin 155.What aminoglycoside causes disruption of CN I? Streptomycin Rifampin

156.Which anti-Tb drug gives orange urine, saliva, and tears?

157.What is the drug of choice for amebic dysentery caused by Bacillus fragil%s? Metronidazole 158.Which topical agent blocks the enzyme isoprenyl phosphate? Bacitracin 159.What is the drug of choice for Legionella, Mycoplasma, and Campylobacter inf ections? Erythromycin 160.What drug is used to treat Tb, tularemia, and the plague? Streptomycin

161.What is the drug of choice for methicillin-resistant Staphylococcus aureus? Vancomycin 162.Which drugs bind to the 30S ribosomal subunit and interfere with the initiat ion complex, causing a misreading of mRNA? Aminoglycosides 163.What is the drug of choice for asymptomatic meningitis carriers? 164.Which three aminoglycosides have vestibular toxicity? 2. Gentamicin 3. Tobramycin Rifampin

1. Streptomycin

165.What are the two most important features in the diagnosis of malaria? 1. Splenomegaly 2. Anemia (With a high index of suspicion) 166.Which antihelmintic increases membrane permeability to Ca2+ and is the drug of choice for schistosomiasis? Praziquantel 167.What is the drug of choice in the treatment of the lepra reaction? Clofazim ine 168.What is the drug of choice for taeniasis which inhibits oxidative phosphoryl ation in cestodes? Niclosamide 169.What is the drug of choice for threadworm, trichinosis, and larva migrans? Thiabendazole 170.What inhibitor of microtubule synthesis is the drug of choice for whipworm a nd pinworm? Mebendazole 171.Which bacteriostatic drug inhibits folic acid synthesis and a major side eff ect is the lepra reaction? Dapsone 172.What mosquito is responsible for the transmission of malaria? s mosquito 173.What is the drug of choice for filariasis and onchocerciasis? arbamazine Anophele Diethylc

174.What drug of choice for ascaris causes neuromuscular blockade of the worm? Pyrantel pamoate 175.What drug blocks glucose uptake, leading to decrease formation of adenosine 5' triphosphate (ATP) and resulting in immobilization of the parasite? Albendaz ole 176.What drug is an irreversible inhibitor of the Na+/K+ pump? Omeprazole 177.What is the drug of choice for: Herpes simplex virus (HSV) infections? Acyclovir or trifluridine

178.What is the drug of choice for: Varicella-zoster?

Acyclovir Ganciclo

179.What is the drug of choice for: Cytomegalovirus (CMV) infections? vir

180.What is the drug of choice for: Human immunodeficiency virus (HIV) infection s? azidothymidine (AZT) 181.What is the drug of choice for: Influenza and rubella infections? ne Amantadi

182.What is the drug of choice for: Respiratory syncytial virus (RSV) infections ? Ribavirin 183.What is the drug of choice for: Human papilloma virus (HPV) infections? a-Interferon 184.What antihistamine is used in the treatment of serotonergic crisis? Cyprohep tadine 185.What are the three nonsedating antihistamines? zole 3. Loratadine 186.Skin necrosis is caused by a deficiency in what? ciency 1. Terfenadine 2. Astemi Transient protein C defi

187.What D2 receptor blocker is used as an antiemetic? Chlorpromazine 188.What prostaglandin El (PGE1) analog is used in the treatment of ulcers cause d by the excessive use of NSAIDs? Misoprostol 189.What 2-agonist is used as a prophylactic agent in the treatment of asthma? Salmeterol 190.What is the only form of insulin that can be given IV? Regular insulin

191.What drug, if given during pregnancy, would cause the uterus to exhibit sign s of progesterone withdrawal and induce an abortion? RU 486 192.What two forms of insulin, if mixed together, would result in precipitation of zinc? 1. Lente insulin 2. NPH insulin or protamine zinc insulin (PZI) 193.What blood disorder is a side effect of metformin? Megaloblastic anemia (de creased absorption of vitamin B12 and folic acid) 194.Which antineoplastic agents' site of inhibition is in: M phase of the cell c ycle? Vinblastine and vincristine Gl phase of the cell cycle? L-asparaginase and mitomycin G2 phase of the cell cycle? Bleomycin Between GI and S phases of the cell cycle? Hydroxyurea Between S and G2 phases of the cell cycle? Etoposide S phase of the cell cycle? Cytarabine, methotrexate, 6-mercaptopurine, and 6-thioguanine 195.What is the physiologic basis for the actions of birth control pills? They block the midcycle surge of luteinizing hormone (LH) 196.True or false: Oral sulfonylureas increase the number of insulin receptors. False-verify this answer.

197.What drugs combined together produce neuroleptanalgesia? increase insulin release and decrease glucagon release. 198.What drugs combined together produce neurolepanalgesia? entanyl

True. They also Droperidol and f

199.What ultra-short-acting barbiturate induces hypnosis and is associated with cardiovascular and respiratory depression? Thiopental 200.What is the most common pain killer used during pregnancy? Meperidine 201.What antimicrobial agent's major side effect is: Gray baby syndrome? Chloramphenicol 202.What antimicrobial agent's major side effect is: CN VIII damage (vestibuloto xic)? Aminoglycosides 203.What antimicrobial agent's major side effect is: Teratogenicity? azole Metronid

204.What antimicrobial agent's major side effect is: CholestaHc hepatitis? Erythromycin 205.What antimicrobial agent's major side effect is: Hemolytic anemia? Nitrofur antoin 206.What antimicrobial agent's major side effect is: Dental staining if used in the pediatric population? Tetracycline 207.What antimicrobial agent's major side effect is: Altered folate metabolism? Trimethoprim 208.What antimicrobial agent's major side effect is: Auditory toxicity? Vancomyc in 209.What antimicrobial agent's major side effect is: Cartilage abnormalities? Quinolones 210.What are the five zero-order processes? 3. Phenytoin 4. Alcohol 5. Aspirin toxicity 1. Sustained release 2. IV drip Hydromor

211.What morphine derivative is used in patients with renal failure? phone

212.Which group of antihypertensive agents decreases left ventricular hypertroph y the best? Thiazide diuretics 213.What is the site of action of: Osmotic diuretics? g the thick ascending limb 214.What is the site of action of: Loop diuretics? The entire tubule barrin Ascending limb

215.What is the site of action of: Thiazide diuretics? Early distal tubule 216.What is the site of action of: K+-sparing diuretics? duct 217.What is the site of action of: Aldosterone antagonists? d tubules Early collecting Distal convolute

218.What is the only diuretic that works on the blood side of the nephron? Spironolactone (binds to aldosterone receptors) 219.What is the active metabolite of spironolactone? Canrenone

220.What drug is given transdermally for chronic pain but can cause chest wall r igidity if given IV? Fentanyl 221.What is the lipid solubility and potency of a drug if the induction and reco very from the drug were both rapid? Low lipid solubility and potency (They a re inversely proportional.) 222.What is the drug of choice for hypertensive patients with a decreased renal function? a-Methyldopa (Guanabenz or clonidine is also used.) 223.What hormone is released by the atria due to an increase in blood pressure t o cause an increase in glomerular filtration rate (GFR), Na+ retention, and reni n-angiotensin release? Atrial natriuretic factor 224.What is the drug of choice in treatment for the late phaseof asthma? Corticosteroids 225.Which antigenic thrombolytic agent causes a decreased level of circulating f ibrinogen? Streptokinase 226.What is the only class of diuretics to retain Cl- used in the short-term tre atment of glaucoma and also in the treatment of acute mountain sickness? Acetazolamide 227.What diuretic is used to decrease intraocular and intracranial pressures? Mannitol 228.Which thrombolytic agent, activated in the presence of fibrin, is manufactur ed by recombinant DNA process? Alteplase 229.Which diuretic causes irreversible ototoxicity and GI bleeding as its main s ide effects? Ethacrynic acid 230.What IV agent is used to treat respiratory depression associated with withdr awal from alcohol usage? Naloxone 231.What is the best form of treatment if an elevated blood pressure is due to: Elevated heart rate? -Blockers 232.What is the best form of treatment if an elevated blood pressure is due to: Elevated force of contractions? -Blockers 233.What is the best form of treatment if an elevated blood pressure is due to: Increase in fluid volume? Diuretics or angiotensin converting enzyme (ACE) inhibitors 234.What is the best form of treatment if an elevated blood pressure is due to: Increase in TPR? Centrally acting sympatholytics, a-antagonists, or Ca2+ channel bockers 235.Which class of diuretics blocks Na/Cl cotransport in the distal tubules? Thiazide diuretics 236.Why should codeine be carefully administered acetaminophen or aminosalicylic acid? Because it has an additive effect with with these agents

1.What muscle comprises the upper esophageal sphincter? Cricopharyngeus 2True or false: chewing is essential for digestion? the surface area of the food. False; it just increases

3.In which region of the stomach are parietal and chief cells located? Body or corpus 4.What hormone, released in response to low pH, inhibits gastric emptying by dec reasing antral contractions, increases constriction of the pyloric sphincter, an d also increases bicarbonate secretions from the pancreas? Secretin 5.In which region of the stomach are G cells located? trin: G for gastrin.) Antrum (They secrete gas 3 to 4 days 2 to 4 hours

6.How long is the transit time through the large intestine? 7.How long is the transit time through the small intestine?

8.What hormone causes contractions of smooth muscle, regulates interdigestive mo tility, and prepares the intestine for the next meal? Motilin 9.What is the main function of HCl in the stomach? pepsin Converts pepsinogen into

10.What hormone increases the intestinal secretions of electrolytes and H2O, rel axes smooth muscle, dilates peripheral blood vessels, and inhibits gastric secre tions? Vasoactive intestinal peptide (VIP) 11.What gland produces 20% of salivary secretions and contributes to almost all of the amylase secretions? Parotid gland (serous secretions) 12.What hormone causes contractions of the gallbladder, augments the action of s ecretin to produce an alkaline pancreatic juice, inhibits gastric emptying, and increases constriction of the pyloric sphincter? Cholecystoldnin (CCK) 13.What are the four functions of saliva? 1. Provides antibacterial action 2. Lubricates 3. Begins carbohydrate digestion 4. Begins fat digestion 14.What is composed of skeletal muscle, innervated by the pudendal nerve, and in a voluntary constant state of contraction that relaxes for defecation? External anal sphincter 15.What hormone is stimulated by glucose and fat in the duodenum, inhibits gastr ic secretions and motility, and stimulates insulin secretion? Gastrin inhibito ry peptide (GIP) 16.Which portion of the autonomic nervous system regulates salivary flow? Parasympathetic portion 17.A pH of less than 4.5 stimulates the release of what hormone? (It inhibits acid Secretion.) Secretin

18.What is composed of smooth muscle, innervated by pelvic splanchnics and hypog astric nerves, and involuntary? Internal anal sphincter 19.What gland produces 70% of total salivary secretions? Submandibular gl

and (produces both mucous and serous secretions) 20.What is the tonicity of pancreatic juice? Isotonic Helicobacter pylori

21.What organism is associated with gastric ulcers?

22.What is the only gastric secretion required to sustain life? Intrinsic factor (IF) 23.What three structures increase the surface area of the GI tract? e circularis 2. Villi 3. Microvilli 1. Plica

24.What hormone is the primary regulator of HCO3 secretion from the pancreas? Secretin \ 25.What cells of the GI tract secrete mucus? Goblet cells 26.What are the five F's associated with gallstones? e 4. Familial 5. Fertile 1. Fat 2. Forty 3. Femal Lactase 1. Maltose 2. Ma 5% (95% reabsorbed via e

27.Lactose intolerance is caused by a lack of what enzyme? 28.What are the three end products of amylase digestion? ltotetrose 3. Alpha limit dextrans (a-1,6 binding) 29.What percentage of bile acids are excreted daily? nterohepatic circulation)

30.Which glands of the upper duodenum secrete a bicarbonaterich solution? Brunner's glands 31.What is the major route for excretion of cholesterol? 32.What hormone potentiates the effect of secretin? 33.What is the major phospholipid in bile? Lecithin 7-a-Hydr CCK Bile

34.What is the rate-limiting step in the formation of bile acids? oxylase

35.What two amino acids are conjugated to bile acids to increase H2O solubility? 1. Glycine 2. Taurine 36.What is absorbed in the gallbladder to concentrate bile? Water

37.If a substance is removed from circulation by an organ, is its arteriovenous (AV) difference positive or negative? Positive AV difference 38.In laminar flow, which area has the fastest flow? The center of the tube

39.What are four ways to get an increased pump function of the heart? 1. Exerc ise 2. Increase heart rate 3. Increase in arterial pressure 4. Increase contract ility 40.What system has an increased pressure, decreased resistance, increased flow, increased compliance, and blood volume that is proportional to flow? Pulmonar y circuit 41.What two organs have local metabolites as the main determinant of blood flow? 1. Brain (cerebral circulation) 2. GI tract (after a meal) All other organs are

under neural control. 42.What are the three sympathetic effects on the pacemaker cells of the heart? 1. Increase the slope of prepotential 2. Take less time to reach threshold 3. In crease the rate of firing 43.What are the three parasympathetic effects on the pacemaker cells of the hear t? 1. Hyperpolarize the cells by increasing K+ conductance 2. Take longer t o reach threshold 3. Decrease the rate of firing 44.What are the two major causes of arterial pressure? 1. Contraction of the he art 2. Hydrostatic pressure 45.What causes an increase in cardiac performance with no increase in preload? Contractility (inotropic) 46.Change in what intracellular ion causes a change in contractility? Calcium

47.What are the two main circulations with extrinsic regulation that are most af fected by nervous reflexes? 1. Cutaneous circulation 2. Resting skeletal mus cle 48.What is the third heart sound caused by? g diastole) 49.What is the fourth heart sound caused by? diastole) Ventricular filling (heard durin Atria] contraction (heard during

50.If a substance is put into circulation by an organ, is its arteriovenous diff erence positive or negative? Negative 51.What is the baroreceptor response to an increase in blood pressure? Increase afferent activity of CN IX and CN X to decrease heart rate (parasympathetic) 52.What is a perfusionlimited situation? d equilibrate for a substance When alveolar and capillary bloo

53.Which region of the lungs has a low perfusion pressure and a high resistance so that there is little blood flow? Apex 54.What fluid is monitored directly by central chemoreceptors? Cerebrospinal fl uid (H+;CO2) 55.On a pressure-volume loop, what is seen with: Aortic regurgitation? Increase in stroke volume On a pressure-volume loop, what is seen with:Aortic stenosis? Increase in afte rload, decrease in stroke volume, increase in peak tension On a pressure-volume loop, what is seen with:Increased contractility? Increase in stroke volume by decreasing the end-systolic volume On a pressure-volume loop, what is seen with:Heart failure? Increase in endsystolic volume, decrease in afterload, decrease in peak tension, increase in pe ak tension 56.What two compensatory mechanisms occur to reverse hypoxia at high altitudes? 1. Increase in erythropoietin 2. Increase in 2,3-bisphosphoglycerate (2,3-BPG) 57.What would you give to neutralize the excess base in an alkalotic patient? NH4Cl(strong acid can lyse RBCs) 58.What would you give to neutralize the excess acid in an acidotic patient?

NaCO2 (CO2 eliminated by lungs) 59.What is a diffusion-limited situation? When alveolar gas and capillary blood attempt to equilibrate but do NOT (i.e., CO2) 60.What must occur in order for PaCO2 to remain constant when there is an increa se in the body's metabolism? Need to increase alveolar ventilation (if not hy percapnia would result) 61.What enzyme is needed for conversion of testosterone to estradiol? e Aromatas

62.What two anions compete with iodine for the iodine pump in the thyroid gland? 1. Perchlorate 2. Thiocyanate 63.What enzyme is associated with osteoblastic activity? tase Alkaline phospha

64.What form of plasma calcium is the physiologically active form and is regulat ed within narrow limits? Free calcium (ionized) 65.Which three factors cause the release of epinephrine from the adrenal medulla ? 1. Exercise 2. Emergencies (stress) 3. Exposure to cold (The three Es) 66.What phase of the female cycle ALWAYS lasts for the same number of days (14 d ays in most women)? Luteal phase 67.What serves as a marker for 24-hour growth hormone secretion? nsulin-like growth factor type 1 (IGF-1) levels Plasma i

68.What three things inhibit the secretion of glucagon? 1. Insulin 2. Somatostat in 3. Hyperglycemia 69.Which three organs or structures have gluconeogenic capabilities? 2. Kidney 3. GI epithelium 70.Which type of diabetes is more likely to lead to ketoacidosis? insulin-dependent diabetes mellitus [IDDM]) 1. Liver Type I (

71.Excess bone demineralization and remodeling can be detected by checking urine levels of what substance? Hydroxyproline (breakdown product of collagen) 72.What two things cause l-a-hydroxylase activity to increase? 1. Parathyroid h ormone (PTH) 2. A decrease in PO4 levels 73.What type of membrane is permeable to water and small solutes? ely permeable membrane Selectiv

74.What is the movement of ions in an electrical held known as? Conductance 75.What two components of a body of water cannot be measured and need to be calc ulated? 1. Intracellular fluid (ICF) (water minus extracellular fluid) 2. Inters titial fluid (ISF) (extracellular fluid minus plasma volume) 76.What phase of an action potential has the greatest rate of Na+ influx? Phase 0 77.Which phase of an action potential requires energy? Phase 4 (via the Na+/K+ pump)

78.What type of muscle is associated with one T tubule and two cisternae (triad) ? Skeletal muscle 79.What is the region of an axon where no myelin is found? 80.What types) of muscle contain the thin filament troponin? diac muscle Nodes of Ranvier Skeletal and car

81.Where are antidiuretic hormone (ADH) and oxytocin produced? The supraoptic a nd paraventricular nuclei of the hypothalamus 82.What is the only hormone to INCREASE with a DECREASE in pituitary function? Prolactin 83.Which enzyme converts cholesterol to pregnenolone? step (RLS) in steroid hormone synthesis Desmolase-rate limiting

84.What are the four "stress" hormones? 1. Growth hormone (GH) 2. Glucagon 3. Co rtisol 4. Epinephrine 85.Proopiomelanocortin (POMC) is cleaved into what two substances? 1. Adren ocorticotrophic hormone (ACTH) 2. (alpha)-Lipotropins (melanotropins and endorph ins) 86.What are the six substances that promote the secretion of insulin? 1. Gluco se 2. Amino acid (arginine) 3. Gastrin inhibitory peptide (GIP) 4. Glucagon 5. A lpha-Agonists 6. ACh 87.What is the thin filament that has the attachment site for the crossbridges a nd also activates adenosine triphosphatase (ATPase)? Actin 88.What types of muscle have a sarcomere? Skeletal and cardiac muscle

89.Where is the action potential generated on a neuron? Axon hillock 90.What is the name for the load that the muscle is working against during stimu lation? Afterload 91.What type of contraction has an active tension when the length is shortened? Isotonic contraction 92.What type of muscle has high creatinine phosphokinase (CPK), high ATPase acti vity, and no myoglobin; is anaerobic; and is for short-term use? White mu scle (fast) 93.What type of muscle uses calmodulin? Smooth muscle 94.What thick filament has crossbridges and ATPase activity? 95.What causes actin-myosin crossbridge dissociation? 96.What is used as an index of cortisol secretions? Myosin

Binding of ATP Urine 17-OH steroids

97.What would be the two major consequences if the zona fasciculata and the zona reticularis were removed? 1. Circulatory failure 2. Inability to mobilize energy stores 98.How many carbons do androgens have? Androgens are 19-carbon steroids. 99.How many carbons do estrogens have? Estrogens are 18-carbon steroids. (Remov

al of one carbon from an androgen = an estrogen.) 100.The level of what hormone tends to DECREASE with stress? Insulin

101.On what two occasions are cortisol releasing hormone (CRH) secretions elevat ed? 1. Early morning 2. During stress 102.What is an inhibitory interneuron known as? Renshaw neuron 103.What is the summation of mechanical stimuli known as? 104.What is the thin filament that binds to calcium? Tetany

Troponin C

105.What determines the maximum velocity of shortening muscle? The muscle's ATP ase activity 106.What type of muscle has end plates? Skeletal muscle 107.What type of contraction has an active tension, but the overall length of th e con traction does not change and no work is done? Isometric contraction 108.What thin filament covers the attachment site in resting muscle so that the crossbridges are unavailable for binding? Tropomyosin 109.What is the load on a muscle in the relaxed state known as? Preload 110.Total tension - preload = what? 111.What types of muscle are uninuclear? Active tension (contraction) Cardiac and smooth muscle

112.In a contractile muscle, what is the source of the calcium? Sarcoplasmic ret iculum (The source is NOT extracellular.) 113.What is the maximum force of a contraction determined by? tor units activated during the contraction The number of mo

114.What types) of muscle have T tubules associated with them? Cardiac and skel etal muscle 115.What type of muscle has myoglobin, low CPK, and low ATPase activity; is aero bic; and is for long-term use? Red muscle (slow-twitch muscle) 116.What event signifies the first day of the menstrual cycle? The first day of bleeding 117.What hormone is essential for induction of ovulation and formation of the co rpus luteum? Luteinizing hormone (LH) 118.How many days before the first day of bleeding is ovulation? in most women (Remember: The luteal phase is always constant.) 119.What is required to maintain lactation? cretion) 14 days

Suckling (stimulates oxytocin se Estrogen

120.What hormone, in high levels, blocks milk production?

121.What hormone is necessary for maintenance of the corpus luteum for the first 3 months of pregnancy? Human chorionic gonadotropin (hCG)- from the trophoblast 122.Up to how many hours after ejaculation are sperm able to fertilize the egg?

72 hours 123.What hormone induces myometrial contraction and causes milk letdown? Oxytocin 124.What hormone is necessary for the maintenance of the uterine endometrium fro m the fourth month of pregnancy on? Progesterone (Estrogen is needed for pro gesterone to be effective.) 125.How long after ovulation does fertilization occur? 8 to 25 hours 126.What hormone thins cervical mucus, stimulates LH receptors on granulosa cell s, elicits the LH surge, and increases proliferation of the uterine mucosal laye rs? Estradiol 127.What hormone is secreted by the placenta late in pregnancy, stimulates mamma ry growth during pregnancy, mobilizes energy stores from the mother so that the fetus can utilize them, and has an amino acid sequence like GH? Human chorionic somatomanunotropin (hCS) or human placental lactogen (hPL) 128.What hormone causes an increase in the production of milk? Prolactin 129.What is the force necessary to collapse the lung known as? Lung recoil 130.For what hormone do Leydig cells have receptors? LH

131.What vitamin needs thyroid hormone for conversion to its active form? Vitamin A 132.What is the tonicity of fluid that leaves the loop of Henle? c 133.What enzyme converts androgens to estrogens? Aromatase Hypotoni

134.What does excess production of thyroid-stimulating hormone (TSH) cause? A goiter 135.What type of cell reabsorbs bone? Osteoclast; Blasts make; clasts take.

136.What is the major form of androgen secreted from the adrenal gland? Dehydroe piandrosterone (DHEA) 137.What cells of the genitourinary system produce testosterone in males? Leydig cells 138.What type of urine does ADH cause to be excreted? e of the water reabsorption in the collecting duct) Hypertonic urine (becaus

139.What is the term for the volume of plasma removed from a substance per unit time? Clearance 140.What is the most potent male sex steroid? 141.What two substances stimulate Sertoli cells? mone (FSH) and testosterone Dihydrotestosterone - DHT Follicle stimulating hor 1. Liver 2. Kidne

142.At which three sites in the body is T4 converted to T3? y 3. Pituitary gland (via 5'-deiodinase enzyme)

143.The fresh air being delivered to the respiratory zone per minute is known as

what? Alveolar ventilation (the first 150 ml is not included) 144.What region of the lungs gets very little ventilation? Apex

145.Where does polyuric originate if the patient is dehydrated and has electroly te deficiencies? Before the collecting duct (There is no electrolyte dist urbance in the collecting duct.) 146.What substance is free filtered but partially reabsorbed by passive mechanis ms? Urea 147.What hormone promotes mobilization of energy stores, enhances the capacity o f glucagon and catecholamines, and increases the capacity to withstand stress? Cortisol 148.What is used as an index of androgen secretion? Urine 17-ketosteroids

149.What are the pituitary hormones associated with: Thyrotropin releasing hormo ne (TRH)? Thyroid stimulating hormone (TSH) 150.What are the pituitary hormones associated with: Cortisol releasing hormone (CRH)? Adrenocorticotrophic hormone (ACTH) 151.What are the pituitary hormones associated with: Gonadotropin releasing horm one (GnRH)? Luteinizing hormone (LH) and follicle stimulating hormone (FSH) 152.What are the pituitary hormones associated with: Growth hormone releasing ho rmone (GHRH)? Growth hormone (GH) 153.What are the pituitary hormones associated with: Somatostatin? GH secretion Inhibits

154.What are the pituitary hormones associated with: Prolactin inhibiting factor (PIF) [dopamine]? Inhibits prolactin secretion 155.Which hormones are released from the: Zona glomerulosa? t) 156.Which hormones are released from the: Zona fasciculata? 157.Which hormones are released from the: Zona reticularis? Aldosterone (sal Cortisol (sugar) Androgens (sex)

158.Which hormones are released from the: Medulla? NE:Epi (1:4) If the zona glomerulosa were removed from the adrenal gland, what would be seen? Decrease in Na+ causing a decrease in the ECF volume, leading to a decrease in B P, and eventually to circulatory shock and death 159.What does subatmospheric pressure (negative) do to the lungs? It causes t hem to expand (because of the decrease in intrathoracic pressure) 160.Where is the last conducting zone of the lungs? Terminal bronchioles (No gas exchange occurs here.) 161.Where is there summation, hyperpolarization of the postsynaptic membrane, an increase in Cl- conductance, and local gradation? Inhibitory postsynaptic po tential (IPSP) 162.Which extravascular chemoreceptor detects low NaCI concentrations? a densa 163.What is the major stimulus for cell division in chondroblasts? Macul Insulin-li

ke growth factor-1 (IGF-1) 164.The total air in and out of the respiratory system per minute is known as wh at? The total ventilation (minute volume or minute ventilation) 165.What is the major hormone secreted by the ovarian follicle? 17 alpha-Estradi ol 166.What two conditions cause ADH to be released? levated plasma volume (high solute concentration) 1. Low blood volume 2. E

167.What cell converts androgens to estrogens? Granulosa cell 168.What hormone acts on this cell? FSH 1. Resid

169.What three lung volumes cannot be measured with a spirometer? ual volume 2. Total lung capacity 3. Functional residual capacity

170.What two conditions decrease the secretion of aldosterone? 1. An increase i n blood pressure 2. Weightlessness 171.What cell in the female genitourinary system is stimulated by LH and is the site where androgens are produced? Thecal cell 172.What serves as a marker of endogenous insulin secretions? C peptide

173.What do you have when there is depolarization of the postsynaptic membrane o wing to an influx of Na+, resulting in summation and local gradation? Excitato ry postsynaptic potential (EPSP) 174.What are days 15 to 28 in the female cycle known as? Luteal phase

175.What hormone is secreted by the Sertoli cells to decrease FSH production? Inhibin 176.What hormone regulates osmolarity because it controls water excretion? ADH (It causes water reabsorption.) 177.What is the term for the air in the system after maximal inspiration? Total lung capacity (TLC) 178.What is a sign of a Sertoli cell tumor in a man? blood Excess estradiol in the

179.What hormone is responsible for the negative feedback onto LH and FSH of the anterior pituitary and positive feedback onto the granulosa cells? Estrogen 180.What is the term for the total dead space of the lungs? space 181.The surge of what hormone induces ovulation? LH Physiologic dead

182.What does positive pressure do to the lungs? It collapses them. 183.What is the term for the air that can be taken in after normal inspiration? Inspiratory reserve volume (IRV) 184.What is the first zone of the lungs that is capable of O2 exchange? atory bronchioles (because they have alveoli) Respir

185.What is the term for ventilation of underperfused alveoli? Alveolar dead sp ace 186.What is the temperature of the scrotum? 4 degrees cooler than the body By a countercurr

187.How is the lower temperature of the testes maintained? ent heat exchanger in the spermatic cord

188.What happens to sex steroids, LH, and FSH: If the gonads are removed? Sex steroids decrease; LH increases; FSH increases. 189.What happens to sex steroids, LH, and FSH: In postmenopausal women? Sex ster oids decrease; LH increases; FSH increases. 190.What happens to sex steroids, LH, and FSH: After the administration of testo sterone? Sex steroids increase; LH decreases; nothing happens to FSH. 191.What happens to sex steroids, LH, and FSH: After the administration of inhib in? Nothing happens to sex steroids; nothing happens to LH; FSH decreases. 192.What happens to sex steroids, LH, and FSH: With constant infusion of GnRH? Sex steroids decrease; LH decreases; FSH decreases (needs to be given pulsatile) . 193.What region of the lungs is incapable of gas exchange? ace Anatomic dead sp

194.What is the term for the amount of air that can never leave the lungs? Residual volume 195.If you increase the depth of breathing, what ventilatory parameters can be i ncreased? Total ventilation and alveolar ventilation 196.If you increase the rate of breathing, what ventilatory parameters can be in creased? Total ventilation 197.What is the term for the air left in the lungs after normal expiration? Functional residual capacity (FRC) 198.The lung volume from maximum inspiration to maximum expiration is known as w hat? Vital capacity (VC) 199.Where is renin produced? In the juxtaglomerular ( JG) cells of the kidney

200.What phase of the female cycle occurs during days 1 to 15? Follicular phase 201.What hormone level peaks 1 day before the surge of LH and FSH in the female cycle? Estradiol 202.What is the day after the LH surge in the female cycle known as? n Ovulatio

203.By what mechanism does chronic constriction keep blood flow through the peni s low during non-aroused states? Alpha-Adrenergic mediated constriction 204.Days 1 to 7 of the female cycle are known as what? Menses 205.The amount of air that enters or leaves the respiratory system in a single r espiratory cycle is known as what? Tidal volume

206.What part of the autonomic nervous system is responsible for the movement of semen through the vas deferens and related structures? Sympathetic nervous syst em 207.Which pancreatic cells secrete glucagon? 208.Which pancreatic cells secrete somatostatin? Alpha cells Delta cells

209.What term describes how easily a vessel stretches? Compliance (pulse pressu re is inversely proportional to compliance) 210.What is the most compliant artery in the body? Aorta Via total periph Afterload

211.What is the best way to regulate mean arterial pressure? eral resistance (TPR) 212.What is the term for resistance to ventricular outflow?

213.What is the main determinant of resistance? The radius of the vessel (also t he viscosity and length) 214.What is the relationship between Na+ reabsorption and O2 consumption? An increase in Na+ causes Oz consumption to increase. 215.What vessels have the greatest cross-sectional area? Capillaries

216.What is the nontitratable acid that buffers secreted H+ in the kidney buffer ed as? NH4+ (ammonium). H2PO4 (dihydrogen phosphate) is the titratable acid tha t buffers secreted H+. 217.What are the five ways to promote turbulent flow? 1. Increase velocity 2. Branching 3. Narrow orifice 4. Increase tube diameter 5. Decrease viscosity 218.What part of the cardiovascular system has the lowest drop in pressure? Right atrium 219.What vessels are the resistance vessels and have the largest drop in pressur e? Arterioles 220.What part of the autonomic nervous system is the main controller of blood fl ow when a person is at rest? Sympathetic nervous system (alpha constricts, be ta-2 dialates. 221.What process occurs when hydrostatic pressure exceeds plasma oncotic pressur e? Filtration 222.What vessels have the greatest blood volume? wer extremities Systemic veins of the lo

223.What vessels have the smallest total cross-sectional area? The aorta, then the vena cava 224.What is the only way to increase O2 delivery to the myocardium? the blood flow Increase

225.What happens to airway resistance during inspiration? It decreases. Sy mpathetics decrease resistance; parasympathetics increase resistance. 226.What is the main drive for ventilation? The PCO2 of systemic circulation

227.Where does depolarization in the heart begin? e and from the endocardium to the epicardium

From the apex to the bas

228.Where is the greatest venous PO2 in resting tissue? Renal circulation 229.The load on the muscle in the relaxed state is known as what? (also the end-diastolic volume [EDV]) 230.What cells of the heart have the highest rate of automaticity? al (SA) nodal cells Preload Sinoatri

231.What are the slowest conducting cells of the heart? Atrioventricular (AV) no dal cells 232.What is the main control of flow in exercising muscle? bolites Vasodilator meta

233.During what phase of the cardiac cycle do coronary vessels receive their blo od flow? Diastole 234.On the venous pressure curve, what do the following waves represent -a wave, -c wave, -v wave? Atrial contraction, Ventricular contraction, Atrial fill ing (venous filling) Atrial contraction, Venous 235.What causes the second heart sound? Aortic closure 236.Where is the second heart sound on an EKG? At the T wave 237.What causes the diastolic interval to decrease? te Increase in the heart ra

238.What happens to cerebral circulation during hypoventilation? Blood fl ow increases because PCO2 is increased. (The opposite occurs during hyperventila tion.) 239.What is the main factor affecting PaCO2? Alveolar ventilation (Hyperventi lation decreases PaCO2 and vice versa. Body metabolism also affects PaCO2') 240.What are the normal values for: PaO2? 241.What are the normal values for: PaCO2? 242.What are the normal values for: PvO2? 243.What are the normal values for: PvCO2? 100 mmHg 40 mmHg 40 mmHg 47 mmHg 1. Increase preload (EDV

244.What are the two ways to increase stroke volume? ) 2. Decrease end-systolic volume (ESV)

245.What is the main factor that determines the glomerular filtration rate (GFR) ? Hydrostatic pressure 246.What is the normal value for the GFR? 120 ml/min

247.What happens to pulmonary blood flow under conditions of low alveolar PO2? A decrease in blood flow secondary to vasoconstriction 248What is the normal compensatory mechanism for a state of metabolic alkalosis? Hypoventilation (respiratory acidosis)

249.What anion is excreted in large amounts in the urine in a patient with a com pensated alkalosis? Bicarbonate (alkaline urine) 250.Which hormone affects the osmolarity? ADH

251.What are the sympathetic effects on the kidney? A decrease in GFR and an increase in filtration fraction: FF = GFR/RPF. (There is a larger decrease in t he RPF than the GFR, resulting in an increase in the filtration fraction.) 252.What are the effects of angiotensin II on the kidney? the efferent arterioles Constriction of

253.What four changes occur with an increase in contractility? 1. Increased slo pe of action potential 2. Increased peak left ventricular pressure 3. I 254.Increased rate of relaxation 4. Decreased systolic interval 255.What fibers of the heart have the lowest intrinsic rate of automaticity? Purkinje fibers 256.What causes the first heart sound, and when does it occur on an EKG? Mitral valve closure at the QRS complex 257.What is the main determining factor of filtration fraction? Renal plasma flo w (decreases flow; increases filtration fraction) 258.What is the normal osmolarity of the filtrate in the renal tubule? 300 mOsm 259.What prohibits the filtering of protein anions into the renal tubule? The negative charge on the filtration membrane 260.What are the fastest conducting fibers in the heart? Purkinje fibers

261.From which point to which point does repolarization travel in the heart? From the base to the apex and from the epicardium to the endocardium (opposite o f depolarization) 262.What are the three characteristics of autoregulation? 1. Flow independ ent of BP 2. Flow proportional to local metabolism 3. Flow independent of nervou s reflexes 263.What is/are the major autoregulators of: Cerebral circulation? in PCO2 264.What is/are the major autoregulators of: Coronary circulation? in PO2; increase in PCO2 and adenosine 265.What is/are the major autoregulators of: Exercising skeletal muscle? Lactate 266.What process occurs if the capillary oncotic pressure is greater than the hy drostatic pressure? Reabsorption 267.What area of the circulatory system houses the greatest blood velocity? Aorta 268.What two things happen to cutaneous circulation when the sympathetic nervous system is stimulated? 1. Constriction of arterioles to decrease blood flow 2. Constriction of the venous plexus to decrease the blood volume 269.Adenosine in the kidney, decreased PO2 in the lungs, and thromboxane A2 (TXA Increase Decrease

2) have what effect in the circulation? Vasoconstriction 270.What is the period when higher than normal stimulation is required to induce a second action potential? Relative refractory period 271.During an action potential, what is the stimulus for opening the Na+ channel s? Depolarization 272.What substance "affects" the action potential? Na+ conductance K+ conductance

273.What substance "affects" the resting membrane potential?

274.In which direction do osmotically active substances cause water to move? Toward them 275.What are the three tracers for total body water? itrated water 1. Urea 2. Thiourea 3. T

276.What are the three characteristics of an action potential? 1. All or none 2 . Propagated 3. No summation 277.In what system is the second greatest blood volume found? Pulmonary system

278.What are the four ways to increase total peripheral resistance (TPR)? 1. Decrease the radius 2. Increase the viscosity 3. Increase the length 4. Decre ase the number of parallel channels 279.What type of system is a high resistance system with flow equal at all point s and where total resistance is the SUM of the individual resistances? Vessels connected in a series 280.What effect on a blood vessel does each of the following have: histamine, br adykinin, prostaglandins (A2, E2, I2), nitric oxide, adenosine, an increase in K +, H+, PCO2 and a decrease in PO2? Vasodilatory effect 281.What are three characteristics of a subthreshold potential? 1. Graded 2. Sum mation 3. Not propagated 282.What is the depolarization phase of an action potential caused by? Na+ infl ux 283.If the ventilation-perfusion ratio is less than 1, what part of the lung is involved and what physiologic process is occurring? The base, because flow e xceeds delivery of O2 284.How do you compensate for metabolic acidosis? ory alkalosis) 285.How many liters of water How many liters of water are How many liters of water are How many liters of water are How many liters of water are are there there in: there in: there in: there in: Hyperventilate (respirat 42 L

in: Total body water? ICF? 28 L ECF? 14 L ISF? 10.5 L Plasma volume? 3.5 L

286.What hormone affects fluid volume? Aldosterone (Na+ content determines the volume of the plasma.) 287.The repolarization phase of the action potential is caused by what? K+ efflu x (depolarization opens the gates)

288.What is the name of the period in which, no matter how strong the stimulus, a second action potential cannot be generated? Absolute refractory period 289.What hormone is necessary to maintain normal thyroid hormone levels? GH 290.What region of the lung has the greatest blood flow? The base

291.During inspiration, which region of the lung receives the greatest level of ventilation: the apex or the base? The base 292.What is the function of the stretch receptors in the lungs? To prevent overd istention of the lungs (inhibits inspiration) 293.Where does the inherent rhythm for respiration originate? center of the medullary oblongata In the medullary

294.Where is the deep breathing center located? Apneustic center in the pons 295.What type of system is a low-resistance system in which the total resistance is always less than any individual resistance, and the reciprocal of the total resistance is the sum of the reciprocal resistances? System connected in para llel 296.What is happening to the renal arteriole in each of the following situations : Increased GFR, increased glomerular pressure, decreased RPF, increased FF? Constriction of efferent arteriole 297.What is happening to the renal arteriole in each of the following situations : Decreased GFR, increased RPF, decreased glomerular pressure, decreased FF? Dilatation of the efferent arteriole 298.What is happening to the renal arteriole in each of the following situations : Decreased GFR, decreased RPF, decreased glomerular pressure? Constriction of the afferent arteriole 299.What is happening to the renal arteriole in each of the following situations : Increased GFR, increased RPF, increased glomerular pressure? Dilatation of th e afferent arteriole 300.Which region in the lungs gives the best ventilation - perfusion ratio? The hilum 301.What causes peripheral chemoreceptors to be stimulated? A decrease in di e arterial PO2, H+, and PCO2 of the normal drive for ventilation) 302.What is secreted by the parafollicular C cells of the thyroid? in 303.What is the titrated acid that the secreted H+ is buffered as? Calciton H2PO4

304.What type of dehydration is associated with hemorrhage, burns, vomiting, and diarrhea? Isotonic dehydration 305.What is the potential at which concentrations are equal and opposite to the electrical forces, and also at which there is no net flux of ions across the mem brane? Equilibrium potential (Nernst's equation) 306.What hydration state is caused by the ingestion of salt water? ic overhydration Hyperton

307.What is the free water clearance if the osmolarity of urine is greater than 300 mOsm? Negative free water clearance (concentrated urine) 308.What is the term for the process of water traveling from a low solute to a h igh solute concentration? Osmosis 309.What three factors increase simple diffusion? 1. Increased solubility 2. Increased concentration gradient 3. Decreased thickness of the membrane 310.What type of dehydration is associated with Addison's disease? c dehydration 311.When is GH released? At night and during puberty Hypotoni

312.What is protein-mediated transportation down a concentration gradient known as? Facilitated transport 313.What determines the level of alveolar ventilation? Central chemoreceptors ( PCO2) 314.Which point in the lungs is involved if the ventilation - perfusion ratio is greater than 1? Apex 315.Why is the V-P ratio in the apex of the lung greater than 1? exceeds the flow 316.How is CO2 carried in the blood? As plasma bicarbonate Delivery

317.To what hydrated state can excess ingestion of water or syndrome of inapprop riate antidiuretic hormone (SIADH) lead? Hypotonic overhydration 318.What are the four major anabolic hormones? 1. Insulin 2. Thyroid hormone 3. GH 4. Sex steroids 319.What are the eight insulin INdependent tissues? 1. CNS 2. RBCs 3. Renal tubules 4. Testis 5. Teeth 6. cells 7. Liver 8. Intestinal epithelium 320.What are the growth factors released from the liver called? Somatomedins 321.What state of hydration would you be in if you had edema and if you ingested an excessive amount of salt? Isotonic overhydration 322.What type of cell lays down bone? take. Osteoblast Remember: Blasts make; clasts

323.What is the only condition in which giving enriched O2 will not significantl y increase PaO2? Pulmonary shunt 324.What is the biologically active form of thyroid hormone? T3

325.For how many months can you store thyroid hormone? 2 to 3 months 326.What is the ratio of T4 to T3? 20:01 Osteocvte

327.What type of cell is surrounded by mineralized bone?

328.What type of dehydration is associated with excess sweating, decreased water intake, fever, alcoholism, lithium salts, excess evaporation, and diabetes insi pidus? Hypertonic dehydration

329.During what part of the cardiac cycle do you hear: Aortic stenosis? Systole 330.During what part of the cardiac cycle do you hear: Mitral stenosis? Diastole 331.During what part of the cardiac cycle do you hear: Mitral regurgitation? Systole (pan) 332.During what part of the cardiac cycle do you hear: Aortic regurgitation? Diastole 333.What four factors affect the rate of diffusion for any process? 1. Surfa ce area 2. Thickness of the membrane 3. Concentration gradient 4. Solubility (ma in factor) 334.What does a decrease in GH in adolescence lead to? Dwarfism 335.What type of transportation requires ATP and is protein mediated against a c oncentration gradient? Primary active transportation 336.What does an increase in GH in adolescence lead to? Gigantism 337.What gas has a low driving force but high solubility? 338.What are two causes of diffusion impairment in the lungs? urface area 2. Increase in membrane thickness (PAO2 > PaO2) CO2 1. Decrease in s

339.What is evident in the urinalysis of a compensated acidotic patient? Low HCO3 - excretion (acidotic) 340.What does angiotensin II do to restore blood pressure? vasoconstrictive effect. It has a direct

341.When is systemic venous blood delivered to the left side of the heart withou t O2 exchange in the alveoli? In a pulmonary shunt 342.What gas has a high driving force and low solubility? O2

343.In what type of shunt do you see an increase in right atrial, ventricular, a nd pulmonary arterial PO2, along with an increase in pulmonary blood flow? Left-to-right shunt 344.What causes Ca+ and PO4 to be reabsorbed from the kidney and Ca+ and PO4 to be absorbed from the GI tract, and also promotes bone synthesis? Vitamin D3 345.What does excess secretion of GH in an adult lead to? Acromegaly

346.The rate at which a substance is filtered into Bowman's capsule is known as what? Filtered load rate (GFR X plasma concentration) 347.What part of the nephron has the greatest osmolarity? of Henle (1200 mOsm) 348.At what region of the nephron does H+/HCO3 - exchange occur? ubule Tip of the loop Distal t

349.During what type of heart block do the atria and the ventricles beat indepen dently of each other? Third-degree heart block

350.What is the length of systole on a pressure curve? From the beginning of th e isovolumic contraction (IVC) to the beginning of the isovolumic relaxation (IV R) 351.From which point to which point does it appear on an EKG? the T wave (S1 to S2) From the QRS to

352.What are the four features of aortic stenosis? 1. Increase in afterload 2. Increase in LV pressure 3. Increase in the pressure gradient between the LV and the aorta 4. Crescendo-decrescendo systolic ejection murmur (early systolic ejection click) 353.When is surface tension the greatest in a respiratory cycle? nd of inspiration At the e

354.What type of transportation requires ATP, can be co- or countertransport, an d is a protein-mediated transport with a concentration gradient? Secondar y active transportation 355.What lung pathology is associated with a decrease in FEV1/FVC? structive) 356.What is the most important factor in describing lung recoil? tension (also fibers of tissue) COPD (ob Surface

357.What is the free water clearance if the osmolarity of urine is less than 300 mOsm? Positive free water clearance (dilute urine) 358.What four characteristics are common to all protein-mediated transportation? 1. More rapid than diffusion 2. Zero-order kinetics 3. Chemical specificity 4. C ompetition for carriers 359.What causes an increase in Na+ and water loss from the kidney by increasing GFR, stimulated by stress and high Na+ concentrations? Atrial natriuretic facto r (ANF) released from the right atrium 360.What growth factors are chondrogenic, working on the epiphyseal end plates o f bone? Somatomedins (insulin-like growth factor type 1[IGF-1]) 361.What causes an increase in Ca+ reabsorption from the distal tubule, a decrea se in PO4 reabsorption from the kidney, and an increase in Ca+ and PO4 reabsorpt ion from the GI tract? Parathyroid hormone (PTH) 362.If a patient is irritated, excited, and emotionally unstable and has overall symptoms of -adrenergic stimulation, would you assume that this patient is hyper thyroidic or hypothyroidic? Hyperthyroidic 363.What is needed for proper postnatal and perinatal mental growth and also for proper bone ossification and GH secretion? Thyroid hormone 364.What type of heart block is associated with slowed conduction through the AV node and PR intervals greater than 0.21 second? First-degree heart block 365.What two occurrences cause an increase in the force of contraction? 1. Incre ase in preload 2. Increase in contractility by increased intracellular Ca+ 366.What are three features of mural regurgitation? 1. Increase in v wave 2. Increase in preload 3. Increase in atrial pressure and volume 367.What is the length of diastole on a pressure curve, and where is it on an EK

G? From the beginning of the IVR to the beginning of the IVC, and from the T wave to the QRS complex (S2 to Sl) 368.What are the three features of aortic regurgitation? 1. Increase in p reload 2. Increase in systolic pressure 3. Decrease in aortic diastolic pressure 369.What are the three features of mitral stenosis? 1. Increase in a wave 2. Decrease in LV filling 3. Increase in atrioventricular pressure 370.What type of heart block is characterized by: Progressive lengthening of the P-R interval until there is failure of the impulse to be transmitted? Second-d egree heart block, Wenckebach (Mobitz type I) 371.What type of heart block is characterized by: Constant P-R interval but with an occasional failure of conduction, resulting in an atrial rate greater than t he ventricular rate? Second-degree heart block, non-Wenckebach (Mobitz type I I) 372.What are the three functions of surfactant? 1. Increased compliance 2. Decre ased surface tension 3. Decreased probability of pulmonary edema formation 373.More negative intrathoracic pressure causes what to happen to systemic venou s return and what to the pulmonary vessels? Promotes systemic venous return into the chest and increases the caliber and volume of the pulmonary vessels 374.What four factors cause the oxygen-hemoglobin dissociation curve to shift to the right? 1. Increased PCO2 2. Decreased pH 3. Increased 2,3-BPG 4. Increa sed temperature 375.What part of respiration, on a pressure volume curve, acts "like the chest w all"? Inspiration (collapse is due to elastic recoil) 376."Secretion + filtration = Excretion" is the transport maximum (Tm) for what substance? te (PAH) Paraaminohippura

377.What has happened if the amount filtered and the amount excreted per unit ti me are the same? Nothing; there has been no tubular modification. 378.What happens to the following parameters in an obstructive versus restrictiv e lung problem: Lung recoil? Decrease (obstructive); increase (restrictive) 379.What happens to the following parameters in an obstructive versus restrictiv e lung problem: FRO? Increase (obstructive); decrease (restrictive) 380.What happens to the following parameters in an obstructive versus restrictiv e lung problem: TLC? Increase (obstructive); decrease (restrictive) 381.What happens to the following parameters in an obstructive versus restrictiv e lung problem: FVC? Decrease (obstructive); decrease (restrictive) 382.What happens to the following parameters in an obstructive versus restrictiv e lung problem: FEVI? Decrease (obstructive); decrease (restrictive) 383.What happens to the following parameters in an obstructive versus restrictiv

e lung problem: Peak flow? Decrease (obstructive); increase (restrictive) 384.What happens to the following parameters in an obstructive versus restrictiv e lung problem: RV? Increase (obstructive); decrease (restrictive) 385.What hormone increases reabsorption of Na+ by the principal cells and promot es excretion of H+ and K+ by the intercalated cells of the kidney? Aldoster one 386.What three situations cause the rennin-angiotensin-aldosterone axis to fire? 1. A decrease in blood pressure in the afferent arteriole 2. Low Na+ levels at t he macula densa 3. 1-Sympathetic nervous system input 387.What pathology is associated with low ACTH levels and high levels of cortiso l? Cushing's syndrome (adrenal) 388.When do you see low urine flow, high urine osmolarity, high ECF volume, low ECF osmolarity (low Na+), high ICF volume, and low ICF osmolarity? SIADH (w ater retention) 389.What is the term for the process in which excretion is less than the filtere d load? Net positive reabsorption (glucose, Na+, urea) 340.What has happened when everything that is filtered is reabsorbed until the c arriers are saturated and the excess is excreted in the urine? The transport ma ximum has been reached (Tm glucose = 375 mg/min). 341.What four factors cause aldosterone to be released? 1. Conversion of angiote nsin I to angiotensin II 2. Hyperkalemia 3. Hyponatremia 4. A decrease in blood volume 342.What disease state includes buffalo hump, moon fades, hyperglycemia, hyperli pidemia, hypertension, hypokalemia, osteoporosis, and thinning of the hair? Cushing's disease 343.What condition involves high urine flow, low urine osmolarity, low ECF volum e, high ECF osmolarity, low ICF volume, and high ICF osmolarity? Diabetes insipidus (lose water) 344.Which condition involves elevated ACTH and cortisol levels? Cushing's diseas e (pituitary tumor) 345.Which condition involves high ACTH, low cortisol, high ADH, elevated renin l evels, hypotension, and low body hair? Addison's disease (primary adrenal insuf ficiency) 346.What process has taken place in the kidney when excretion is greater than th e filtered load? Net negative secretion (PAH, creatinine)

Das könnte Ihnen auch gefallen